PANCE Prep Cardio + Practice Qs

¡Supera tus tareas y exámenes ahora con Quizwiz!

What is the classic CXR findings for tetrology of fallot?

"Boot shaped heart"

Comparision of the *DISEASES OF THE VENOUS SYSTEM* (1) Superficial thrombophlebitis (2) Chronic venous insuff (3) DVT

(1) *SUPERFICIAL THROMBOPHLEBITIS* *Clinical features*: local tenderness, erythema along the course of a superficial vein *Diagnosis*: clinical *Tx*: Analgesics, monitor spread for cellulitis (2) *CHRONIC VENOUS INSUFF* *Clinical Features*: achings of LE worse at the end of the day; relieved by elevation of legs and worsened by recumbency; edema, pigmentation, ulcers *Diagnosis*: Clinical *Tx*: leg elevation, avoid long periods of standing, elastic stockings; if uclers develop unna-boots and wet-dry dressings (3) *DVT* *Clinical Features*: usually asx, calf pain may be present *Diagnosis*: Duplex u/s, D-dimer *Tx*: Anticoagulate

What treatment options are indicated for DVT?

(1) Anticoagulation - heparin bulus followed by warfarin x 3-6 months (2) Thrombotic therpay (-kinase) for pts w/ massive PT, hemodynamically unstable, right sided heart failure (3) Prophylactic IVC filter (greenfeild filter) - used in pts who are c/i for anticoag and thrombolytics - effective ONLY in preventing PE NOT DVT

Name the CHD associated w/ the following conditions: (1) Downs (2) FAS (3) Turners (4) Marfans (5) CHARGE

(1) Downs = VSD/ASD (2) FAS = VSD/ASD (Shot glass up side down and right side up) (3) Turners = COA, Bicupsid Aortic Valve (4) Marfan Syndrome: Mitral Valve Prolapse; Aortic Aneurysm; Aortic Root Dissection (5) CHARGE Association: Ventricular, Atrioventricular, and Atrial Septal Defects

In pts w/ chronic venous insuff, what is the management before a venous ulcer is present?

(1) Leg elevation *ABOVE* the heart during the day and night (2) Avoiding long periods of sitting/standing (3) Heavy-wt elastic stocking (knee-length) are worn during walking

What are the predisposing factors for aortic dissection?

(1) Longstanding systemic HTN (present in 70% of patients) (2) Trauma (3) Connective tissue diseases, such as Marfan's and Ehlers-Danlos syndrome (4) *Bicuspid aortic valve* (5) *Coarctation of the aorta* (6) Third trimester of pregnancy

What are the signs for AS?

(1) Murmur - Harsh crescendo-decrescendo systolic; Heard in second right intercostal space; Radiates to carotid arteries (2) S4 (3) *Parvus et tardus*-diminished and delayed carotid upstrokes (4) Sustained PMI (5) Precordial thrill

What are the target LDL levels for the following pt populations: (1) NO est CHD (2) Est. CHD/DM

(1) NO est CHD: LDL < 130 (2) CHD/DM: LDL <100

What are the complications that can occur due to a DVT?

(1) PE (2) Posthrombotic syndrome (chronic venous insuff) - occurs in 50% of pt and can lead to ambulatory HTN (3) Phlegmasia cerulea dolens - Occurs in extreme cases of DVT-indicates that major venous obstruction has occurrence. Severe leg edema compromises arterial supply to the limb, resulting in impaired sensory and motor function. - venous thrombectomy is indicated

What are the hemorrhagic and nonhemorrhagic causes of hypovolemic shock?

(1) hemorrhagic - trauma, GI bleed, retroperitoneal (2) nonhemorrhagic - voluminous vomiting, severe diarrhea, severe dehydration, burns, third-space losses in bowel obst

What are the clinical features of a MI?

(1) intense substernal CP - "crushing" or "elephant on chest" (2) Radiation to neck, jaw, arms, or back - *MOST COMMON* on *LEFT* side (3) Some experience epigastric discomfort (make sure to r/o w/ GERD sx) (4) Dyspnea (5)diaphoresis (6)weakness, fatigue (7) N/V, sense of impending doom (8) syncope

What patient population can have an asx or atypical MIs?

(1) post-op (2) DM (3) Elderly (4) Women

What is the management if ulcers develop in a pt w/ chronic venous insuff?

(1) wet-dry saline dressing 3x/day (2)Unna Venous boot (ext compression stocking) - changed every 7-10 days (3) For ulcers that do not heals w/ unna venous boot then apply *splint thickness skin graft* w/ or w/o ligation of adjacent perforator veins

What is the classical CXR findings for TAPVC?

*"Snowman"*, cardiomegaly, increased pulmonary vasculature

If the aneurysm is > _____ in diameter or symptomatic, surgerical resection w/ synthetic graft placement is recommended (infrarenal aorta is replaced w/ fabric tube)

*> 5 cm* (normal renal aorta is 2 cm) <5 cm - tx is controversial but routine RAD is recommended

What HTN treatment is indicated for diabetics?

*ACE-I* (or possibly ARB), beta blocker with diuretic

What HTN treatment is indicated for a CHF pt?

*ACE-I*, diuretic, carvedilol, some beta blockers, ARB *NOT CCB*

What drugs are used post-MI for maintance therapy?

*ASA* - decreases mortality - prevents platelet aggregation to prevent thrombosis formation *BB* - decreases mortality - decreases HR/Contractility/afterload *ACEi* - initiate w/in hours of hospitalization if no c/i - been shown to decrease mortality - esp important in *DIABETICS* *Statins* - reduce risk of other coronary events

Name all the Acyanotic and Cyanotic CHD.

*Acyanotic:* Atrial septal defects Patent ductus arteriosus Ventricular septal defect Pulmonary stenosis Aortic stenosis Coarctation of aorta *Cyanotic* (Remember: Cyanotic 5Ts and H!!!!) TETRALOGY OF FALLOT TRICUSPID ATRESIA TRANSPOSITION OF THE GREAT ARTERIES TRUNCUS ARTERIOSUS TOTAL ANOMALOUS PULMONARY VENOUS CONNECTION HYPOPLASTIC LEFT HEART SYNDROME

What is the management option for a non-hemodynamically stable sustained VT pt?

*BOX TRUMPS PILL* IMMEDIATE synchronous DC cardioversion Follow w/ Amiodarone to maintain sinus rhythm

What are the EKG findings for V.Fib?

*COMPLETELY IRREGULAR* - No atrial P waves or QRS complexes can be IDed - in sum NO WAVES can be IDed

What is used for diagnosis of endocarditis?

*DUKE CRITIERIA* MUST HAVE: Two Major; One Major and Three Minor; OR Five Minor *Major* (1) 2 positive blood cultures (2) Evidence on echocardiogram of: Vegetation; Abscess, OR Dehiscence of a prosthetic valve *Minor* (1) Predisposing condition (2) Fever >38 degrees C (3) Immunologic signs (*glomerulonephritis, Osler's nodes, Roth spots, rheumotoid factor*) (4) 1 positive blood culture (5) Positive echo not meeting major criteria

What is the *classical* CXR finding for "TOGA"?

*EGG ON A STRING* Other: ↑ pulm vasculature; narrow mediastinum, cardiomegaly

What EKG findings are characteristic of cardiac tamponade?

*ELECTRICAL ALTERNANS* (alternate best variation in the direction of the EKG wave forms) - due to pendular swinging of the heart within the pericardial space, causing a motion artifact

What is the treatment for hemorrhagic tamponade secondary to trauma?

*EMERGENT SURGERY* is indicated to repair the injury Pericardiocentesis is only a temporarly solution, and surgery should *NOT* be delayed to perform pericardiocentesis.

What are the "Big 6" in regards to risk factors for CAD?

*FSH DAD* F - Family Hx of premature CAD/MI (M < 45; F < 55) S - Smoking (Cigarettes) H - HTN, Homocysteine levels elevated D - DM/Metabolic syndrome A - Age (M: > 45; F: > 55) D - Dyslipidemia - Elevated LDL; Low HDL

What diagnostic test is as accurate as doppler but is less operator dependent?

*Impedance plethysmography* • A noninvasive alternative to Doppler ultrasound • Blood conducts electricity better than soft tissue, so electrical impedance decreases as blood volume increases. • High sensitivity for proximal DVT, but not for distal DVT (calf veins) • Poor specificity because there is a high rate of false positives

What is the effects of niacin?

*LOWER TGs*, lowers LDL, increases HDL 2nd line for lowering LDL

What is the *MOST COMMON* findings for cardiac tamponade?

*MC = Elevated JVP* - prominent x-descent w/ absent y descent

Where is the *MOST COMMON* location for acute arterial occlusion?

*MC* = femoral artery Less commonly = in situ thrombus

What is the *MOST COMMON* site for occulusion/stenosis in PVD?

*MC* = superficial femoral artery (in *hunter's canal*) Other locations: popliteal artery, aortoiliac occlusive disease

What is the *MOST COMMON CAUSE* of embolus from the heart?

*MCC* = AFib Other: post-MI, endocarditis, myxoma

What is the *MCC* of cardiogenic shock?

*MCC* = After acute M Other causes: cardiac tamponade, tension pneumo, arrhythmias, massive PE leading to RVF, CM, myocarditis

What is the *MOST COMMON CAUSE* for Acute Endocarditis?

*MCC* = Staph Aureus

Describe the "classical" murmur associated w/ MVP.

*MIDSYSTOLIC OR LATE SYSTOLIC CLICK* *MID - LATE SYSTOLIC MURMUR* Standings and Valsalva *increase* the murmur/click (b/c it reduces LV chamber size) Squatting *decreases* murmur and click (because it increases LV chamber size) Sustained handgrip *increases* MVP; in contast to the murmur of HCM with *decreases*

What is the *HALLMARK* of the PDA murmur?

*Machinary Type Murmur* S&S: Acyanotic, Early diastolic murmur + crescendo-decrescendo holosystolic/ pansystolic murmur, Sweating w/feeding, Bounding peripheral pulses, Wide pulse pressure

What is the medical and surgical management for MR?

*Medical* a. Afterload reduction with vasodilators; also salt reduction, diuretics, digoxin, and antiarrhythmics b. Chronic anticoagulation if patient has atrial fibrillation c. IABP as bridge to surgery for acute MR *Surgical* - Mitral valve repair or replacement (Must be performed before left ventricular function is too severely compromised)

Explain orthodromic and antidromic AVRT.

*Orthodromic AVRT (MORE COMMON):* Narrow Complex tachycardia in which the wave of depol travels down the aVnode and retrograde up the accessory pathway (narrow QRS - bypass going down normal conduction pathways) - Retrograde *Antidromic AVRT (LESS COMMON):* Wide Complex tachycardia in which wave of depol travels down the accessory pathway and retrograde up the AV node (The wider the QRS the farther the conduction is from the std conduction pathways and it is slower) - Anterograde

What are the 6Ps for acute arterial occlusion?

*P*ain (acute onset) *P*allor *P*olar (cold) *P*aralysis *P*aresthesis *P*ulselessness (use doppler)

Compare PVD and Acute Arterial Occlusion:

*PVD* Clinical features: intermittent claudication, rest pain, decreased pulses, ischemic ulcers Dx: Arteriogram Tx: Intermittent claud - conservative, Rest pain - surgery *Acute Arterial Occlusion* Clinical Features: 6Ps Dx: arteriogram Tx: Anticoagulate, emergent surgery

What is the EKG findings for A. Flutter?

*Pathogneumonic* - "saw-tooth" flutter waves best seen in II/III/avF Rate will be 250-350

What are the characteristic EKG findings for a MI?

*Peaked T waves* - occur EARLY and may be missed *ST Elevation (STEMI)* - TRANSMURAL MI - diagnostic of an acute infarct *Q Waves* - evidence of NECROSIS (SPECIFIC) - usually seen LATE, not acutely *T wave inversion* - sensitive NOT specific *ST Depression (NSTEMI)* - SUBENDOCARDIAL injury

What is the characteristic CXR finding for COA?

*RIB NOTCHING* Others: Cardiomegaly; normal pulm vasculature; dilatation

What are the symptoms of AS?

*SAD* - Syncope, Angina, DOE

What is the *TOC* for all valvular heart diseases?

*Surgery* - because these are "structural deformities" that cannot be fixed w/ drugs.

What is the *DEFINITIVE TREATMENT* for constrictive pericarditis?

*Surgical*: Complete resection of the pericardium is definitive therapy and is indicated in many patients. It has a significant mortality rate, however.

Explain the management of V.Fib.

*THIS IS A MEDICAL EMERGENCY! IMMEDIATE DEFIBRILLATION AND CPR ARE INDICATED* Give EPI (1 mg IV initially and then every 3-5 min) (increase Myocardial/cerebral blood flow and decreases the defibrillation threshold) then attempt to defibrillate 30-60 seconds both first epi dose.

What is the *SOC* to evaluate both the location and size of the AAA?

*ULTRASOUND*

What is neurogenic shock characterized by?

*peripheral vasodilation and decreased SVR* Neurogenic shock results from a failure of the sympathetic nervous system to maintain adequate vascular tone (sympathetic denervation) . Causes include spinal cord injury, severe head injury, spinal anesthesia, pharmaco- logic sympathetic blockade

How can you diagnose multifocal atrial tachycardia?

*three different P-wave morphologies are required to make an accurate diagnosis* Rate is 100-200 Can also be diagnosed w/ vagal maneuvars or adenosine to slow AV block **Note that a wandering pacemaker is similiar but the rate is <100**

What is the pathophys for cardiac tamponade?

*ventricular filling is impaired during diastole* - decreases SV and CO Characterized by the elevation and equalization of intracardiac and intrapericardial pressure

What is the classifcal CXR finding for aortic dissection?

*widended mediastium* (>8 mm on AP view)

If a patient presents with a severe HA and markedly elevated BP, what steps should you take in management?

1) Lower BP w/ Antihytensive Agent 2) order a CT of the head to r/o intracranial bleeding (SAH) 3)if CT is negative, one may proceed to a lumbar puncture

What three patient profiles can be diagnosed with unstable angina (USA)?

1) Pts with chronic angina with increasing freq, duration, and intensity of CP 2) Pts with new-onset angina that is severe or worsening 3) Patients w/ *ANGINA AT REST*

What TLC can be suggested for a HTN pts?

1) Reduce Salt Intake - recommended is a no added salt diet or DASH diet (<4g Na+/day) or a low sodium diet (<2g Na+/day) 2) Wt loss and exercise 3) AVOID alcohol consumption 4) Stop unncessary meds that can lead to HTN 5) Follow a low saturated fat diet rich in fruits, veggies, and low fat dairy

What are the risk factors for HTN?

1. Age-Both systolic and diastolic BP increase with age. 2. Gender-*more common* in men (gap narrows over age 60); men have higher complication rates 3. Race-It is *twice as common* in African-American patients as in Caucasian patients; African-American patients have* higher complication rates* (stroke, renal failure, heart disease). 4. Obesity, sedentary lifestyle 5. Family history 6. Increased sodium intake-This correlates with increased prevalence in large populations, although not in individuals; individual susceptibility to the effects of high salt intake varies. 7. Alcohol-Intake of more than 2 oz (8 oz of wine or 24 oz of beer) per day is associated with HTN.

What is the main goal and treatment obtions for acute arterial occlusion?

1. Main goal: assess viability of tissues to salavage the limb a. Skeletal muscle can tolerate 6 hours of ischemia; perfusion should be reestab- lished within this time frame. b. If paralysis or paresthesias are present, amputation is probably necessary. 2. Immediately anticoagulate with IV heparin. 3. Emergent surgical embolectomy is indicated via cutdown and Fogarty balloon. Bypass is reserved for embolectomy failure. 4. Treat any complications such as compartment syndrome that may occur.

What are the S&S for aortic dissection?

1. Severe, tearing/ripping/stabbing pain, either in the anterior or back of the chest a. Anterior chest pain is *more common* with proximal dissection (type A). b. Interscapular back pain is *more common* with distal dissection (type B). 2. Diaphoresis 3. Most are hypertensive, but some may be hypotensive 4. Pulse or BP asymmetry between limbs 5. Aortic regurgitation (especially proximal dissections) 6. Neurologic manifestations (hemiplegia, hemianesthesia) due to obstruction of carotid artery

Explain the process of "Shock"

1. Shock is severe cardiovascular failure caused by poor blood flow or inadequate distribution of flow. 2. Inadequate oxygen delivery to body tissues results in shock, which may lead to organ failure and death. 3. The physical responses to shock are mediated by catecholamines, renin, antidiuretic hormone, glucagon, cortisol, and growth hormone

What is the *MOST COMMON* location for an abdominal aortic aneurysm (AAA)?

Renal Arteries and Iliac Bifucation

What are the S&S of a non-rupture AAA?

1. Usually asymptomatic and discovered on either abdominal examination or a radio- logic study done for another reason 2. Sense of "fullness" 3. Pain may or may not be present-if present, located in the hypogastrium and lower back and usually throbbing in character 4.Pulsatile mass on abdominal examination 5.Symptoms suggesting expansion and impending rupture include the following: a. Sudden onset of severe pain in the back or lower abdomen, radiating to the groin, buttocks, or legs b. Grey Turner's sign (ecchymoses on back and flanks) and Cullen's sign (ecchymoses around umbilicus)

Pts with MS are usually asx until the MV area is reduced to approximately __________ cm2.

1.5 cm2 Normal = 4-5 cm2

What is the *MOST COMMON CAUSE* of secondary HTN?

Renal Artery Stenosis

What is the *MOST COMMON* age to find a venous hum?

3-6 y.o

In regards to A. Fib and rate control, what is the target rate? Perferred Tx?

60-100 bpm CCB - Perferred Tx (Alt: BB)

What is the numerical value for TGs that designates pharmacological intervention?

>500 mg/dl

What are the risk factors for AAA?

>65 Male Smoker

Explain the pathophys of constrictive pericarditis and note the defining difference btween constrictive pericarditis and cardiac tamponade.

A fibrotic, rigid pericardium restricts the diastolic filling of the heart. Ventricular filling is *unimpeded* during early diastole because intracardiac volume has not yet reached the limit defined by the stiff pericardium. When intracardiac volume reaches the limit set by the noncompliant peri- cardium, ventricular filling is halted abruptly. (In contrast, ventricular filling is *impeded throughout diastole* in cardiac tamponade.)

How often should people be screened for dyslipidemia?

A lipid profile should be performed every 5 years starting at 20 y/o

Describe the murmur of MR?

A. *Holosystolic* murmur at the *apex*, which radiates to the back or clavicular area, depending on which leaflet is involved B. AFib is common finding C. Other findings: *S3* gallop; laterally displaced PMI; loud, palpable P2

What EKG finding is defined as "irregularly irregular"?

A. Fib (Atrial rate is > 400 and vent rate is 75-175) *MOST COMMON ARRYTHMIA*

What is "holiday heart syndrome"?

A. Fib caused by excessive alcohol intake

If pt presnts with the classic symptoms of shock + trauma, GI bleed, vomiting, or diarrhea, what type of shock is most likely? A. Hypovolemic B. Septic C. Neurogenic D. Cardiogenic

A. Hypovolemic

Name the NYHA described..."symptoms occur w/ *vigorous activities*, such as playing a sport, pts are nearly *asymptomatic*? A. I B. II C. III D. IV

A. I

Which of the following abnormalities is NOT presents in tetrology of fallot? A. LVH B. Pulmonary Stenosis C. VSD D. RVH

A. LVH Remember...*"PROV"* *P*ulmonary Stenosis *R*VH *O*veriding Aorta *V*SD

If a pt comes in with a BP of 110/70, what is their classification of HTN? A. Normal B. PreHTN C. Stage 1 HTN D. Stage 2 HTN

A. Normal (<120/80) Requires NO treatment

Septic shock is associated w/ severe peripheral vaso_______. Whereas hypovolemic shock is associated w/ peripheral vaso_______. A. vasodilation, vasoconstriction B. vasoconstriction, vasodilation

A. Septic shock causes severe peripheral vasodilation (flushing, warm skin) whereas hypovolemic shock is assocaited w/ peripheral vasoconstriction (cool skin)

Define the following arrhythmia description: "Absence of conduction of atrial impulses to the ventricles; no correspondence between P waves and QRS complexes. A ventricular pacemaker (escape rhythm) maintains a ventricular rate of 25 to 40 bpm" A. Third Degree (Complete) Heart Block B. Mobitz type II C. Winkebach D. First defree AV block

A. Third Degree (complete) heart block -Characterized by *AV DISSOCIATION* -Pacemaker impletation is necessary

What management should you take in a pt w/ hypovolemic shock?

ABCs Hemorrage --> direct pressure, IVF, PRBCs Nonhemorrhagic --> IVF (LR/NS)

What is the tx for cardiogenic shock?

ABCs Tx underlying cause Vasopressors - *DOPAMINE = TOC* -dobutamine w/ dopamine for futher increased in CO -NE/Phenyephrine for severe/resistance cases NTG is not generally indicated b/c of the hypotension (but give MOA) *IVF ARE LIKELY TO BE HARMFUL IF LV PRESSURE ARE ELEVATED*

What is the management for HCM patients?

ALL patients *AVOID* strenuous exercise *FIRST LINE TX* = BB (2nd line = CCB) Diuretics if fluids retention Pacemaker Thromboembolism/endocarditis prophylaxis Surgery - Myomectomy has high success rate for relieving sx - excision of part of the myocardial septum (reserved for severe disease) AVOID Dig, B-Agonist, Nitrates

Pt presents with a diastolic decresendo murmur best heard at the left sternal borders with a widened pulse pressure and a displaced PMI (down&left). What type of murmur is this?

AR

What cardiac abnormaliities that are associated w/ Eisenmengers?

ASD, VSD, TA, TOGA

What phenomenon is described as follows... ABERRANT VENTRICULAR CONDUCTION DUE TO A CHANGE IN QRS CYCLE LENGTH. Describes a particular type of wide complex tachycardia that is often seen in atrial fibrillation. It is more often misinterpreted as a premature ventricular complex.

ASHMAN'S PHENOMENON

What are the EKG findinds for HCM?

Abnml Q waves in inferior and apical leads, may have evidence of LVH

What are the clinical features of V. Fib?

Absent BP, Pulse, Heart Sounds No QRS complex IDed Untx --> Leads to "sudden cardiac death"

Explain WPW and the Tx?

Accessory pathways between the atrium and ventricle may occasionally be seen *MOST COMMON* is 'bundle of Kent' seen in Wolff-Parkinson-White syndrome *SHORTENED PR INTERVAL AND A "DELTA WAVE"* caused by early excitation of the ventricle via the accessory pathway. TX: Radiofreqency catheter abalation

Which form of endocarditis is associated w/ a "normal heart valve"? A. Acute B. Subacute

Acute If untx, fatal in less than 6 weeks

What is the *MOST COMMON CAUSE* of a MI?

Acute Coronary Thrombosis Remember time is muscle w/ an MI o 20-40 min: irreversible cell damage o 3-6 hours: necrosis

What are the causes of ACUTE and CHRONIC MR?

Acute: (Higher Mortality) (1) Endocarditis (*MC* = Staph Aureus) (2) Papillary mm rupture (via MI) or dysfunction (form ischemia) Chronic: (1) Rheumatic Fever (2) Marfan Syndrome (3) CM

What are the S&S for AS in children?

Acyanotic, SEM thrill loudest at RUSB, Narrow pulse pressure *MCC* is Rheumatic fever, Congenital bicuspid valve

What is the initial LT treatment for moderate-severe CHF (III/IV)

Add Dig to loop and ACEi Can add Dig at anytime for *systolic dysfunction*

What management should be taken with USA?

Admit w/ IV access and O2 Medical Management: *"HANG"* Heparin (LMWH) Aspirin NTG - FIRST LINE THERAPY Glycoprotein IIb/IIa + BB - FIRST LINE THERAPY + Cardiac Cath

What prophylaxis should you use for endocarditis in patients w/ *known valvular heart disease or prosthetic valves* who are about to undergo *oral surgery or GI/GU* procedures?

Amoxicillin

A 57-year-old man with advanced lung cancer presents with 24 hours of increasing dyspnea and pleuritic chest pain. He has no previous history of cardiac disease. On examination, blood pressure is 85/40 mmHg and heart rate is 120 beats/minute. The neck veins are elevated above the clavicle in the sitting position. There are no murmurs or extra sounds. The breath sounds are decreased in the right base. A right heart catheter is inserted from the right internal jugular vein, and the following pressures are obtained: Location Pressure (mmHg) Right Atrium 21 (Normal 0-5) Pulmonary Artery 45/20 (Normal 13-28/3-13) Pulmonary Capillary Wedge 21 (Normal 3-11) Which of the following is the correct diagnosis? A) Cardiac tamponade B) Pulmonary embolism C) Pneumonia D) Constrictive pericarditis E) Acute myocardial infarction

Answer A. Cardiac tamponade Explanation This patient with lung cancer has a brief history of increasing dyspnea, has hypotension, and elevated neck veins. Right heart catheterization reveals elevated and equalized diastolic pressure (or close enough RA 21, PA 20, PCWP 21), typical of cardiac tamponade. Several cancers, most frequently carcinomas of the lung and breast and various lymphomas, are characterized by a high incidence of pericardial metastases and pericardial effusion can accumulate rapidly. Neck veins demonstrate gross elevation with a single rapid x descent, and the arterial blood pressure has an inspiratory fall of more than 10 mmHg.

A 55-year-old man received tissue plasminogen activator for an ST segment elevation myocardial infarction (STEMI). Three days later, he had an episode of similar chest pain, relieved in 10 minutes by sublingual nitroglycerin. Physical examination demonstrates a pericardial friction rub. An ECG shows nonspecific ST-T wave changes. Which of the following procedures should be done next? A) Coronary angiography B) Treatment with nonsteroidal antiinflammatory drugs C) An exercise stress test with nuclear perfusion imaging D) An adenosine stress test with nuclear perfusion imaging E) Treatment with prednisone

Answer A. Coronary angiography Explanation This patient has recurrent chest pain that is similar to his original symptoms and is relieved by nitroglycerin. Recurrent symptoms after reperfusion terapy warrants coronary angiography.

You are seeing a 22-year-old for the first time for a pre-work physical exam. The patient reports he has had a few ear infections but is otherwise very healthy. His immunizations are up-to-date. On physical examination, his vital signs are normal. Everything appears fine except you hear a systolic ejection murmur best heard at the left upper sternal border. Which of the following would make you suspect an atrial septal defect? A) Fixed split second heart sound B) Midsystolic click C) Early systolic ejection click D) Cyanosis E) Greater arterial pulses in the upper extremities compared to the lower

Answer A. Fixed split second heart sound Explanation Most children and young adults with an ASD are asymptomatic, although large defects can cause fatigue and rarely CHF. Clinical findings of an ASD consist of pulmonary ejection systolic murmur best heard at left upper sternal border, widely fixed split S2 (due to delayed closure of the pulmonic valve), and early to mid-diastolic rumble at the lower left sternal border (due to increased flow across tricuspid valve).

A 23-year-old male migrant worker presents to the emergency department with complaints of shortness of breath and fatigue. The symptoms have been progressively worse over the last 2-3 years, but he has deferred seeking medical help due to difficulties with his immigration status. On examination, the patient is dyspneic at rest without labored breathing. His lungs are clear with good air entry. There is mild elevation in the JVP and a jugular venous pulse wave is noted. Cardiac exam demonstrates a normal S 1 with a loud pulmonary component of the second heart sound and a regular rhythm. There is a soft systolic murmur at the left sternal border. Palpation of the chest reveals a right ventricular heave. Pulses in the extremities are present. There is cyanosis of the toes, but not of the fingers. Clubbing is noted. Right ventricular strain pattern was evident on electrocardiogram. Enlarged pulmonary arteries with no infiltrates were noted on chest radiography. Cardiac silhouette is suggestive of right atrial and right ventricular hypertrophy. Which of the following is most likely to be found on cardiac catheterization? A) Patent ductus arteriosus (PDA) B) Ventricular septal defect (VSD) C) Pulmonic stenosis (PS) D) Aortic stenosis (AS)

Answer A. Patent ductus arteriosus (PDA) Explanation Eisenmenger syndrome is the development of pulmonary hypertension secondary to increased pulmonic blood flow and arises from a left-to-right intracardiac shunt. Over time, the increased flow through the pulmonary vascular beds increases resistance to the point where flow reverses to a right-to-left shunt. This shunt causes significant diversion of unoxygenated blood to the systemic circulation. This can occur with shunts through an atrial septal defect (ASD), a VSD, or a PDA. A VSD can lead to Eisenmenger syndrome, but the cyanosis is more generalized. Critical PS can lead to difficulty with pulmonic blood flow and can demonstrate signs of right heart failure. Similarly, a critical aortic stenosis would exhibit signs of left heart failure. However, neither isolated PS nor isolated AS is associated with cyanosis. A PDA accounts for the findings noted including the pattern of cyanosis. The differential cyanosis with duskiness to the toes but not to the hands occurs due to the distal insertion of a patent ductus into the aorta. The classic machinery murmur of an untreated PDA is typically altered when the flow of blood is reversed, and there may actually be no murmur at all.

A 50-year-old woman comes in for her routine checkup. She has not had any problems and relates that she is doing well, except she says that her feet swell frequently and that this has just started to occur. PAST MEDICAL HISTORY: Essentially negative; delivered 3 children by normal, spontaneous vaginal delivery at 28, 30, and 35 years ago MEDS: "Menopause pill" SOCIAL HISTORY: Lives with her husband and son age 35 Drinks a 6-pack of beer/week Smokes 2 packs/cigarettes daily Works as a weather forecaster FAMILY HISTORY: Mother with CHF diagnosed at age 50 Father with CHF diagnosed at age 50 Brother age 51, good health REVIEW OF SYSTEMS: No shortness of breath Chronic cough especially in the morning for years No edema of the hands or elsewhere noted No orthopnea No dyspnea on exertion or at rest No weight gain or weight loss PHYSICAL EXAMINATION: Ht 5' 5", Wt 120, BP 110/80, Temp 98° F, RR 12, P 88 HEENT: PERRLA, EOMI, discs sharp TMs clear Throat clear Neck: Supple, no masses Jugular venous pulse is 3 cm H2O Hepatojugular reflux is negative Heart: RRR without murmurs, rubs, or gallops Lungs: CTA Abdomen: Bowel sounds present; no ascites; no masses Extremities: 2+ pitting edema to just past the ankle area bilaterally No cyanosis, clubbing noted Which of the following should not be considered in the differential? A) Right heart failure B) Venous varicosities C) Hypoalbuminemia D) Pelvic thrombophlebitis E) Cyclic edema

Answer A. Right heart failure Explanation Ankle edema does not equal heart failure! Look at her physical examination: She has no jugular venous distention—it is very, very unlikely for her to have heart failure with normal jugular pressures. All of the other items listed should be considered before you even think about right heart failure. Also remember that calcium channel blockers (e.g., amlodipine, nifedipine, diltiazem) used for treating HTN can also cause lower extremity edema by blunting postural cutaneous vasoconstriction.

An unidentified male, in his 30s, is recovered after falling through the ice at the local park. He is believed to have been submerged for 20-30 minutes. EMS was called and found him in asystole. Emergency resuscitation was instituted. The patient was intubated; CPR was continued, and he was brought to the local emergency department. On physical examination, there is no pulse, no spontaneous respirations, and his core body temperature is 80° F (26.7° C). Cardiac compressions and external ventilation have been in progress for 20 minutes without spontaneous resumption of cardiac conduction or respirations. What is the best next appropriate step in the treatment of this patient? A) Obtain an urgent neurology consultation to assist with a brain death evaluation. B) Aggressively treat the hypothermia and continue with resuscitation efforts until core temperature is > 33° C (91.4° F). C) Due to the extended period of non-responsiveness, pronounce the patient dead. D) Transfer the body to the operating room in preparation for organ harvest.

Answer B. Aggressively treat the hypothermia and continue with resuscitation efforts until core temperature is > 33° C (91.4° F). Explanation Terminating resuscitation efforts in a patient who has experienced hypothermia is a difficult clinical decision. The ability of physiological functions to resume at low core body temperatures is very restricted. Treatment of the hypothermia can raise the core temperature, along with the cellular responsiveness to interventions, and increases the potential for success. Unless the core temperature is returned to at least 33° C, cardiac resuscitation should be abandoned only after careful consideration. Brain death evaluation is most appropriate after cardiac and pulmonary statuses have been stabilized and not in the midst of CPR. Organ harvest in an unidentified patient is inappropriate and should not be implemented until every reasonable effort toward resuscitation has been exhausted.

A 23-year-old man is referred to you for evaluation of a heart murmur that was found after an episode of ventricular tachycardia. He also has noted to have "fleeting" chest pains. Your cardiac examination shows a regular rate and rhythm with a normal sounding S 1 and S 2 . You note an S 4 as well. He has a systolic ejection murmur, which has the following characteristics: It increases with standing and decreases with a sustained handgrip. Palpation of his chest reveals a "double tap" over the cardiac apex. Which of the following agents could be used for therapy? A) Fluid restriction and volume depletion B) Amiodarone C) Diuretics D) Sustained release nitrates E) Anticoagulants

Answer B. Amiodarone Explanation He has hypertrophic cardiomyopathy (HCM). The initial problem in this disorder is diastolic dysfunction. Also, patients with HCM have a tendency for ventricular tachycardias. Young patients with the familial form of the disease are at highest risk for sudden death. Remember that HCM and mitral valve prolapse are the only conditions that have a murmur that increases with standing. To differentiate between the two, do the hand-grip maneuver—in HCM it decreases the murmur and in MVP it increases the murmur. Anything that decreases left ventricular volume is dangerous in HCM; thus, diuretics, nitrates, and volume depletion are not a good idea. Anticoagulants have no place in therapy. Amiodarone is useful to prevent ventricular arrhythmias.

You begin evaluation of a 27-year-old male who presented to the emergency department with severe chest pain that started abruptly 3 hours previously. The pain was in the anterior central chest and lasted for 1-2 hours. He experienced light-headedness but no diaphoresis. The pain has currently improved to a moderate ache. On examination, the patient is distressed and appears to be mildly uncomfortable. He is wearing glasses secondary to a lens displacement that is being followed by ophthalmology. His heart rate is 94/min, respiratory rate is 12 breaths/min, and blood pressure is 142/88 mmHg. The patient is afebrile. His height is 6' 6" (198.1 cm), and he weighs 157 lbs (71.4 kg). Exam of the chest reveals moderate pectus excavatum. He has a regular rhythm and is noted to have a soft systolic murmur at the apex with a late systolic click. Lungs are clear with good air entry. No JVD is present. Peripheral pulses are present. There is no swelling or tenderness in the extremities. There is widening of the mediastinum noted on an x-ray of the chest. ECG demonstrates no acute changes or dysrhythmias. Which of the following is the most likely etiology of this person's chest pain? A) Mitral valve prolapse (MVP) B) Aortic dissection C) Acute myocardial infarction (MI) D) Pulmonary embolism (PE) E) Acute pericarditis

Answer B. Aortic dissection Explanation Marfan syndrome is characterized by: 1) musculoskeletal findings including a tall, thin habitus, spider-like fingers and long extremities, and often with chest and spinal abnormalities; 2) lens displacement (ectopia lentis) with resulting vision disturbances; and 3) aneurysmal disease, most commonly in the aortic root. With this person's physical findings and history of pain consistent with a dissecting aneurysm, prompt evaluation of the thoracic aorta is indicated. Mitral valve prolapse is common in Marfan syndrome but would not account for the acute symptoms, unless it was associated with a dysfunction of the chordae tendineae or papillary muscles. In these circumstances, pulmonary complaints and findings would be much more prominent. Patients with Marfan syndrome are not exempt from acute pericarditis, myocardial infarction (MI), or pulmonary embolism (PE), but are not at increased risk either. A normal respiratory rate and lack of evidence of thrombotic disease make the diagnosis of PE less likely. The patient lacks a rub on auscultation; the pain is not positional, presented acutely, and has improved without intervention, which makes pericarditis also less likely. The ECG findings are not suggestive of cardiac ischemia. Though it is less likely in this scenario, myocardial ischemia is a common and serious process, and evaluation and intervention for ischemia should proceed concurrently with the evaluation for aortic dissection.

Which of the following cardiac conditions tolerates exercise well? A) Aortic stenosis B) Aortic regurgitation C) Hypertrophic cardiomyopathy D) Eisenmenger syndrome E) Mitral stenosis

Answer B. Aortic regurgitation Explanation Aortic regurgitation tolerates exercise well because exercise does 2 good things: 1) dilates the peripheral vasculature, allowing more blood to move forward, and 2) produces tachycardia, which shortens the time for regurgitation during diastole. Exercise is contraindicated in aortic stenosis, hypertrophic cardiomyopathy, and in patients with Eisenmenger syndrome. Exercise is not dangerous in patients with mitral stenosis, but it is not well tolerated.

An 80-year-old man has had recent syncope that appears to be exercise-induced. He is now hospitalized for congestive heart failure. He has developed progressive shortness of breath for the past four days and increasing pedal edema. He tried increasing his usual dose of furosemide, without relief. Past Medical History: No prior hospitalizations for CHF History of coronary artery disease; coronary bypass grafting 5 years ago Hypertension for 30 years; currently on fosinopril 20 mg q day and furosemide 20 mg q day NIDDM; diet controlled Social History: Widowed 4 years ago Lives alone in a trailer Drinks 2 six-packs/beer a week for 30 years Smokes 1/2 pack a day of cigarettes; occasional cigar Veteran of WWII Family History: Mother and Father died at early ages of unknown causes Brother died at age 68 of MI Sister died at age 68 of MI 3 sons; all with hypertension Review of Systems: Deferred for this question (whew, aren't you glad? So is the question writer!) Physical Examination: Well-developed, well-nourished man in mild distress BP 163/80 mmHg, P 80, RR 18, Temp 98.4° F HEENT: PERRLA, EOMI, bilateral cataracts (mild) TMs: Clear Throat: Clear Neck: Supple, +JVD to 8 cm Heart: RRR with III/VI harsh systolic murmur Lungs: Bilateral crackles to mid lung fields Abdomen: Bowel sounds present; no fluid wave; liver down about 5 cm below right costal margin Extremities: No cyanosis; mild clubbing present; 3+ pitting edema to knee GU: Mild scrotal edema Laboratory: Echocardiogram: Disproportionately thickened ventricular septum and systolic anterior motion of the mitral valve Based on your findings, which of the following is likely to be present? A) Delayed carotid upstroke B) Decrease of the murmur with handgrip C) Reduced left ventricular ejection fraction D) Radiation of the murmur to the carotids E) Signs of mitral stenosis

Answer B. Decrease of the murmur with handgrip Explanation His echocardiogram showing evidence of a disproportionately thickened ventricular septum and systolic anterior motion of the mitral valve strongly suggests hypertrophic cardiomyopathy, also known as idiopathic hypertrophic subaortic stenosis (IHSS). The harsh systolic murmur that he has should not radiate to the carotids with this condition. Additionally, the murmur will decrease when he applies a handgrip—this will increase ventricular volume due to isometric exercise. The carotid upstroke is brisk and frequently is bifid. His congestive failure is likely occurring because of the reduced ventricular compliance despite normal left ventricular systolic function. Mitral regurgitation is much more likely than is mitral stenosis.

A 19-year-old female collapses and dies during a sprint at a track meet. She was previously healthy and had no abnormalities on her school's routine physical examination for athletes. Unfortunately, of autopsies done on individuals like this woman, a majority show no abnormalities. If an abnormality is found at autopsy, which of the following is most likely? A) Large mitral valve leaflets with infiltration of myxomatous material on microscopic examination B) Hypertrophic cardiomyopathy C) Severe pulmonic stenosis D) An anomalous origin of the left anterior descending coronary artery from the right coronary cusp

Answer B. Hypertrophic cardiomyopathy Explanation There has been heightened awareness of sudden death of young people partaking in strenuous exercise after several highly publicized cases. The largest series of autopsies in these individuals have shown no abnormalities in the majority of patients. Death was presumably due to a ventricular arrhythmia from an unrecognized source, possibly a subclinical cardiomyopathy or an unknown genetic disorder. If any abnormality is found on autopsy, hypertrophic cardiomyopathy is the most common, followed by anomalous origin of the coronary arteries.

A 67-year-old man with Type 2 DM and history of myocardial infarction 6 years ago is found to have a pulsatile abdominal mass on exam. Ultrasound reveals a 7-cm aortic aneurysm that requires surgery. He walks 8-10 blocks every day without symptoms. Exam: BP: 120/70 mmHg, pulse: 90 bpm Chest: Clear Cardiac: Irregular rhythm, normal S1 and S2, no murmur ECG: Atrial fibrillation with PVCs Laboratory: Hct: 38% Glucose: 160 mg/dL Which of the following would be the most appropriate next step? A) Postpone surgery. B) Risk assess with dipyridamole thallium stress test. C) Risk assess with cardiac catheterization. D) Risk assess with exercise treadmill test. E) Proceed with surgery.

Answer B. Risk assess with dipyridamole thallium stress test. Explanation This patient is undergoing a high-risk procedure, and both AHA and ACP protocols define him as a high-risk patient. Both protocols recommend stress testing prior to surgical planning. A functional test would be more useful than going straight to catheterization, and due to the presence of atrial fibrillation, pharmacologic myocardial perfusion imaging is appropriate. (Also, an exercise treadmill test alone will not provide the perfusion imaging that is necessary in this case.) This high-risk patient with atherosclerotic disease undoubtedly has CAD, given his past MI. The key question to answer: Is the myocardium at risk?

A hypertensive patient presents to you with concentric left ventricular hypertrophy and a left ventricular ejection fraction of 80%. He is on digoxin and furosemide. Which of the following is the best therapy for such a patient? A) Continue digoxin and furosemide; add an angiotensin-converting enzyme inhibitor, low-dose carvedilol, and spironolactone. B) Stop digoxin, decrease the dose of furosemide, and start beta-blockers. C) Admit him for infusion of intravenous amrinone. D) Continue digoxin and furosemide and schedule coronary angiography. E) Continue digoxin and furosemide; add an angiotensin-converting enzyme inhibitor, low-dose carvedilol, and spironolactone; and schedule him for intermittent outpatient dobutamine infusion.

Answer B. Stop digoxin, decrease the dose of furosemide, and start beta-blockers. Explanation This man needs negative inotropic therapy with beta-blockers or verapamil. He may need some diuretic, but not enough to produce volume depletion and postural hypotension. He would need the therapy "Continue digoxin and furosemide; add an angiotensin-converting enzyme inhibitor, low-dose carvedilol, and spironolactone" if he had poor left ventricular function. Digoxin has no place in the treatment of a patient with hyperdynamic left ventricular function. The 2 intravenous inotropic agents are occasionally used for severe end-stage systolic heart failure, although they do not have a Class I recommendation for this use.

If you find an increased PT, a normal PTT, and a normal platelet count, it would indicate which one of the following types of deficiency or abnormality? A) Factor XIII deficiency B) Warfarin administration C) Anti-platelet antibody D) Factor I deficiency E) Factor X deficiency

Answer B. Warfarin administration Explanation Increased PT with a normal PTT and platelets usually indicate warfarin administration. Factor VII deficiency could do this also.

A 27-year-old woman presents for a routine examination. She reports no health problems and has no significant family history. She does not use drugs, tobacco, or alcohol. She takes no medications. Review of systems is unremarkable. On exam, her blood pressure is 176/88 mmHg and pulse is 70 bpm. On funduscopic exam, AV nicking is present. A systolic and diastolic bruit over the right side of the abdomen is auscultated. The remainder of the exam is normal. ECG: Sinus rhythm with no Q waves, no evidence of hypertrophy, and no ST-T wave changes. Repeat blood pressure in one week is 174/90 mmHg. You begin a workup for secondary causes of hypertension. Her potassium is normal. Which of the following is most likely to confirm the diagnosis? A) Measurement of vanillylmandelic acid and metanephrines in a 24-hour urine collection B) Evaluation of renal function C) Evaluation of the renal arteries with angiography D) Evaluation of the adrenal gland with contrast tomography and electrolyte measurements E) Evaluation of the renal arteries with duplex ultrasound

Answer C. Evaluation of the renal arteries with angiography Explanation This patient most likely has fibromuscular dysplasia (FMD), a non-atherosclerotic cause of renovascular hypertension. FMD is the most common, correctible cause of secondary hypertension and is diagnosed by visualizing a "string of beads" in the distal two-thirds of a renal artery using angiography. Patients who present with hypertension at an early age (< age 30), have the onset of hypertension after the age of 50, have hypertensive urgency/emergency, or have refractory hypertension should be considered good candidates to screen for secondary causes. In patients with renovascular hypertension and typical atherosclerosis risks (hyperlipidemia, family history, smoking, diabetes), atherosclerosis of the renal artery is the most common cause of renovascular hypertension; however, in patients without those risk factors, especially in younger women, FMD must be considered. In addition, the finding of a unilateral abdominal (i.e., renal) bruit also suggests renovascular disease as the cause. This patient has no reason to have renal failure, no risks for diffuse atherosclerosis, and no signs or symptoms of pheochromocytoma, a very rare cause of secondary hypertension. Hyperaldosteronism is a common, reversible cause of secondary hypertension, but in this clinical setting would not be as likely as FMD, especially with the normal potassium.

A 27-year-old Caucasian male who lives in Jackson, MS, presents to his local emergency department complaining of dizziness over the past few days. However, he reports that over the past 4 days he has had 3 episodes of syncope followed by periods of unresponsiveness! He has always been in excellent health and is a personal trainer to the "stars" of Jackson. His last illness was about 4-5 months ago when he developed fever, chills, and generalized weakness while he was spending a month as a guest personal trainer in a spa off of Long Island, NY. PAST MEDICAL HISTORY: Negative except for above Denies use of steroids; says he is "natural" SOCIAL HISTORY: Lives with wife of 3 years; she is a plastic surgeon in town Doesn't smoke Doesn't drink alcohol Denies drug use FAMILY HISTORY: Father 60, with HTN Mother 55, with HTN Brother 32, with severe obesity REVIEW OF SYSTEMS: Essentially negative PHYSICAL EXAMINATION: Ht 5' 8", Wt 230 lbs, BP 110/80, P 30, RR 18, Temp 98.6° F HEENT: PERRLA, EOMI TMs clear Throat: Clear Neck: Supple Heart: RRR with severe bradycardia Lungs: CTA Abdomen: Bowel sounds present; abs of steel; no rebound; no hepatosplenomegaly Extremities: No cyanosis, clubbing, or edema GU: Normal male genitalia; rectal heme-negative LABORATORY: CXR normal Serum chemistries normal Drug screen +cocaine ECG: Complete heart block with nonspecific ST- and T-wave changes; no findings consistent with prior myocardial infarction Which of the following is the most likely cause of his complete heart block? A) Infection caused by HIV B) Infection transmitted by a bite from a louse C) Infection transmitted by a bite from a tick D) Myocardial infarction from coronary artery embolus E) Myocardial infarction from cocaine use

Answer C. Infection transmitted by a bite from a tick Explanation His picture is consistent with manifestations of Lyme disease, which is transmitted by the bite of an Ixodes scapularis (formerly I. dammini). Lyme disease is due to Borrelia burgdorferi. His exposure occurred while he was "moonlighting" off Long Island, which is a high-risk area for Lyme disease. Lyme carditis is most often manifested by AV nodal conduction disturbances, anywhere from 1st through 3rd degree heart block. Treatment consists of a 3rd generation cephalosporin or doxycycline. It will usually resolve with treatment. Cocaine can cause myocardial ischemia and infarction; but we have no evidence on ECG that this has occurred and his symptoms have been prolonged now for a while, so the ECG should have some evidence of damage at this point. HIV carditis would not present this way, and a coronary artery embolus would not present in this fashion over several days.

A 22-year-old college student seeks a pre-employment physical from you prior to applying for the position of ski instructor at the Crisp Ski Resort in Vail, CO. You note his tall thin habitus and obtain height and arm-span measurements of 73" each. Your examination documents a Grade II/VI systolic ejection "flow" murmur in the pulmonary outflow tract. Additionally, he has fixed splitting of the second heart sound. His EKG demonstrates right axis deviation, and there are prominent R-waves in Lead V1 with an R:S ratio of [1.4:1]. His apparent right ventricular hypertrophy (RVH) is probably a result of which of the following? A) Membranous ventricular septal defect (VSD) B) Cystic fibrosis C) Ostium secundum atrial septal defect (ASD) D) Primary pulmonary hypertension E) Type A Wolff-Parkinson-White syndrome (WPW)

Answer C. Ostium secundum atrial septal defect (ASD) Explanation Again, from a test-taker's point of view, the noting of fixed splitting of the second heart sound means that only one answer is possible on a test. In "real-life," there are occasional patients with ventricular septal defect who have this finding, but that would never be the point on an examination of this quality. The flow murmur is typical and represents the shunt volume in the pulmonary artery. In fact, there may be a murmur resulting from the flow across the septal defect itself, but this is, in fact, a diastolic murmur and is seldom heard clinically. This phenomenon is also largely exhalation-dependent and is therefore, nonetheless, exactly what explains the fixed splitting: Whereas inspiration volume loads the right ventricle in the usual fashion (via acceleration of caval flow), expiration volume loads the right ventricle again (via the increase in shunt volume across the septal defect itself in exhalation), thereby resulting in a balance of loading phenomena affecting the right heart (and effectively "holding" P2 away from A2).

A 45-year-old woman presents with acute shortness of breath and chest pain. She has a recent history of breast cancer and began chemotherapy 2 weeks ago. Her BP is 70/50 mmHg. She is admitted to the ICU and undergoes right heart catheterization. Noted are the following findings: RA pressure is 18 mmHg; PA pressure is 34/18 mmHg; PCWP is 17 mmHg; and her BP remains low at 70/48 mmHg. Based on these findings, which of the following is most likely occurring? A) Pulmonary hypertension B) Mitral stenosis C) Tamponade D) Cardiogenic shock E) RV infarction

Answer C. Tamponade Explanation Note that the diastolic pressure in the RA, diastolic pressure in the PA, and PCWP are all the same. The systemic BP is low. Be on the lookout for tamponade in a patient with cancer who suddenly presents decompensated.

All of the following can be a side effect of digitalis toxicity except: A) Arrhythmias B) Blurred vision C) Confusion D) Gingival hyperplasia E) Nausea and vomiting

Answer D. Gingival hyperplasia Explanation Arrhythmias are obviously one of the side effects of toxicity. Blurred vision and "halo" lights are common. GI complaints are very common. Gingival hyperplasia, however, is something that is not associated with digoxin—you can see it with calcium channel blockers like nifedipine and classically with anti-seizures medications. Headaches are also common.

A 17-year-old high school student is referred by his basketball coach to you for a physical examination. He is healthy and has no complaints. PAST MEDICAL HISTORY: No immunizations since age 12 (had Td booster then); has received 2 MMRs; no hepatitis B vaccination SOCIAL HISTORY: A/B honor roll student Works afternoons at the movie theater; runs the popcorn machine Sleeps about 4 hours a night after getting all his homework done Doesn't smoke or drink Is sexually active with girls; always wears a condom Wears seatbelts FAMILY HISTORY: Mother 44, healthy Father 44, healthy Brother 16, healthy Brother, died suddenly while playing basketball at age 19; no autopsy obtained Sister 15, healthy REVIEW OF SYSTEMS: Negative PHYSICAL EXAMINATION: Ht 6'3", Wt 210 lbs BP 120/70, P 65, RR 18, Temp 98.5° F HEENT: PERRLA, EOMI TMs clear Throat clear Neck: Supple; no murmurs heard in neck Brisk carotid upstroke Heart: RRR with III/VI harsh systolic murmur Murmur increased with Valsalva Murmur decreased with passive leg-raising Lungs: CTA Abdomen: Bowel sounds present; no hepatosplenomegaly Extremities: No cyanosis, clubbing, or edema GU: Tanner V pattern hair growth and gonadal development Skin: Acne on face and back Based on your findings in the history and physical examination, which of the following is your most likely diagnosis? A) Severe mitral stenosis B) Constrictive pericarditis C) Anomalous coronary artery disease D) Hypertrophic cardiomyopathy (idiopathic hypertropic subaortic stenosis) E) Upper arch aortic aneurysm

Answer D. Hypertrophic cardiomyopathy (idiopathic hypertropic subaortic stenosis) Explanation He has all of the classic findings of hypertropic cardiomyopathy. He has a harsh systolic murmur without radiation to the neck. The murmur is due to the intracavitary obstruction in the left ventricle. The murmur increases with Valsalva or any other maneuver that will decrease left ventricular size, which will increase the outflow obstruction and the intensity of the murmur. Conversely, maneuvers that increase left ventricular blood volume (squatting and passive leg-raising) will "move" the muscular obstruction protruding into the outflow track away from the opposite wall, decreasing the obstruction and the murmur. The brisk carotid upstroke is also part of this condition. Additionally, the history was helpful; the death of his older brother suggests the "familial" form of this disease.

A 56-year-old male presents with a systolic murmur. Echocardiography shows a left ventricular ejection fraction of 50%, an LV systolic dimension of 60 mm, severe mitral valve prolapse, and severe mitral regurgitation. He exercises for 4 minutes on a Bruce protocol to a peak heart rate of 105 and peak blood pressure of 130/80, stopping because of dyspnea. Rest and peak exercise ECGs follow. Which of the following should be done next? A) Observe for 6 more months and repeat the evaluation. B) Dobutamine stress echo. C) Therapy with beta-blockers. D) Refer for mitral valve replacement.

Answer D. Refer for mitral valve replacement. Explanation This patient has severe mitral regurgitation on echo and has an LV systolic dimension of more than 55 mm. In addition, his exercise capacity is severely limited. The indications for mitral valve surgery in patients with severe MR include any symptomatic patients or asymptomatic patients with LV systolic dimensions of more than 55 mm, so either way this patient qualifies.

A 60-year-old man has questions about the safety of his medications. He drinks 2 glasses of grapefruit juice each morning, but recently saw a "20/20" TV show segment on the dangers of grapefruit juice in patients taking "heart medicines." He is currently prescribed benazepril, hydrochlorothiazide, warfarin, omeprazole, simvastatin, and loratadine. Which of his drugs is affected by grapefruit juice? A) Omeprazole B) Warfarin C) Loratadine D) Simvastatin E) Benazepril

Answer D. Simvastatin Explanation Grapefruit juice can increase statin levels markedly, resulting in hepatitis and/or myositis. This is especially true with simvastatin and lovastatin. The other drugs are not affected by grapefruit juice. While not listed as a choice, nifedipine serum levels are also affected by this juice, and patients should be counseled not to ingest grapefruit within an hour of taking nifedipine.

You are on duty in the emergency department when a group of four companions present their friend, a 21-year-old woman who works for the local escort service. She is acutely ill with a temperature of 104° F, HR 130/min, respirations 18/minute, and apparent distress. Her systemic venous pressure is elevated, and the jugular venous pulse contour is dominated by large "cv" waves that swell with each inspiration. Your auscultatory examination reveals a Grade III/VI holosystolic murmur heard best at the lower left (and right) sternal border(s), and which accentuates to Grade IV/VI intensity with inspiration. The liver seems to pulsate with each systole. A chest x-ray reveals scattered focal white fluffy opacities. You recognize what is, no doubt, a complication of her intravenous heroin usage. Which of the following is the most likely diagnosis, manifested as acute? A) Bleeding pulmonary arteriovenous (AV) fistula B) Mitral regurgitation C) Ruptured sinus of Valsalva aneurysm D) Tricuspid regurgitation E) Aortic valvular regurgitation

Answer D. Tricuspid regurgitation Explanation The patients of this nature are so familiar in some geographic settings that the diagnosis is nearly routine. The characteristic features of tricuspid regurgitation accompanied by septic pulmonary emboli and evidence of parenteral drug usage predict that Staphylococcus aureus will be isolated as the causative agent in the majority of cases. Some of these patients have even had their tricuspid valves removed rather than attempting to implant a prosthetic device in a patient who could return to bacteremia-associated behaviors (via relapse). This is something that can be done if the patient has no additional heart disease. An alternative to this approach might include a plastic repair of the native valve, a so-called "vegetectomy."

A 26-year-old woman presents to the emergency department after fainting. An ECG shows a long QT interval. She admits that she had been staying up late for several weeks working and has spent long hours in the hot tub to relax. The day before fainting, she was told that she might be laid off of her job, and she panicked. Which of the following would most likely prolong her QT interval? A) Low-fat diet B) Hypercalcemia C) Hypermagnesemia D) Tricyclic overdose E) Hyperkalemia

Answer D. Tricyclic overdose Explanation Tricyclic overdose is most consistent with prolonged QT intervals. Of the choices, hyperkalemia would not cause a prolonged QT interval (but hypokalemia does). Hypocalcemia, hypomagnesemia, and a liquid protein diet would as well. Other etiologies include starvation, CNS insult, erythromycin, and ischemia.

A 39-year-old male has sudden onset of severe chest pain. It radiates into both arms and is associated with diaphoresis. By the time he arrives in the emergency department, the pain has resolved. He reports his father died of a heart attack at home at the age of 45. The patient has a history of mild hypertension. He has a high arched palate on physical examination. Cholesterol: 245 mg/dL, ECG: 1st degree AV block/no ischemic changes. Which of the following would you do next? A) Administer TPA B) Upper endoscopy C) Cardiac catheterization D) Admission to CCU to obtain serial enzymes to rule out MI E) Contrast chest CT scan

Answer E. Contrast chest CT scan Explanation This patient has acute onset of chest pain radiating down his arms. He has a high arched palate on physical exam and a vague family history of his father dying of a "heart attack" at home at the age of 45. This patient has Marfan syndrome and is experiencing an aortic dissection. The best test is either a transesophageal echo or a contrast chest CT scan. It is appropriate to follow this patient in the CCU and obtain cardiac enzymes, but before anything else is done, he should be evaluated for aortic dissection.

A 64-year-old male had an inferior STEMI and was treated with aspirin, beta-blockers, an angiotensin-converting enzyme inhibitor, and a statin. He was standing at the nurse's station when telemetry showed the following: Wenckebach Which of the following should you do? A) Refer for an electrophysiology study. B) Insert a temporary transvenous pacemaker. C) Refer for coronary angiography. D) Refer for permanent pacemaker implantation. E) Decrease the beta-blocker dose.

Answer E. Decrease the beta-blocker dose. Explanation This patient with a recent myocardial infarction was found on telemetry to have an episode of 2nd degree atrioventricular block: Mobitz type I (Wenckebach block). This conduction abnormality is common in patients with an inferior MI and does not constitute an indication for a pacemaker, particularly in an asymptomatic patient. Decreasing the beta-blocker dose is a reasonable response. Remember that the indication for pacing is symptomatic bradycardia.

In an adult, which of the following conditions would most likely benefit from prophylactic antibiotics to prevent endocarditis? A) Isolated ostium secundum atrial septal defect (ASD). B) Effective surgical repair of a ventricular septal defect (VSD) in childhood. C) Previous coronary artery bypass graft (CABG) surgery. D) Implanted cardiac defibrillator. E) None of the choices would benefit.

Answer E. None of the choices would benefit. Explanation Note that the guidelines changed in 2007: SBE Prophylaxis for Dental Procedures: Prosthetic cardiac valve Previous history of endocarditis Congenital heart disease: Unrepaired cyanotic Completely repaired with prosthetic material or device for ≤ 6 months postprocedure Cardiac transplant recipients who develop cardiac valvulopathy Note that all of the valvular abnormalities have been taken out for needing antibiotic prophylaxis. For adults the only reason you'll need to give antibiotic prophylaxis is for those with prosthetic valves or who have a history of endocarditis. Summary of Major Changes in Updated Guidelines: Limit SBE prophylaxis to only those conditions listed above. Antibiotic prophylaxis is no longer recommended for any other form of congenital heart disease. Antibiotic prophylaxis for procedures on respiratory tract or infected skin; etc., only for conditions listed above. No antibiotic prophylaxis for GU or GI procedures (regardless of cardiac valve abnormality)!

A 35-year-old presents to your office complaining of palpitations. These occur most often when she drinks coffee or an energy drink. She has never had syncope. The palpitations can last up to 3-5 minutes and spontaneously resolve. PAST MEDICAL HISTORY: Negative SOCIAL HISTORY: Married with 1 child Smokes 2 packs/day of cigarettes FAMILY HISTORY: Mother 60 with HTN Father 50 with HTN REVIEW OF SYSTEMS: Negative PHYSICAL EXAMINATION: BP 130/70, P 90, RR 16, T 98.8° F, Ht 5'1", Wt 230 lbs HEENT: PERRLA, EOMI TMs clear Throat clear Neck: Supple Heart: RRR without murmurs, rubs, or gallops Lungs: CTA Abdomen: Benign Extremities: No cyanosis, clubbing, or edema Resting ECG in your office is normal. Which of the following would be the most useful test in evaluation of this patient? A) 24-hour ambulatory ECG (Holter) monitoring B) Stress test C) Left heart catheterization D) Right heart catheterization E) Echocardiogram

Answer: A. 24-hour ambulatory ECG (Holter) monitoring Explanation She likely is having an arrhythmia (such as SVT) from her caffeine intake; coffee and energy drinks can contain moderate to high levels of caffeine. The best test would be to perform 24-hour ambulatory ECG (Holter) monitoring and then see what arrhythmia is occurring. The other tests are not indicated at this time.

Imagine it is Boston during the Civil War. A woman takes some bread to some starving immigrant neighbors, one of whom has scarlet fever. A few years later, she is dying of a cardiac-related illness. Which of the following would she most likely have on physical examination? A) A sound heard with the diaphragm at the apex shortly after S2 B) A soft S1 C) A sound heard best with the bell at the apex after S2 D) A soft, decrescendo blowing murmur heard at the lower left sternal border in the sitting position. E) A midsystolic click followed by a systolic murmur heard at the left sternal border

Answer: A. A sound heard with the diaphragm at the apex shortly after S2 Explanation The woman has rheumatic mitral stenosis. Mitrial stenosis would likely be an opening snap heard at the apex after S2. The opening snap is a high-pitched sound, and thus heard better with the diaphragm. Scarlet fever is caused by a group A strep (GAS) infection, which can cause acute rheumatic fever (ARF). Rheumatic heart disease develops over a period of years after 1 or more bouts of untreated ARF. Mitral stenosis is a common finding that can occur anywhere from 5 to 20 years later. S1 is typically loud in mitral stenosis. Aortic regurgitation would produce a decrescendo blowing murmur at the left lower sternal border with the patient sitting forward. Mitral valve prolapse would present with the click and murmur. A sound heard with the bell at the apex after S2 would be an S3, which patients with mitral stenosis are very unlikely to have, since the left ventricle is not filled rapidly.

A 72-year-old man with Type 2 DM and a history of MI 6 years ago is found to have a pulsatile abdominal mass on exam. Ultrasound reveals a 7-cm aortic aneurysm. He walks 8-10 blocks every day without symptoms. You are asked to examine this patient and advise on what is needed prior to aortic surgery. Exam: BP: 120/70 mmHg Pulse: 75 bpm Chest: Clear Cardiac: Normal S1 and S2, no murmur Hct: 38% Glu: 160 mg/dL ECG is as follows: atrial fibrillation and a controlled ventricular rate Which of the following would you recommend? A) Complete a risk assessment with stress myocardial perfusion imaging. B) Obtain TTE before surgery. C) No surgery. D) Proceed with surgery. E) Complete a risk assessment with cardiac catheterization

Answer: A. Complete a risk assessment with stress myocardial perfusion imaging. Explanation This patient is undergoing a high-risk procedure, and both AHA and ACP protocols define him as a high-risk patient due to the presence of vascular disease, and the high-risk nature of the operation he will be undergoing, which is a major vascular surgery. Both protocols recommend stress testing prior to surgical planning. A functional test would be more useful than going straight to catheterization. He has an abnormal baseline ECG with atrial fibrillation and a controlled ventricular rate, and images would help with sensitivity and specificity in determining cardiac risk. This patient with atherosclerotic disease undoubtedly has CAD, given his past MI. The key question to answer: Is the myocardium at risk? Cardiac catheterization and TTE would not assess functional status.

A 67-year-old man is seen in follow-up after an anterior myocardial infarction. He is doing well and has no complaints. PAST MEDICAL HISTORY: Negative before the MI MEDICATIONS: ECASA one qd Propranolol 40 mg PO tid Isosorbide dinitrate 20 mg tid SOCIAL HISTORY: Retired history professor Lives with wife Has never smoked cigarettes Drinks occasional beer on weekend FAMILY HISTORY: Unknown; adopted REVIEW OF SYSTEMS: Unremarkable PHYSICAL EXAMINATION: BP 120/70, RR 18, P 42, Temp 98.3° F HEENT: PERRLA, EOMI TMs clear Throat clear Neck: Supple, no JVD Heart: RRR without murmurs, rubs, or gallops Lungs: CTA Abdomen: Bowel sounds present, no masses Extremities: No cyanosis, clubbing, trace pedal edema Which of the following is the next best step in his management? A) Decrease the propranolol dose. B) Discontinue aspirin therapy. C) Insert temporary pacemaker. D) Schedule for electrophysiologic testing. E) Order thyroid function tests.

Answer: A. Decrease the propranolol dose. Explanation Be aware that some patients may have an increased sensitivity to the effects of beta-blockers. The easiest thing is to decrease the dosing interval to twice a day and see if that works. The other tests are not indicated. Remember, he is asymptomatic, so insertion of a temporary pacemaker is not indicated at this point

A 29-year-old woman presents with fatigue and chest discomfort with exertion. Review of systems is negative for syncope, cough, skin rashes, arthritis, fevers, and weight loss. She reports no family history of coronary artery disease. Physical examination reveals clear lung fields, a systolic lift along the left sternal border, and a normal split S 2 with a loud second component. No cardiac murmur is present. Chest radiograph is remarkable only for bilateral prominent pulmonary arteries without pulmonary congestion. Spirometry is normal. Which of the following is the most appropriate next step in diagnosis? A) Echocardiogram B) Cardiac stress test D) Open lung biopsy E) Methacholine challenge test

Answer: A. Echocardiogram Explanation The history and physical examination suggest the possibility of Group 1 pulmonary hypertension (formerly known as idiopathic or primary pulmonary hypertension). Pulmonary artery pressures can be estimated by echocardiography. If elevated, then right heart catheterization with monitoring of the response to treatment is warranted. Echo is also necessary to exclude less obvious cardiac causes of secondary pulmonary hypertension.

You see a 63-year-old man in the emergency department with long-standing hypertension and hyperlipidemia for chest pain. He is an 80-pack-year smoker. One year ago, he underwent cardiac catheterization because of exertional pain; at that time, he was noted to have 3-vessel coronary artery disease, normal valves, and a normal ejection fraction. His CAD was not amenable to percutaneous coronary intervention (PCI) or CABG. At that time, medical management with a beta-blocker, ASA, and a statin was begun. His exertional chest pain completely resolved. Today, he describes his pain as severe centrally, radiating to his left arm and back without shortness of breath. He has no other symptoms. This pain was acute and not associated with any sort of exertional pain or shortness of breath prior to today. On exam, his blood pressure is 180/110 mmHg, pulse 90 bpm, respirations 14, and he is afebrile. There is no elevation of jugular venous pulsations, no S3, and no rales. You hear a grade II/VI decrescendo murmur in the 3rd left interspace after the 2nd heart sound. There is no edema. ECG shows 3-mm ST-segment elevations in leads I, aVL, and V2 through V5, and reciprocal ST depressions in leads II and III. Cardiac troponins are elevated to three times the upper limit of normal. Which of the following is the next appropriate step in the management of this patient? A) Immediate bedside transesophageal echocardiogram B) Upper endoscopy C) Thrombolytic therapy D) Pericardiocentesis E) High-dose ibuprofen

Answer: A. Immediate bedside transesophageal echocardiogram Explanation Based on the history, elevated blood pressure, and murmur of (new) aortic regurgitation, this patient should be suspected of having an aortic dissection. He is also having a myocardial infarction, which is a known complication of aortic tears that dissect proximally to the coronary arteries. In suspected aortic dissection, rapid imaging is necessary to confirm the diagnosis and to guide therapy, especially in the setting of concomitant MI. Use of thrombolytic therapy is contraindicated in this setting and could be potentially devastating. CXR was commonly used in the past as the initial test, and you would expect to see a wide mediastinum—but, this test has low sensitivity. Transesophageal echocardiography is the test of choice followed by MRI. Pericardiocentesis would not be indicated in this setting. High-dose ibuprofen would treat pericarditis, which this patient does not have. Upper endoscopy is not warranted.

A 70-year-old man has been having recurring episodes of dizziness over the last several months. You admit him to the hospital because of a fainting spell that occurred earlier today. Initial laboratory and ECG are normal. There is no evidence of acute myocardial infarction by history or laboratory. He says that he notices the dizzy spells, and that is about it. He has no prodrome and no other symptoms with them. This fainting spell is the first one that he has had. He was not aware that it was about to happen. PAST MEDICAL HISTORY: Prostatic hypertrophy diagnosed 5 years ago Colon polyp removed 3 years ago MEDS: ECASA qd Nifedipine 30 mg qd SOCIAL HISTORY: Former logger, retired for 20 years Lives with 4th wife Chews tobacco for 60 years Doesn't smoke Drinks a beer every now and then; none in 6 weeks FAMILY HISTORY: "Outlived everyone in my family" Mother died at age 67 of stroke Father died at age 50 of MI Brother died at age 60 of stroke Sister died at age 60 of MI Son died at age 30 of motor vehicle accident REVIEW OF SYSTEMS: Negative PHYSICAL EXAMINATION: Well-developed, well-nourished man in no distress Ht 6' 1", Wt 190 (unchanged in 2 years), BP 120/70, RR 18, P 76 regular, Temp: 97.9° F HEENT: PERRLA, EOMI, discs sharp TMs: Clear Throat: Clear Neck: Supple Heart: RRR with II/VI systolic murmur heard for 20 years Lungs: CTA Abdomen: Bowel sounds present; no masses; no hepatosplenomegaly Extremities: No cyanosis, clubbing, or edema Rectal: No masses; heme-negative LABORATORY: CBC normal Electrolytes normal ESR 12 Thyroid function tests normal The next morning, when the nurse's aide comes in to take vitals, he tells her that 5 minutes ago he had another dizzy spell. She notifies the nurse who is on her coffee break. The charge nurse comes in and looks at the monitor and sees that he is in Normal Sinus rhythm. You come in on rounds and he relates this story to you. You go back and look at the rhythm strip from the approximate time this event happened. You note absent QRS complexes every 3rd beat. The PR interval is slightly prolonged but is constant from beat to beat. P waves are present at regular intervals. Which of the following is the most appropriate action to take at this point? A) Insertion of a permanent cardiac pacemaker. B) Administration of atropine, one-time dose 2 mg IV. C) Administration of isoproterenol at a constant infusion rate 2 mg/min. D) This is a benign arrhythmia; therefore, no specific therapy is indicated.

Answer: A. Insertion of a permanent cardiac pacemaker. Explanation What the ECG is describing is the sudden failure of atrial ventricular conduction without a preceding change in the PR interval. The patient has Mobitz type 2, 2nd degree AV block. This is bad and usually means significant disease of the conducting system. It is also an unstable thing to have and frequently will progress to complete heart block. Additionally, he is severely symptomatic. Therefore, the only correct solution is to put in a permanent pacemaker in anticipation that he will continue to have worsening of his conduction abnormalities.

A 35-year-old woman is 16-weeks pregnant. Her BP is 170/110, and her urinalysis shows she has 2+ proteinuria. Otherwise, she has no complaints and is referred by her obstetrician for treatment of her hypertension. Of the following, which is the best agent to use? A) Methyldopa B) Lisinopril C) Hydrochlorothiazide D) Fosinopril E) Nitroprusside drip

Answer: A. Methyldopa Explanation Methyldopa is the only drug listed that is safe in pregnancy. Beta-blockers and labetalol are commonly used as well. HCTZ may lead to volume depletion in a pregnant woman. Lisinopril, fosinopril, and all ACE inhibitors are absolutely contraindicated because they cause fetal abnormalities and death. Nitroprusside drip would poison the fetus.

An 18-year-old ballet dancer seeks your consultation in the outpatient clinic because of cardiac consciousness (manifested as an intermittent "skipping" and "flipping" inside her chest). You are struck by her graceful physiognomy and measure her height as 70" and arm span as 71.5". Her joint laxity permits easy apposition of her thumbs to her forearm surfaces and, indeed, she reports the childhood ability to "touch her elbows together behind her back." Her physical examination includes the following: There is a mid-systolic click and a Grade II/VI late-systolic murmur. There is murmur augmentation with both standing and Valsalva strain. In addition, with handgrip, the click moves closer to S1 and the murmur gets louder. There is murmur diminution with prompt squatting, followed (often) by dramatic augmentation of the murmur with resumption of the standing posture, occasionally to Grade VI intensity (e.g., audible in the room without a stethoscope). Based on your findings, you are certain that she has which of the following? A) Myxomatous mitral valve prolapse (MVP) B) Ostium primum atrioventricular septal defect (AVSD) C) Acquired (muscular) ventricular septal defect (VSD) D) Hypertrophic obstructive cardiomyopathy (HOCM; previously IHSS) E) Ostium secundum atrial septal defect (ASD)

Answer: A. Myxomatous mitral valve prolapse (MVP) Explanation Auscultatory findings are consistent with mitral valve prolapse. Augmentation of murmur with standing and Valsalva strain exclude ASD, AVSD and VSD, and presence of mid-systolic click favors MVP in comparison to HOCM. In addition, the murmur of HOCM gets softer with handgrip.

A 74-year-old female comes to the emergency department with chest pain and dyspnea and is found to have a non-ST segment elevation MI (NSTEMI). Physical exam and chest x-ray show pulmonary edema. She is given a beta-blocker, intravenous morphine, sublingual nitroglycerin, and 160 mg of intravenous furosemide. She initially improves but one hour later becomes hypotensive. A right heart catheter is inserted and the following pressures are obtained: Location Pressure (mmHg) Right atrium 2 Pulmonary artery 20/10 Pulmonary capillary wedge 6 Which of the following treatments is appropriate? A) Normal saline infusion. B) Insert an intraaortic balloon pump. C) Fibrinolytic therapy. D) Blood cultures and empiric antibiotics. E) Refer for emergency percutaneous coronary intervention

Answer: A. Normal saline infusion. Explanation This patient presents with an NSTEMI and evidence of pulmonary edema. She is then administered three medications that cause vasodilation or volume depletion, and she becomes hypotensive. A right heart catheter demonstrates very low filling pressures consistent with intravascular volume depletion. The appropriate therapy is judicious fluid replacement with intravenous saline. In retrospect, the large dose of IV furosemide was excessive, especially when combined with the other medications. Furthermore, the beta-blockers limited the reflex tachycardia that would ordinarily occur as a compensation for volume depletion.

A 46-year-old woman presented initially to the hospital with acute substernal chest pain. She described no previous cardiac history. Her initial ECG showed acute ST-segment elevation of 5 mm and Q waves in the inferior-lateral leads. Subsequently her CPK increased to 3,300. It is now the 4th day post-MI, and you are seeing her for daily rounds. She says that in the last few hours, she has become short of breath and feels like she is not feeling as well as yesterday. She has not had any syncopal episodes. Significant aspect of the physical examination: She now has a holosystolic murmur, which is loudest at the apex. No signs of cardiac tamponade are present. You order a stat echocardiogram. At the same time as ordering the echocardiogram, which of the following do you do next? A) Notify your cardiovascular surgeon to come right away and evaluate her for emergent mitral valve replacement. B) Notify your cardiovascular surgeon to come right away and evaluate her for emergent repair of VSD. C) Notify your cardiovascular surgeon to come in the next day or two to evaluate her for elective repair of post-MI VSD. D) Schedule her for cardiocentesis. E) No surgery is indicated; this is a nonsurgical emergency only, and she will respond to pressor agents quickly.

Answer: A. Notify your cardiovascular surgeon to come right away and evaluate her for emergent mitral valve replacement. Explanation She has likely had a papillary muscle rupture and developed acute mitral insufficiency. The murmur is classically a holosystolic murmur at the apex, as opposed to being located at the left sternal border with acute ventricular septal defects. Pericardiocentesis is not indicated, because she has no evidence indicating tamponade, which you would see with a left ventricular free wall rupture. The lack of syncope also points more toward ruptured papillary muscle instead of ventricular septum rupture or left ventricular free wall rupture.

A 78-year-old man with a history of hypertension and DM falls and fractures his shoulder. He will require shoulder surgery. He lives alone and performs all of his activities of daily living. He has no history of CAD or heart failure. Review of systems is negative for chest pains, orthopnea, paroxysmal nocturnal dyspnea, and lower extremity edema. Physical Examination: BP: 150/90 mmHg Pulse: 90 bpm Chest: Clear Cardiac: Normal S1/S2, +S4, no murmur Ext: No edema Laboratory: HB: 11 mg/dL Hct: 33% ECG: NSR, mild LVH Glu: 148 mg/dL Which of the following would you recommend? A) Okay for immediate surgery. B) Persantine thallium stress test. C) Transfuse 2 units of blood. D) Echocardiogram.

Answer: A. Okay for immediate surgery. Explanation This patient is undergoing nonvascular surgery and is low-to-moderate risk. Neither the AHA nor ACP guidelines would recommend cardiac workup prior to this surgery (thus, no need for Persantine thallium stress test or echocardiogram). His surgery should not be postponed for a transfusion. However, he may well need the transfusion postoperatively.

A 56-year-old woman is admitted with an anterior MI. She was stabilized and initially received aspirin, thrombolytic therapy, heparin, nitrates, and beta-blockers. On day 4, she has multiple runs of ventricular tachycardia that are asymptomatic and last only a few seconds. Eventually, her thrombolytics and anticoagulants are discontinued. On day 6, she becomes acutely short of breath. Vital signs: BP: 86/55 mmHg Pulse: 106 bpm Respiration: 19 On exam, there is a new, harsh, pansystolic murmur that is audible over the entire chest and associated with a thrill. Which of the following is the most appropriate next step in patient care? A) Order an immediate surgical consultation for repair of a ventricular septal defect. B) Immediately aspirate the pericardial sac for presumed tamponade. C) Order an immediate surgical consultation for repair of a ruptured papillary muscle. D) Prescribe massive volume resuscitation for presumed right ventricular infarction. E) Take to the cath lab and reinstitute thrombolytic therapy for a recurrent MI.

Answer: A. Order an immediate surgical consultation for repair of a ventricular septal defect. Explanation Mechanical complications of acute MI include ventricular free wall rupture (< 1%), intraventricular septal rupture (2%), and acute mitral regurgitation. This patient's exam is consistent with ventricular septal defect, which is more common in female patients and in the setting of anterior myocardial infarction, and can happen within 1 to 14 days following an acute infarction.Thrombolytic therapy increases the risk of interventricular septal rupture. Ventricular septal defect as a mechanical complication of acute MI can present with chest pain, dyspnea, hypotension, and cardiogenic shock. Presentation of papillary muscle rupture may be similar, but the murmur will be different—more likely to have a softer quality and will not be associated with a thrill. This patient's exam is not consistent with pericardial tamponade. RV infarction may present with hypotension but would not lead to pulmonary edema or the murmur described.

A 19-year-old college student seeks a pre-employment physical from you in preparation for applying for the position of lifeguard at the local country club. You note his tall, thin habitus and obtain height and arm-span measurements of 72" each. Your examination documents a Grade II/VI systolic ejection "flow" murmur in the pulmonary outflow tract and apparent fixed splitting of the 2nd heart sound. His ECG demonstrates a mean electrical QRS axis of 115 degrees in the frontal plane, and there are prominent R waves in Lead V1 with a R:S ratio of (1.3:1). You recognize that his apparent right ventricular hypertrophy (RVH) is surely a result of which of the following? A) Ostium secundum atrial septal defect (ASD) B) Type A Wolff-Parkinson-White syndrome (WPW) C) Primary pulmonary hypertension D) Cystic fibrosis E) Membranous ventricular septal defect (VSD)

Answer: A. Ostium secundum atrial septal defect (ASD) Explanation The fixed splitting of the 2nd heart sound points you to the single best answer. The flow murmur is typical and represents the shunt volume in the pulmonary artery. In fact, there may be a murmur resulting from the flow across the septal defect itself, but this is a diastolic murmur and is seldom heard clinically. This phenomenon is also largely exhalation-dependent and is, therefore, exactly what explains the fixed splitting: Whereas inspiration volume loads the right ventricle in the usual fashion (via acceleration of caval flow), expiration volume loads the right ventricle again (via the increase in shunt volume across the septal defect itself in exhalation), thereby resulting in a balance of loading phenomena affecting the right heart (and effectively "holding" P2 away from A2).

A 63-year-old man with severe osteoarthritis of his hip is scheduled for elective left hip replacement in one week. During your preoperative physical, you find that he has a Mobitz II 2nd degree heart block, which has not been previously seen on prior ECGs. The patient has no other known medical problems and is on no medications. Which of the following should be done regarding his heart condition prior to surgery? A) Permanent pacemaker B) Nothing else needs to be done, he may proceed to surgery C) Clinical EP study D) Temporary pacemaker

Answer: A. Permanent pacemaker Explanation Unlike Mobitz I, which has a progressive prolongation of the PR interval, Mobitz II has a much higher tendency to progress to complete heart block. Therefore, for Mobitz II, a permanent pacemaker is indicated before the elective surgery is performed.

You receive the report of an electrophysiologic study (EPS), which was performed for a 35-year-old male. This patient has been seen in the emergency department multiple times for symptomatic tachycardia, and subsequent evaluation included an EPS. The report documents an accessory pathway and indicates a propensity for the conduction to be antegrade down the accessory pathway and retrograde in the normal pathway. The patient strongly desires a resolution to the problem but has been exceptionally nonadherent to medications and follow-up in the recent past. Which of the following is the best intervention to resolve his current cardiac issue? A) Radiofrequency ablation of the accessory pathway B) Placement of a chronic pacemaker C) Long-term propranolol treatment D) Surgical ablation of the accessory pathway E) Long-term digitalis treatment

Answer: A. Radiofrequency ablation of the accessory pathway Explanation Preexcitation syndrome occurs when an alternate pathway exists for the conduction of electrical impulses from the atria to the ventricles. Classically (95%), this presents with a short PR interval, a widened QRS complex, and a slurred upswing in the initial R wave (delta wave). Much more uncommonly, the conduction follows the accessory pathway and reenters through the normal pathway. This condition predisposes to episodes of tachycardia and can degenerate to ventricular tachycardia. Digitalis is contraindicated in these circumstances due to its tendency to worsen the condition. Propranolol can be effective when the conduction is primarily through the normal pathway but is of limited value when the primary conduction is through the accessory pathway. The patient's poor adherence also reduces the likelihood that oral medications would provide long-term control. A pacemaker can control the ventricular tachydysrhythmias but predisposes to atrial fibrillation. Both surgical and radiofrequency ablation can effectively provide long-term treatment of this condition, but radiofrequency ablation is safer and more cost-effective. Surgical ablation is reserved for cases that cannot be treated with other modalities.

A 35-year-old woman comes for a routine visit. She has no history of cardiac disease and is a nonsmoker. Her mother had a myocardial infarction at age 70. On examination, blood pressure is normal, and body mass index is 28. LDL cholesterol is 169. Diet and exercise are recommended. On a follow-up visit in 3 months, she says she has tried to stick to her diet and is exercising twice a week. BMI and LDL cholesterol are unchanged. Which of the following should you do next? A) Refer for exercise and dietary counseling. B) Add a statin. C) Add niacin. D) Check lipoprotein A. E) Add a beta-blocker.

Answer: A. Refer for exercise and dietary counseling. Explanation This young woman's risk factors for coronary disease include obesity, sedentary lifestyle, and elevated lipids. Although diet and exercise were recommended at her first visit, she has really not changed her eating habits or exercise program significantly. Additional efforts to increase compliance, including enlisting the assistance of other practitioners, is indicated. As noted in the 2013 ACC/AHA Guideline on the Treatment of Blood Cholesterol to Reduce Atherosclerotic Cardiovascular Risk in Adults, there are 4 statin benefit groups: patients with known atherosclerotic cardiovascular disease, patients with an LDL cholesterol of 190 mg/dL or greater, patients aged 40 to 75 with diabetes and an LDL cholesterol of 70 mg/dL or greater, and patients without clinical cardiovascular disease who have an estimated 10-year CVD risk of 7.5% or higher. This woman, at her current age and LDL level, does not fit into any of those benefit groups, and thus treatment with initiation of medication is not advised.

A 54-year-old post-menopausal female presents with intermittent chest pain. The pain occurs with exercise and causes shortness of breath, aching pain in the left shoulder, and some palpitations. It resolves in 15-20 minutes with rest. She has no history of diabetes mellitus or hypertension. Her LMP was 6 years previously. She smokes 4-5 cigarettes/day. She undergoes exercise-tolerance testing and experiences an episode of pain similar to her original complaints. Her blood pressure is within expected ranges, and ST depression in the anteroseptal leads is noted. There is marginal response to sublingual nitroglycerin, but the pain and ST changes resolve with rest. She then undergoes cardiac catheterization that reveals normal coronary arteries. What is the most appropriate next step in her evaluation? A) There is no definite test for diagnosis, and it is a diagnosis of exclusion. B) Upper endoscopy with manometry. C) Psychiatric referral. D) Electrophysiological studies (EPS). E) Urine studies for catecholamines.

Answer: A. There is no definite test for diagnosis, and it is a diagnosis of exclusion. Explanation This patient's complaints are highly suggestive of cardiac ischemia and are consistent with cardiac syndrome X. In this condition, the patient experiences ischemia related to microvascular dysfunction in the cardiac tissue with poor blood flow reserve. They have normal coronary arteries on cath. Unfortunately, currently this is a diagnosis of exclusion. Some data supports cardiac magnetic resonance (CMR) perfusion imaging may be useful—but widespread confirmation is lacking. Exercise reproduced her symptoms without significant ectopy. EPS is not indicated without evidence of serious dysrhythmias. Likewise, symptoms exclusively during exercise make a GI source less likely, and a cardiac diagnosis should be fully evaluated before nonlethal noncardiac diseases are pursued. Symptoms occurring only with exercise, with a lack of hypertensive findings and adrenergic symptoms, make a pheochromocytoma less likely. There is no indication for psychiatric referral in this patient.

Which of the following patients is likely to benefit the most from fibrinolytic therapy? A) A 72-year-old woman with the onset of chest pain 3 hours ago, with tall R waves and ST-segment depression in leads V1 and V2 B) A 75-year-old man with the onset of chest pain 5 hours ago and a new left bundle-branch block C) A 65-year-old man with 2 previous coronary bypass surgeries and 2 hours of chest pain, who has 3 mm of ST-segment depression and new T wave inversion in leads V1 to V4 D) An 80-year-old woman with a recent onset of mild right hemiparesis and 2 hours of chest pain, with an ECG showing no Q waves and 4 mm of ST-segment elevation from leads V1 to V6, and 2 mm of ST-elevation in leads I and aVF E) A 46-year-old man with the onset of chest pain 4 hours ago, who has small Q waves and 3 mm of ST-segment elevation in leads II, III, and aVF

Answer: B. A 75-year-old man with the onset of chest pain 5 hours ago and a new left bundle-branch block Explanation Patients with new left bundle-branch block in the clinical setting of an acute ischemic event have the most mortality benefit from fibrinolytic therapy because of the amount of myocardium in jeopardy. Patients with anterior ST-segment elevation benefit less, with inferior ST-segment elevation still less, and are actually harmed with ST-segment depression. The 80-year-old woman has a strong indication for fibrinolytic therapy, except for the recent onset of a neurological deficit--which is a contraindication due to the increased potential for intracranial hemorrhage. The 72-year-old woman with a tall R wave in leads V1 and V2 is having an acute posterior MI and would benefit from fibrinolytic therapy to the same extent as an acute inferior MI.

The lead singer of a grunge rock band comes to your emergency department complaining of severe dyspnea, fever, and chills for the last 3 days. She admits to using intravenous heroin. On examination, her temperature is 102.3° F, heart rate is 125 beats/min, and blood pressure 100/60 mmHg. She has large v waves in her jugular pulse. The carotid pulses are normal. She has a faint systolic murmur heard along the lower left sternal border, which becomes louder on inspiration. A 3rd heart sound is present in the same area. She has scattered rhonchi and wheezes on lung examination. There are no splinter hemorrhages or other manifestations of endocarditis in the extremities, although there are several pustules present over the antecubital veins in both arms. Laboratory data show a white blood count of 15,600 with a preponderance of neutrophils. Hemoglobin is 12.2 g/dL; serum electrolytes, AST, and ALT are normal. Blood samples are sent for bacterial and fungal cultures, as well as hepatitis and HIV screening. Which of the following is most likely to be present on an imaging study? A) An echocardiogram showing a large vegetation on the aortic valve B) A ventilation/perfusion lung scan showing multiple perfusion defects C) An echocardiogram showing a large perforation in the anterior mitral valve leaflet D) A chest x-ray showing pulmonary edema E) A chest x-ray showing left ventricular enlargement

Answer: B. A ventilation/perfusion lung scan showing multiple perfusion defects Explanation The woman has infective endocarditis of the tricuspid valve, a common occurrence in intravenous drug users. An echocardiogram would be very likely to show a tricuspid vegetation. Given her symptoms and the lung findings, she almost certainly has multiple septic emboli to the lungs, a problem that would be best identified by a ventilation/perfusion lung scan. She has several signs of right heart failure and pulmonary hypertension, but no signs of left heart failure. All of the other choices would reflect left heart failure. She will need intravenous antibiotics for an extended period of time, and possibly tricuspid valve removal. Some patients do fairly well without a tricuspid valve, but others have refractory peripheral edema, liver congestion, ascites, and so forth. They may require tricuspid valve replacement, usually with a bioprosthetic valve, since mechanical valves have a high thrombosis rate in the tricuspid position. Clearly, any prosthetic valve in an IV drug abuser is subject to reinfection. High resolution CT of the chest can also be used to distinguish pulmonary septic emboli from TV endocarditis.

You see a 64-year-old woman with long-standing hypertension, mild renal insufficiency, insulin-requiring diabetes, and a cerebrovascular accident 2 years ago with residual dysphasia for a preoperative evaluation prior to hip replacement surgery for severe osteoarthritis. Since her stroke, she has had no further cerebrovascular or cardiovascular problems, and has been able to do her usual activities of daily living without chest pain or shortness of breath. She had a nuclear stress-perfusion cardiac stress test 1 year ago, which was negative for reversible ischemia. She has no other medical history and has never had major surgery. She takes long-acting insulin daily, with rapid-acting insulin at meals, daily aspirin, and hydrochlorothiazide 25 mg daily. Her blood glucose levels typically range from 90 mg/dL (fasting) to 165 mg/dL after meals. On exam, her blood pressure is 138/82 mmHg, pulse 80 bpm, respirations 12/min, and she is afebrile. There is no elevation of jugular venous pulsations, no S3, no murmur, and no rales. There is no edema. ECG is sinus, with normal intervals and no ST-segment or T wave abnormalities. Chemistry panel shows normal electrolytes, a BUN of 24 mg/dL, and creatinine of 2.2 mg/dL, which is at her baseline. Hemoglobin is 11.2 g/dL. Glycohemoglobin is 6.4%. Which of the following interventions is most likely to reduce her risk of perioperative cardiovascular complications? Coronary angiography with percutaneous coronary intervention of stenotic lesions Preoperative ACE inhibitor therapy Preoperative transfusion of packed red blood cells Perioperative beta-blocker therapy No new intervention Topic: Cardiology Answer: E. No new intervention Explanation Using the Revised Cardiac Risk Index (RCRI), this patient has 3 significant risk factors for cardiovascular complications during surgery (cerebrovascular disease, insulin-requiring diabetes, and creatinine > 2.0 mg/dL); in the past, this would be a "compelling indication" for perioperative beta-blocker therapy. However, recent data have shown that perioperative beta-blockers in many patients actually cause more harm than good in patients undergoing noncardiac surgery. Cardiac testing and/or PCI is not necessary in this patient, given the fact that she has not had an MI, and recently underwent cardiac function testing with favorable results. Her hemoglobin of 11.2 is appropriate for the operative setting without a need for transfusion. Perioperative ACE inhibitors have not been shown to reduce cardiac complications of surgery. Current recommendations are no beta-blocker therapy for most patients undergoing noncardiac surgery and with no history of MI or coronary artery disease. Those with known coronary artery disease should be on beta-blockers unless there is a contraindication. A 39-year-old woman with a prosthetic aortic valve presents with bruising. Her last INR 6 weeks ago was 2.4; today‛s INR is 6.5. She has not taken any extra warfarin. Which of the following, when taken on a daily basis, could explain her increased INR? A) Calcium carbonate B) Acetaminophen C) Oral contraceptive pills (OCP) D) Ranitidine

Answer: B. Acetaminophen Explanation This patient developed an unexplained increase in her INR. All the choices are drugs that patients can get over the counter or may get from another provider and not report (OCP). The correct answer is acetaminophen, which can increase the INR with as little as 3 extra-strength tablets a day. In one crossover study, patients on warfarin who took 4 grams of acetaminophen a day had INRs 1.75x higher than a group on warfarin and not taking acetaminophen.

A 39-year-old woman presents with 2 weeks of dyspnea at rest, orthopnea, and paroxysmal nocturnal dyspnea. She was previously well with no significant past medical history. She uses no prescription medications and has no known drug allergies. She denies a history of upper respiratory infection or use of illicit drugs or cigarettes. She is sitting upright in mild respiratory distress. HR 120 and regular; BP 100/60; RR 22; Temperature is 37.5° C JVD: 8 cm Lungs: Symmetric expansion, resonant to percussion, fine crackles to mid-lung fields bilaterally Heart: RRR with non-displaced PMI, S 1 slightly diminished, A 2 and P 2 components of S 2 are normal. A 2/6 regurgitant murmur is heard at the left upper sternal border. An S 3 and S 4 are audible at the apex. Abd: Soft, nontender with mildly tender right upper quadrant and liver span of 11 cm. Hepatojugular reflux is present. Ext: Trace pedal edema ECG: Sinus tachycardia with RBBB CXR: Pulmonary venous congestion with no focal infiltrates Serum ferritin: 200 ng/mL Echo: Global decrease in left ventricular systolic function and estimated EF of 25%. Mild mitral and tricuspid regurgitation and no evidence of significant valvular stenosis or intracardiac shunts. Which of the following is the next most appropriate therapy? A) Heart transplant consultation. B) Admit to the ICU and start intravenous furosemide and an ACE inhibitor. C) Admit to a monitored bed, order iron studies to rule out hemochromatosis. D) Admit to the ICU and start azathioprine or cyclosporine. E) Refer for urgent endomyocardial biopsy.

Answer: B. Admit to the ICU and start intravenous furosemide and an ACE inhibitor. Explanation She has acute heart failure of unknown etiology. In the absence of risk factors for atherosclerosis, the likelihood of coronary artery disease is low in this young woman. Most likely she has viral myocarditis. This is typically seen in young, otherwise healthy individuals, and the prognosis is good. With optimal heart failure management, the risk of death or need for acute transplant is low. Therefore, endomyocardial biopsy is reserved for individuals who fail to respond to conventional therapy, develop high-grade heart block, or severe ventricular arrhythmias. There is no reason to start azathioprine or cyclosporine. Hemochromatosis is unlikely given her negative review of systems and ferritin level that is only borderline high; the ferritin may also be borderline because it is an acute phase reactant. It is best to treat her heart failure with standard therapy.

Your next patient is the shipping agent at your local Gigantic Falcon Food Store, where he works a vigorous 60-hour week. He will consume 24 packs of unfiltered cigarettes during such a week and, indeed, you have already recognized his developing (emphysematous) chronic obstructive pulmonary disease (COPD). He has mentioned to you that if he tries to walk with any vigor, he develops squeezing calf pain that interrupts his exercise. His BP has inched up to 152/94 mmHg despite efforts at both weight loss and reduced salt intake. You are now seeing him in the office for what you feel sure is exertional angina pectoris. You order a stress exercise evaluation that confirms an apparent ischemic response. The patient declines your suggestion of diagnostic coronary catheterization, and you now contemplate empiric therapy. You will select sublingual nitrates, daily aspirin, and which of the following? A) Pindolol B) Atenolol C) Propranolol HCl D) Enalapril maleate E) Losartan K+

Answer: B. Atenolol Explanation A patient like this, with both hypertension and ischemic heart disease, really does warrant the use of a beta-adrenolytic agent so that the "cardio-protective" properties of this drug family may be employed. In a patient with both obstructive pulmonary disease and peripheral vascular disease with claudication, this would be the perfect scenario for the preferential use of the selective agent such as atenolol (or metoprolol, etc.). Use of propranolol in this setting would likely add a bronchospastic component such that the patient's COPD could worsen. ACE inhibitors are not considered a usual treatment for angina pectoris alone. To date, there are conflicting data regarding a possible benefit in reducing exercise-induced angina. Obviously, ACE inhibitors are becoming 1st line for therapy of hypertension, particularly in diabetic patients. Other agents proven useful in management of angina pectoris include nitrates and calcium channel blockers.

You are called to the emergency department to be introduced to a retired 66-year-old high school English teacher who has been sitting up at night for the past week, unable to lie flat for more than 15 minutes. Your examination in the emergency department reveals a HR of 100/min, respirations of 20/min, jugular venous distension to the angle of the jaw with the patient sitting bolt upright, hepatomegaly with a vertical expanse of percussive dullness of 15 cm in the mid-clavicular line, pitting ankle edema, and a murmur of tricuspid regurgitation that is holosystolic at the lower left sternal border and which increases from Grade I/VI to III/VI on inspiration (known as the Rivero-Carvallo maneuver). Careful auscultation further reveals gallop sounds early in diastole from both ventricles (RVS3 and LVS3). The lung fields are filled with diffuse inspiratory rales and crackling noises. Your patient is clearly suffering from which of the following? A) Acute pulmonary thromboembolism B) Biventricular congestive heart failure C) Cor pulmonale heart disease D) Eisenmenger transformation E) Pericardial effusion with pericardial tamponade

Answer: B. Biventricular congestive heart failure Explanation The key to the correct answer here is the noting of left ventricular gallop rhythm and pulmonary edema, thereby implicating left heart failure, as well as right. Acute pulmonary thromboembolism represents, effectively, acute cor pulmonale. Normally, we think of chronic cor pulmonale heart disease occurring in the COPD-type patient (e.g., the "blue bloater" patient with chronic bronchitis). The choice of pericardial effusion with pericardial tamponade would be unacceptable for the same reason (e.g., failing to account for the left heart failure), but also because there are no data per se presented to support a diagnosis of pericardial disease. Eisenmenger transformation requires a shunt lesion, which is not there; this patient is most often a young adult with a large ventricular septal defect whose torrential left-to-right shunting is converted, at least to a balanced shunting phenomenon, if not an actual reversal to right-to-left shunting.

A 60-year-old man presents to the Emergency Department with symptoms of increasingly frequent chest pains. He says that the pain used to occur only with significant physical activity, but the last 3 nights he has been waking up with chest pain. He says that he gets lightheaded with the chest pain on exertion and needs to sit down for the pain to go away. The pain at night will usually subside after 5 or 10 minutes, but last night the pain lasted 20 minutes. He previously smoked 3 packs-a-day of cigarettes but said he quit last week. Cardiac enzymes are negative, but ECG shows ST segment depression in lateral leads. In the ED, he has recurrence of his chest pain. Based on this history, what is the next step in the workup? A) Ask him about depression as an etiology of his symptoms. B) Cardiac catheterization. C) Thallium exercise stress test. D) Echocardiogram. E) Treadmill stress test; images are not required with his history.

Answer: B. Cardiac catheterization. Explanation OK—this guy has unstable angina or crescendo angina. He needs to get to the cath lab quickly, and discern how many vessels he needs for his CABG or if angioplasty/stenting might be beneficial. His symptoms, however, suggest left main or 3-vessel disease with the lightheadedness indicating a fall in systemic blood pressure and significant myocardial ischemia with the ECG changes noted.

Your next patient is a 29-year-old with known phenylketonuria (PKU) and for whom you have attempted to provide general care for the last 10 years. Maintenance of a low-protein diet has been sporadic at best, in part because of her constant mobility. She found herself unexpectedly pregnant last year, and she has since given birth to a child who has been evaluated by one of your pediatrician colleagues. She reports the pediatrician's observation of brachio-femoral delay and an upper:lower limb BP asymmetry of 100 systolic in both arms as compared to 80 mmHg in the legs. All pressures were obtained by careful Doppler measurements. You recognize that the child has which of the following? A) Takayasu disease B) Coarctation of the aorta C) Tetralogy of Fallot D) Kawasaki disease

Answer: B. Coarctation of the aorta Explanation The equality of the upper limbs and disparity of the upper versus lower limbs helps identify a narrowing beyond the left subclavian artery.

Which of the following is not associated with ST-segment depression? A) ST depression in V1 with a posterior MI B) Cocaine abuse C) Hypokalemia D) Digitalis toxicity E) Myocardial ischemia

Answer: B. Cocaine abuse Explanation In the setting of acute cocaine use with chest pain, look for ST-segment elevation, not depression. All of the other items cause ST-segment depression.

A 43-year-old woman presents to the emergency department with malaise, arthralgias, and a skin rash. She has had a progressive weakness for the past week. Today, she is extremely weak. She has no prior significant medical history. Physical Examination: Temp 99.5° F, pulse 80 bpm, RR 30, BP 130/80 In general, she is weak, pale, and ill appearing. Her skin exam is notable for diffuse, irregular lesions of about 5 mm in size. Some of the lesions are palpable and purpuric, while others are ulcerative. She has moderate jugular venous distention. Her heart sounds are normal except markedly decreased in intensity. Her lung sounds are clear. She has 1+ pedal edema. Neurologic exam is normal except she is very somnolent, but easily aroused. She falls asleep in mid-sentence. CXR: Pulmonary vascular congestion ECG is normal except for notable decrease in amplitude of the QRS complexes. Sodium: 132 mg/dL Potassium: 5 mg/dL CO2: 19 mEq/L BUN: 105 mg/dL Creatinine: 8.0 mg/dL Urinalysis shows hematuria with free red blood cells, as well as RBC and WBC casts. She has 2+ proteinuria. Which of the following is the next most appropriate step in diagnosis? A) Bilateral lower extremity Doppler ultrasound B) Echocardiogram C) p-ANCA D) ANA, anti-dsDNA E) Ventilation/perfusion lung scan

Answer: B. Echocardiogram Explanation She has signs and symptoms suggestive of cardiac tamponade: jugular venous distension that is sustained (no collapse during diastole), muffled heart sounds, and decreased amplitude of the QRS complexes. Because this is an emergent presentation, echocardiogram should be done immediately. Her active urine sediment is consistent with a glomerulonephritis, and her symptoms suggest the onset to be rapidly progressive. The ulcerative purpura is suggestive of granulomatosis with polyangiitis (previously known as Wegener's), a pulmonary-renal syndrome that could be responsible for the concomitant abnormal urine sediment. Likewise, a diagnosis of systemic lupus erythematosus could be entertained. Antibodies including an ANA, anti-dsDNA, and p-ANCA should be performed, but diagnosing the tamponade is more urgent. Pulmonary embolism is unlikely and, therefore, neither lower extremity Doppler ultrasound nor ventilation/perfusion lung scan is indicated.

A 28-year-old woman with diabetes presents in septic shock and develops cardiac arrhythmias. On further questioning, her husband notes that she is 20 weeks pregnant. She had been doing well until this morning when she suddenly felt ill and began vomiting. Soon afterward, she developed fever. Blood cultures are pending. She is intubated and continues to have various arrhythmias with hypotension. Which of the following is contraindicated? A) Procainamide B) Enalapril C) Verapamil D) Elective electrical cardioversion E) Digoxin

Answer: B. Enalapril Explanation Remember—she is pregnant! Of the drugs listed, only enalapril is contraindicated in pregnancy; enalapril is associated with teratogenicity. The other agents, as well as electrical cardioversion, are deemed "OK" during pregnancy. Other agents commonly queried on exams in pregnant patients are ciprofloxacin, warfarin, nitroprusside, doxycycline, and valproic acid.

You are called to evaluate a 35-year-old man who is to undergo significant tooth extraction. PAST MEDICAL HISTORY: Nonsignificant SOCIAL HISTORY: Works as a clown on the rodeo circuit; frequently gets hit in the belly by bulls and horses FAMILY HISTORY: Unremarkable PHYSICAL EXAMINATION: Essentially unremarkable For which of the following would antimicrobial prophylaxis for endocarditis be indicated? A) History of acute rheumatic fever but no cardiac murmur B) History of previous endocarditis C) Ostium primum atrial septal defect D) History of ventricular septal defect repaired 3 years ago without residual murmur E) History of coronary artery bypass grafting

Answer: B. History of previous endocarditis Explanation This would indicate significant risk for recurrent endocarditis. The other items are low-risk and do not require prophylaxis. In 2007, new AHA guidelines were published and essentially eliminated all indications for prophylaxis except for these: Prosthetic heart valves, including bioprosthetic and homograft valves Prosthetic material used for cardiac valve repair A prior history of infective endocarditis Unrepaired cyanotic congenital heart disease, including palliative shunts and conduits Completely repaired congenital heart defects with prosthetic material or device, whether placed by surgery or by catheter intervention, during the first 6 months after the procedure Repaired congenital heart disease with residual defects at the site or adjacent to the site of the prosthetic device Cardiac "valvulopathy" in a transplanted heart Common valvular lesions for which antimicrobial prophylaxis is no longer recommended in the 2007 AHA guidelines include bicuspid aortic valve, acquired aortic or mitral valve disease (including mitral valve prolapse with regurgitation and those who have undergone prior valve repair), and hypertrophic cardiomyopathy with latent or resting obstruction. Additionally, only certain dental procedures (that cause perforation of oral mucosa or involve gingival manipulation [routine cleaning is okay]), respiratory tract procedures, or skin/musculoskeletal tissue (if staphylococci or beta-hemolytic streptococci are likely) procedures require prophylaxis; genitourinary or gastrointestinal procedures no longer require prophylaxis.

An ECG for a 43-year-old female demonstrates the following rhythm: Torsades Which of the following electrolyte abnormalities is most likely to cause this rhythm? A) Hypocalcemia B) Hypomagnesemia C) Hyperkalemia D) Hypernatremia

Answer: B. Hypomagnesemia Explanation Torsades de pointes is a ventricular dysrhythmia with polymorphic QRS complexes. It can arise in many conditions, including electrolyte abnormalities, drug interactions, brain injuries, and heart blocks. The most common electrolyte abnormalities associated with torsades de pointes are hypokalemia and hypomagnesemia. Correction of these abnormalities, external pacing control, and beta-blockers are mainstays of therapy. Hyperkalemia, hypocalcemia, and hypernatremia can all affect heart function, but torsades de pointes is not a common feature of these disturbances.

Jen, a 15-year-old female, presents to your office for a routine visit. She is starting high school this year, and there are no concerns. She has never been hospitalized. She is due for her booster tetanus immunization; otherwise, her immunizations are up-to-date. Review of systems is negative. Physical Exam: Weight and height are both between the 10th and 25th percentile HEENT is within normal limits. Heart with II/VI systolic murmur heard best at left sternal border Radial pulses 2+, femoral pulses 1+, BP 100/65 Lungs are clear Abdomen is benign What would be your immediate next step? A) ECG. B) Measure blood pressure in all four limbs. C) Barium swallow. D) Begin prostaglandin E1. E) Heart catheterization.

Answer: B. Measure blood pressure in all four limbs. Explanation This patient has a heart murmur with pulses stronger in the upper extremities versus the lower extremities. Coarctation of the aorta must be ruled out. With coarctation of the aorta, upper extremity pressures are higher than lower extremity pressures. Echocardiography is the best diagnostic tool once coarctation is suspected. Surgical repair is the conventional treatment and is best if performed between the ages of 3 and 5 years.

A 61-year-old male shopkeeper presents for evaluation. He weighs 205 lbs (body mass index = 31 kg/m2 @ height of 5'8"), and smokes 2½ packs of unfiltered cigarettes daily. His latest serum cholesterol was 326 mg/dL with low- and high-density (LDL & HDL) sub-fractions of 236 and 32, respectively, and triglycerides of 290. He engages a patron in a shouting match, whereupon he is seen to clutch his chest in horror with what he describes as "crushing, heavy, smothering" chest discomfort. The local Emergency Medical Service personnel are summoned and arrive promptly; they administer oxygen and nitroglycerin after documenting a systemic arterial blood pressure of 190/110 mmHg. At the hospital, he is found to have completely normal "cardiac markers," and his electrocardiogram reveals deep symmetrical T wave inversions across all of the precordial leads. You write "unstable angina" as your impression on the chart, knowing that the most likely acute pathophysiology of this event has been which of the following? A) Intense vasospasm of the left anterior descending coronary (resulting in total occlusion) B) Mural thrombus formation on a ruptured or eroded atherosclerotic plaque C) Myocardial oxygen demand due to accelerated hypertension D) Dissection of the thoracic aorta at the left coronary ostium E) Adverse rheologic properties of the coronary circulation (via hyperlipidemia)

Answer: B. Mural thrombus formation on a ruptured or eroded atherosclerotic plaque Explanation During the past 20 years, we have learned a great deal about the pathogenesis and treatment of unstable angina. In most cases of unstable rest angina, the pathogenesis is a mural thrombus formation on a ruptured or eroded atherosclerotic plaque. All of the other factors are legitimate entities, and could even be operative in this particular patient, but the key to a correct answer is the most likely concept, rather than attempting to focus on lesser issues. Here are normal pressures: RA Pressure: 0-5; Pulmonary Artery Pressure: 12-28/3-13; Pulmonary Capillary Wedge Pressure: 3-11. Which of the following values would be associated with constrictive pericarditis? RA Pressure Pulmonary Artery Pressure Pulmonary Capillary Wedge Pressure 16 22/12 10 RA Pressure Pulmonary Artery Pressure Pulmonary Capillary Wedge Pressure 16 75/30 11 RA Pressure Pulmonary Artery Pressure Pulmonary Capillary Wedge Pressure 16 35/15 16 RA Pressure Pulmonary Artery Pressure Pulmonary Capillary Wedge Pressure 16 45/22 20 RA Pressure Pulmonary Artery Pressure Pulmonary Capillary Wedge Pressure 16 99/29 28 Topic: Cardiology Answer: C. RA Pressure Pulmonary Artery Pressure Pulmonary Capillary Wedge Pressure 16 35/15 16 Explanation Here the diastolic pressures are equal in the left and right heart chambers. This indicates external compression as is seen with constrictive pericarditis.

A 34-year-old female presents to the clinic with complaints of pain and swelling in her right leg that has been worsening over the previous 24 hours. She is markedly obese. She has no previous history of deep venous thrombosis. D-dimer measurement would be most helpful in association with which of the following conditions? A) Second trimester of pregnancy B) No swelling or signs of inflammation on right leg exam C) Doppler exam demonstrating a right leg deep venous thrombosis D) A high probability ventilation-perfusion scan

Answer: B. No swelling or signs of inflammation on right leg exam Explanation D-dimer is a fibrin degradation product that is produced during a thrombotic process. It is very sensitive to thrombosis, but there are several conditions other than deep vein thrombosis that can result in positive findings on laboratory testing. Pregnancy, malignancy, liver disease, and advanced age can all result in increased D-dimer levels. Technically, these results are not false positives, because the levels are truly elevated but may not arise from an acute thrombosis. In patients with strong evidence of thrombotic disease, a negative D-dimer is too nonspecific to impact treatment decisions. The test is most useful in patients for whom the clinical suspicion of deep venous thrombosis is low. Pain from nonthrombotic conditions may resemble a DVT, and a normal D-dimer test would rule against the presence of an acute DVT. Caution must be used, however, because the timing of the D-dimer testing can produce a false negative test if it is performed either too early or too late in the process.

A 70-year-old woman had a prosthetic hip joint placed 6 years ago. She has not been to the dentist in years and has decided that maybe she should make a visit. She read on the internet that some people need to take antibiotics before they go to the dentist. She is penicillin-allergic (anaphylaxis). Based on her history so far, which of the following is correct about whether prophylaxis is warranted? A) Yes, she should take amoxicillin 2 grams 1 hour before her visit. B) No, presence of prosthetic joints does not require antibiotic prophylaxis. C) Yes, she should take clindamycin 600 mg 1 hour before her visit. D) Yes, she should take cephalexin 2 grams 1 hour before her visit. E) The answer depends on which device she had implanted—a porcine or mechanical hip.

Answer: B. No, presence of prosthetic joints does not require antibiotic prophylaxis. Explanation There is some controversy about the use of antibiotic prophylaxis for prolonged surgery or surgery involving infected areas—but some authorities do recommend prophylaxis. But for routine dental procedures, no prophylaxis is indicated. Remember: She has anaphylaxis to penicillin—you certainly would not use amoxicillin, and you would be a little leery of using a cephalosporin too.

An 18-year-old man lives on a military base in Germany and presents to you with complaints of fever (102.2° F) and complains of lower back, knee, and left wrist pain. The pain is not localized to any one joint. He had a severe sore throat about 3 weeks ago for which he did not seek treatment. PAST MEDICAL HISTORY: Up-to-date on all his immunizations, including anthrax and yellow fever Treated for syphilis on arrival to base 8 months ago SOCIAL HISTORY: Works as a dishwasher in the mess hall Smokes 1 ppd of cigarettes Drinks 6-pack of beer nightly FAMILY HISTORY: Negative REVIEW OF SYSTEMS: Negative except for above PHYSICAL EXAMINATION: Ht 6', Wt 220, BP 120/50, Temp 102° F, P 99, RR 18 HEENT: PERRLA, EOMI, discs sharp TMs clear Throat now clear Neck: Supple; no masses Heart: RRR without murmurs, rubs, or gallops Lungs: CTA Abdomen: Bowel sounds present, no masses, no hepatosplenomegaly Extremities: Multiple swellings noted on his elbows and wrists; each approximately 0.5 cm Skin: 2 erythematous pinkish areas on the anterior trunk, each about 4-6 cm in diameter LABORATORY: CBC normal Blood cultures negative Throat culture negative ESR 93 ASO (antistreptolysin-O) titer is elevated Based on your working diagnosis, which of the following therapeutic interventions should you order at this point? A) Aspirin only B) Parenteral penicillin and aspirin C) Parenteral penicillin and aspirin and steroids D) Supportive care only E) Parenteral penicillin and steroids

Answer: B. Parenteral penicillin and aspirin Explanation This patient has acute rheumatic fever. Remember: He needs 2 of the following major manifestations: carditis, migratory polyarthritis, chorea, erythema marginatum, and subcutaneous nodules. He has 3 of the 5 major criteria. Additionally, he has fever and evidence of recent Group A Streptococcus infection with the positive ASO titer. Even if Streptococcus pyogenes cannot be isolated, you still treat with a therapeutic course of 1.2 million units of IM penicillin as 1 dose or oral penicillin for 10 days. Most authorities recommend the IM injection in the setting of acute rheumatic fever. He would then go on prophylactic penicillin q month to prevent recurrent attacks. Steroids are not indicated for treatment of the arthritis. The arthritis will usually respond quite well to aspirin. However, many believe that there is a role for steroids in patients with severe carditis accompanied by congestive heart failure. But, neither salicylates nor glucocorticoids influence the future development of valvular heart disease. In adults, prednisone can be started in doses as high as 30 mg 4 times daily in especially severe cases; and as the patient improves, salicylates can be added during the tapering of the steroid dose; this may require 4 to 6 weeks.

A 61-year-old patient seen in the emergency department presents with hypotension (88/60 mmHg), tachycardia (122/min), distended neck veins dominated by the "x" descents, and a globular cardiac silhouette on the chest x-ray. Careful analysis of her blood pressure reveals that the systolic Korotkoff sounds are first heard in exhalation only at 110 mmHg, whereas they are heard throughout the respiratory cycle only after further lowering the cuff pressure to 88 mmHg. You are well acquainted with this remarkable physical finding and immediately suspect the proper diagnosis. Which of the following interventions is most likely to benefit this patient's care? A) IV administration of propranolol B) Pericardiocentesis C) Transfusion of 2 units of packed, washed red blood cells D) IV furosemide E) IV infusion of 1.5 million units of streptokinase over 60 minutes

Answer: B. Pericardiocentesis Explanation Clinical presentation is consistent with hemodynamically significant pericardial tamponade (hypotension, tachycardia, distended neck veins with prominent "x" descent, globular cardiac silhouette and the presence of pulsus paradoxus). The treatment principles in this setting include the following: 1) Positive inotropic support 2) (Rapid) Volume infusion 3) Pericardiocentesis under controlled circumstances 4) Pericardiostomy Patients with pericardial tamponade are preload dependent so they should not receive IV furosemide (as this will drop blood pressure even more!), and because of the possibility that this could be a hemorrhagic effusion, thrombolytics are contraindicated, as they could worsen the effusion (more bleeding!).

You are asked to evaluate a 76-year-old woman admitted to the orthopedic service for a left hip fracture after suffering a fall earlier this morning. She was getting up to go to the bathroom early this morning and says that she "just passed out." PAST MEDICAL HISTORY: Hypertension for 30 years, for which she is taking: Low-dose atenolol 25 mg qd Thiazide 25 mg qd Diltiazem long-acting 240 mg qd SOCIAL HISTORY: Never married Lives alone in 40-room mansion, the "Perkins' House" Smokes 2 packs/day of cigarettes Doesn't drink alcohol REVIEW OF SYSTEMS: Dizziness has been going on for several weeks No syncope before this episode Some paraesthesias of her fingers Headaches on occasion Tongue "itchy" Eyes watery Throat scratchy Neck "swells" up Palpitations all the time Dyspnea on occasion at rest and at exertion Taste different now; has metallic taste Abdominal pains every day ... sharp like a knife, move around a lot Leg cramps all the time Skin itches a lot Increased forgetfulness Hair loss noted on forearms PHYSICAL EXAMINATION: BP 90/50, HR 42, Temp 98.5° F, RR 18 Well-developed, well-nourished woman in mild distress HEENT: PERRLA, EOMI Discs sharp TMs occluded by cerumen Throat clear Neck: Supple, no thyromegaly or masses No bruits Heart: RRR without murmurs, rubs, or gallops, but bradycardic Lungs: CTA Abdomen: Bowel sounds present, no hepatosplenomegaly, no masses Extremities: No cyanosis, clubbing, trace pedal edema GU/Rectal: Heme-negative stool, no masses; normal female external genitalia LABORATORY: ECG: Sinus bradycardia at 45 bpm; occasional sinus pauses lasting 3 seconds; no ST-T wave changes; no Q waves or other abnormalities; axis is normal Which of the following would be the best next step in treatment for her arrhythmia at this point? A) Stop diltiazem and atenolol and observe. B) Placement of a temporary pacing wire, and stop her diltiazem and atenolol. C) Placement of a permanent pacing wire. D) Place temporary pacer now and schedule for permanent placement with her hip repair. E) Give atropine q hour until heart rate returns to normal; stop atenolol and diltiazem.

Answer: B. Placement of a temporary pacing wire, and stop her diltiazem and atenolol. Explanation The patient has significant symptomatic bradycardia with spells of sinus arrest. This is most likely due to her medications, particularly the beta-blocker. Elderly patients can be particularly at risk for AV nodal blockade from beta-blockers and diltiazem, especially when used in combination. Thus, we could probably temporarily pace her and then see if the bradycardia resolves off the medications. She is symptomatic and has significant pauses, so just stopping the medication alone would not be correct; a temporary pacer is needed at this point. Using atropine would not be effective for prolonged periods of time if this were drug-induced; plus, it is likely to cause her more harm.

A 30-year-old woman with no past medical history presents complaining of 6 months of shortness of breath and fatigue. She first noticed something was wrong when she began to experience episodic painful swelling in her ankles and hands that spontaneously resolved. She has noticed a severe reduction in her ability to perform her daily routine. She now has to rest while dressing herself in the mornings. The last episode of joint swelling lasted for 2 weeks and occurred 3 months ago. Review of systems is positive for a mild nonproductive cough, increasing bilateral lower extremity edema, 25-pound weight loss over 1 year, alopecia, 3 spontaneous abortions, and Raynaud phenomenon. Review of systems is negative for orthopnea, paroxysmal nocturnal dyspnea, unilateral leg swellings, and snoring. Family history is positive for maternal hypertension. She takes no medications and denies drug use or excessive alcohol intake. Physical exam reveals a normal radial pulse, a loud P 2 component of the 2 nd heart sound, and normal lung sounds. Edema is present in both feet. Which of the following findings is also likely to be present? A) Equal a and v waves with a low-voltage electrocardiogram B) Prominent a and v waves with hepatomegaly C) Giant a waves with a blunted y descent D) Irregular cannon a waves E) Normal jugular venous pulsations

Answer: B. Prominent a and v waves with hepatomegaly Explanation This is a case of systemic lupus erythematosus (SLE) causing pulmonary hypertension and high right-sided pressures. The correct answer requires knowledge of how pulmonary hypertension affects venous waveforms. Venous waveforms reflect phases of the cardiac cycle, as seen from "the opposite direction" (i.e., in the venous system, proximal in the vasculature to where contractions are occurring). The a wave results from atrial contraction, and the x descent results from atrial relaxation. A smaller v wave occurs in atrial diastole as the atrium fills with blood against a closed tricuspid valve (ventricular systole). This is followed by a y descent when the tricuspid valve opens, and atrial blood fills the ventricle before ventricular systole. Cannon a waves are seen during periods of AV dissociation, when the asynchronous right atrium attempts to contract against a closed tricuspid valve, and the atrial blood volume rushes back into the jugular vein. Cannon a waves appear as random and irregular and are thus distinguished from normal a waves. Equal a and v waves and a low-voltage electrocardiogram would occur in cardiac tamponade. Giant a waves and a blunted y descent occur in settings where the blood flow isn't permitted in an antegrade fashion, and thus goes backward into the venous system, as in the setting of tricuspid stenosis. The jugular venous wave forms would not be normal in the setting of pulmonary hypertension with elevated right-sided pressures causing peripheral edema.

The phenomenon of torsades de pointes in its pure form is best viewed in conjunction with prolongation of which of the following interval measurements? A) PR interval B) QTc interval C) QP interval D) R-R interval E) QRS duration

Answer: B. QTc interval Explanation The phenomenon of torsades de pointes is, in reality, a special form of polymorphic ventricular tachycardia, wherein the QRS axis appears as if it is twisting around a central perpendicular axis. This is imaginative and not necessarily what is happening in the true spacial sense, but the colorful description is embraced by all admiringly with respect to those who first described it. Torsades de pointes occurs in the setting of prolonged QT interval.

A 60-year-old man with recent inferior myocardial infarction is 1-day status post his infarction and has been doing well. This afternoon, however, he develops recurrence of his chest pain. You quickly get him to the cath lab, and there he has a markedly decreased cardiac output, as well as decreased PCWP of 10. His RA pressure is elevated at 15 with a PA pressure of 20/11. He continues to deteriorate and becomes more hypotensive with a BP of 75/50. Based on these findings, which of the following describes what has happened and how you should treat him? A) Biventricular failure; give diuretics, preload and afterload reducers, and inotropic agents. B) RV infarction with secondary failure; administer normal saline. C) Pericardial tamponade; perform emergent pericardial tap. D) Mitral stenosis with secondary RV failure; refer for valvuloplasty. E) Pulmonary hypertension; oxygen.

Answer: B. RV infarction with secondary failure; administer normal saline. Explanation In this setting of a recent acute inferior MI and new recurrence of severe chest pain, you must be worried about RV infarction with secondary failure. The RV has decompensated and is unable to fill the left side. Initial treatment is to give fluids. Think of this in a hypotensive patient with inferior MI! The decompensated RV is dependent upon preload to maintain cardiac output; do NOT give these patients nitrates that will decrease preload. With pericardial tamponade, you would get equal diastolic pressures in all 4 chambers. In addition, the question might give hints in the physical exam such as distended neck veins and soft heart sounds. For biventricular failure, you would expect low cardiac output, high PCWP, and high RA pressure. Treatment listed is correct. With mitral stenosis with secondary RV failure, you would see elevated RA pressure, elevated PA press, and elevated PCWP. In pulmonary hypertension, you would see that the RA pressure is elevated and the PA diastolic pressure is markedly higher than the PCWP. The PA pressures in this example are not markedly elevated.

A patient has been transferred to your hospital because of an episode of syncope and chest pain. At the other facility, a myocardial infarction was ruled out with cardiac markers, and they are at a loss to know why he had this episode. Upon arrival, he is resting comfortably and has had no more chest pain since the event 2 days ago. He had never had a similar episode. He recalls the sudden onset of sharp chest pain while sitting watching television and then waking up with his wife leaning over him. She tells you that he lost consciousness for about 30 seconds and did not have any tonic-clonic movement or loss of bowel or bladder control. He was immediately aware of his surroundings on awakening. He denies any other medical problems except a tibial fracture of his right leg 2 years ago, when he fell off his lawn tractor while negotiating a tight turn on a hillside. On examination, his blood pressure is 130/70 mmHg, and heart rate is 67 beats/min and regular. There are no orthostatic changes. His neck veins are moderately elevated with prominent v waves. He has a prominent left parasternal lift and a 3rd heart sound along the left sternal border. He has a positive Kussmaul sign in the neck veins and a pulsatile liver. His apical impulse is in the normal location and has a normal diameter. There are no murmurs, and his lungs are clear to auscultation. His right leg is mildly edematous below the knee, but not painful. All of his laboratory data are normal except his chest x-ray and ECG. The former shows a prominent right heart border and large proximal pulmonary arteries. The left heart configuration is normal. His ECG shows sinus rhythm, a QRS axis of 80°, tall peaked P waves in the inferior leads, and deeply inverted T waves across the precordial leads. You have been provided with an ECG from 2 years ago. At that time, the QRS axis was 30°, the P waves were normal with a 15° axis, and the T waves were upright in the precordial leads. You ask for a cardiology consult, and your colleague is concerned about the chest pain, syncope, and the newly inverted precordial T waves. She strongly recommends cardiac catheterization and coronary angiography to rule out a recent coronary event, possibly causing ventricular tachycardia as a cause of the syncope. She does not understand, however, how to correlate the physical examination findings with this diagnosis, so she plans to do a right heart catheterization as well. Rather to her surprise, his coronary arteries are completely normal, and contrast left ventriculography shows no wall motion abnormalities. Hemodynamics from the left and right heart catheterization are as follows: Right atrial pressure (mmHg) 18 Right ventricular pressure 70/20 Pulmonary arterial pressure 72/35 Pulmonary capillary wedge pressure 10 Left ventricular end-diastolic pressure 8 She, being an excellent cardiologist, immediately schedules another test. Which of the following will she most likely order? A transthoracic echocardiogram A ventilation/perfusion lung scan An arterial blood gas A transesophageal echocardiogram A carbon monoxide-diffusing lung study (DLCO) Topic: Cardiology Answer: B. A ventilation/perfusion lung scan Explanation This complex patient turned out to have normal coronary arteries and pulmonary hypertension. His chronically edematous leg is the source of multiple pulmonary emboli. There is typically a 10-month delay between the onset of symptoms until the diagnosis is made, and then only when the patient has advanced pulmonary hypertension. The treatment of this condition was nearly hopeless until pulmonary thromboendarterectomy was developed. Any type of echocardiogram would show right heart enlargement and pulmonary hypertension, but the latter is obvious from the hemodynamic data. An arterial blood gas might demonstrate hypoxia, but would certainly not be diagnostic. The DLCO would be abnormal, but not specific. A 45-year-old woman comes in for her routine checkup. She relates that she is doing well. She is not aware of any problems except that her ankles swell frequently, and that this has just started to occur. Past Medical History: Essentially negative Delivered 6 children by normal spontaneous vaginal delivery Meds: None Social History: Lives with her husband and 6 children in Lyme, CT Drinks a 6-pack of beer/week Smokes 2 ppd Works as a weather forecaster Family History: Mother with CHF diagnosed at age 50 Father with CHF diagnosed at age 50 Brother 51 years old, good health Review of Systems: No shortness of breath Chronic cough especially in the morning for years No edema of the hands or elsewhere noted No orthopnea No dyspnea on exertion or at rest No weight gain or weight loss Physical Examination: Vitals: Ht 5'5", Wt 120, BP 110/80, Temp 98° F, RR 12, P 88 HEENT: PERRLA, EOMI, Discs sharp TMs: Clear Throat: Clear Neck: Supple, no masses; Visible jugular venous pulsation is 1 cm H20 above sternal angle when supine; Hepatojugular reflux test is negative Heart: RRR without murmurs, rubs, or gallops Lungs: CTA Abdomen: Bowel sounds present; no ascites; no masses Extremities: 2+ pitting edema to just past the ankle area bilaterally; No cyanosis, clubbing noted All of the following should be considered in the differential except: A) Cyclic edema B) Right heart failure C) Pelvic thrombophlebitis D) Venous insufficiency E) Hypoalbuminemia

Answer: B. Right heart failure Explanation Right heart failure can cause limb edema but it is very, very unlikely for a patient to have right heart failure with normal jugular pressures. All of the other items listed should be considered before you even think about right heart failure. Right heart failure can occur independently (e.g., RV infarct, PE) or secondarily to left heart failure. Normal venous pressure is 1-6 cm H2O. Elevated jugular venous pressure indicates elevated venous pressure. Venous pressure is estimated by measuring the height of the jugular venous distention ([JVD]; or the height of the visible jugular pulsations). When the patient is supine, 5 cm is added to the amount of visible JVD because the sternal angle is about 5 cm vertically above the right atrium. If the patient is raised 45 degrees or more, the sternal angle increases to about 10 cm above the right atrium—so no JVD should be visible at this angle in patients with normal venous pressure. Hepatojugular (abdominojugular) reflux test: Firm sustained pressure is applied to the upper abdomen for about 10 seconds. In normal patients, this causes a 1-3 cm increase in JVD. In a patient with increased venous pressure, the increase is > 3 cm.

A 73-year-old man with Type 2 DM, CAD, HTN, and CHF presents with worsening symptoms of heart failure. He has had increased paroxysmal nocturnal dyspnea and orthopnea. He has had a 15-lb weight gain in the past month. On exam, he has bilateral rales and 2+ pitting edema to the mid calf. Labs: HCT 32 (was 37 a month ago), Bun 20, Cr 1.1. Meds: glyburide, metformin, lisinopril, rosiglitazone, carvedilol, spironolactone, and nortriptyline. What is the most likely cause of his increased CHF symptoms? A) Spironolactone B) Rosiglitazone C) Carvedilol D) Metformin E) Lisinopril

Answer: B. Rosiglitazone Explanation This patient has worsening edema and dilutional anemia. The drugs pioglitazone and rosiglitazone both cause edema. Worsening edema and dilutional anemia are fairly common in patients with known heart failure or who are at risk for heart failure, who take these drugs. Lisinopril, carvedilol, and spironolactone are all treatments for CHF and should not cause worsening. Rarely beta-blockers like carvedilol may cause increased symptoms if given to patients with decompensated CHF. This patient did not have class 4 CHF, so the carvedilol would be unlikely to worsen symptoms. Metformin is unlikely to do this.

A 50-year-old woman had an anterior wall MI 3 days ago. She has been doing well and has not had any problems since her admission. Suddenly, she develops severe chest discomfort and collapses while walking in the hallway. Her BP is 70/palpable with thin thready pulses. She is hypoxic with an oxygen saturation of 88%. She now has a new loud holosystolic murmur along the left lower sternal border. What has most likely happened? A) She has had a thrombus embolize to her brain. B) She has had a ventricular septal rupture occur. C) She has had another infarction. D) She has had a papillary muscle rupture. E) She has coronary artery rupture.

Answer: B. She has had a ventricular septal rupture occur. Explanation This is a life-threatening condition, and she must proceed to surgery immediately. The murmur is the clue here and eliminates the other choices listed. Rupture of the free wall is also possible with sudden syncope and signs of tamponade—this is more common in older hypertensive women. Papillary dysfunction would result in severe MR and pulmonary edema with short, early systolic decrescendo murmurs in odd locations. Ventricular septal rupture usually is manifested by a loud systolic murmur and HF/shock, depending on the defect size and degree of RV/LV dysfunction. It occurs most often within the first 24 hours in patients with STEMI treated with fibrinolytic therapy. Emergency surgical repair is necessary regardless of hemodynamics because the rupture site can expand abruptly, resulting in sudden hemodynamic collapse in previously stable patients. Emergency treatment consists of inotropic and vasodilator agents, with IABP when needed. The surgical mortality rate remains high, ranging from 20% to 87%. Mortality risk is higher for patients with inferior-basal defects than for those with anterior-apical defects, and those who present in shock. Ventricular septal rupture occurrence is more common in anterior MI's, as is rupture of the free wall. Papillary muscle dysfunction/rupture is more common with inferior MIs (especially of the posteromedial papillary muscle as it recieves a single blood supply source from the right coronary artery in most people).

A 55-year-old man is referred to you for evaluation of a heart murmur, recently heard by your nurse practitioner. On exam, you find that he has a systolic ejection murmur heard best in the right upper sternal border. An ejection click is heard just after S 1 . S 2 is muffled. You can make the murmur louder by having him expire or squat. Which of the following would you expect to find on examination of his carotid artery? A) Giant right-sided a waves B) Slowed carotid upstroke C) Water-hammer pulse D) Thready pulse E) Bifid pulse

Answer: B. Slowed carotid upstroke Explanation This patient has the classic murmur of aortic stenosis. The murmur is best heard at the right upper sternal border and will radiate into the carotid artery. Congenital bicuspid aortic valves usually become calcified and stenotic between the ages of 50 and 70. Usually the normal tricuspid valve will become stenotic at > 80 years of age. A bicuspid aortic valve is the most common inherited valvular disorder and the commonest cause of aortic stenosis. Patients with aortic stenosis will have a slowed carotid upstroke. Water-hammer pulse is seen with chronic aortic regurgitation. Thready pulse is seen with acute aortic regurgitation. Giant right-sided a waves are seen with tricuspid stenosis. Bifid pulse is often seen in hypertrophic cardiomyopathy.

A 77-year-old man with known ischemic heart disease and heart failure is admitted to your hospital with severe dyspnea. He reports that his baseline shortness of breath on climbing one flight of stairs has progressed over the past week to where he now has to sit sleeping upright in a chair. He denies chest pain or other symptoms. His other medical problems include diabetes and mild chronic renal failure. A previous echocardiogram showed segmental wall motion abnormalities of the left ventricle and an ejection fraction of 20%. On examination, his heart rate is 105 and blood pressure is 110/65 mmHg. His neck veins are elevated to the angle of the jaw in the sitting position. His lungs have diffuse crackles and wheezes throughout. He has a left parasternal lift, and the apical impulse is palpated to be 4 cm in diameter. There are left parasternal and apical third heart sounds, and a soft apical holosystolic murmur with radiation to the anterior axillary line. His laboratory is remarkable for a hemoglobin of 6.8 g/dL and an MCV of 68 fL, a BUN of 67 mg/dL, and a creatinine of 2.3 mg/dL, and a glucose level of 237 mg/dL. Cardiac markers, including CK and troponin I, are negative. A chest x-ray shows cardiomegaly, pulmonary edema, and bilateral pleural effusions. An electrocardiogram shows sinus tachycardia and an old extensive anterior myocardial infarction. Which of the following statements is true? A) The diameter of the apical impulse is consistent with left ventricular hypertrophy. B) This patient should have a cautious transfusion of packed red blood cells to raise his hemoglobin level. C) Most patients with severe heart failure die of sudden cardiac death. D) There is no evidence of pulmonary hypertension in this patient. E) Obtaining a new echocardiogram is likely be extremely useful in managing this patient.

Answer: B. This patient should have a cautious transfusion of packed red blood cells to raise his hemoglobin level. Explanation This patient has decompensated severe systolic heart failure. The approach to such a patient must consider the underlying cause, the precipitating cause, and the management of the heart failure state itself. The underlying cause is almost certainly the well-documented ischemic heart disease. Perhaps subsequent efforts to evaluate this man for potentially viable areas of "hibernating" myocardium may be useful, but they are not indicated now. The precipitating cause, on the other hand, must be considered. In this case, a new ischemic event must be ruled out, and the severe anemia rectified. The microcytic nature of the anemia suggests ongoing blood loss, and an appropriate evaluation is indicated subsequently. Blood should be replaced slowly with packed red blood cells. An apical impulse of greater than 3 cm in diameter correlates very strongly with left ventricular enlargement. Most patients with severe systolic dysfunction die of relentless progression of heart failure, with sudden death as the next commonest cause. There are several indications of pulmonary hypertension, including the high neck veins, the left parasternal lift, and particularly the right-sided third heart sound.

A 30-year-old man presents to the emergency department with 18 hours of left precordial pain that is worse when lying supine and relieved when sitting upright. He is afraid he is having a heart attack because his grandma (age 92) died last week of an MI. He had a viral illness about a week ago. PAST MEDICAL HISTORY: Negative SOCIAL HISTORY: Works as a psychic on the psychic hot line Married with children Doesn't smoke or drink FAMILY HISTORY: Grandma, mentioned above, died at age 92 of MI Grandpa died at age 92 of stroke Father age 65 and mother age 70, alive and well REVIEW OF SYSTEMS: Low-grade fevers Occasional cough He says his psychic abilities have been diminished the last 2 days PHYSICAL EXAMINATION: BP 130/70, P 110, RR 20, Temp 99.5° F HEENT: PERRLA, EOMI Throat: Slight erythema Neck: Supple, no masses; no bruits Heart: RRR with faint friction rub when he leans forward Lungs: CTA Abdomen: +BS; no masses; no hepatosplenomegaly Extremities: No cyanosis, clubbing, or edema LABORATORY: ECG reveals diffuse concave-up ST elevation with sinus tachycardia Your likely next steps would include all of the following except: A) Admission for serial ECGs and observation B) Thrombolytics for treatment of an acute MI C) Echocardiogram D) Nonsteroidal antiinflammatory agents E) Supplemental oxygen by mask or nasal cannula

Answer: B. Thrombolytics for treatment of an acute MI Explanation This patient has acute pericarditis by his symptoms and physical examination. It is very unlikely that this healthy young man has had an MI. Diffuse concave-up ST elevation is seen with pericarditis, as opposed to localized concave-down ST elevation seen in an acute MI. Everything else listed you would do for symptomatic relief. Admission is warranted to follow his course and look for common etiologies, although it is likely that this will be idiopathic after his viral infection. Finally, thrombolytics in this setting is not only wrong (there is no MI) but could cause a hemorrhagic pericardial effusion and tamponade.

A 50-year-old woman is referred to you for evaluation of a pulsatile abdominal mass. Her history is remarkable for long-standing hypertension, as well as smoking for 30 years (1 pack/day). She has had some mild abdominal pain that brought her initially to her local physician. PAST MEDICAL HISTORY: Negative as above SOCIAL HISTORY: Works in Sun City, Arizona as a transcriptionist (notes she has been more forgetful lately and leaves her transcription equipment in odd places like the bathroom) Married with 2 children Smoking history as above FAMILY HISTORY: Essentially unremarkable REVIEW OF SYSTEMS: No intermittent claudication No peripheral edema No chronic stasis changes No fevers No cough No chills No weight loss PHYSICAL EXAMINATION: BP 120/70, Temp 98.6° F, P 70, RR 18 She is in no acute distress. HEENT: PERRLA, EOMI TMs clear Throat clear Neck: Supple Heart: RRR without murmurs, rubs, or gallops Lungs: CTA Abdomen: Bowel sounds present; there is an ill-defined mid-epigastric pulsatile mass without bruit Extremities: No cyanosis, clubbing, or edema LABORATORY: Ultrasound of the abdomen shows a 5.6-cm infrarenal abdominal aortic aneurysm. Which of the following is the next best step in her management? A) Place her on warfarin. B) Vascular surgery consultation for aneurysm repair. C) Repeat abdominal ultrasound every 6 months until the aneurysm reaches 6 cm. D) Observation is best until she develops symptoms, then proceed to repair. E) Have her wear a tight-fitting corset.

Answer: B. Vascular surgery consultation for aneurysm repair. Explanation In 2005, the ACC/AHA guidelines were published. She has an infrarenal abdominal aortic aneurysm (AAA) that has surpassed the threshold level of 5.5 cm. Below 4 cm, these aneurysms are low-risk for rupture. Between 4 and 5.4 cm, infrarenal or juxtarenal AAAs should be monitored by ultrasound/CT every 6-12 months to detect expansion. In those with AAAs smaller than 4.0 cm, most recommend ultrasound monitoring every 2-3 years. Intervention is not recommended for asymptomatic infrarenal or juxtarenal AAAs if they measure < 5 cm for men and < 4.5 cm for women. In this case, now that she has been found to have a 5.6-cm aneurysm, the best choice is to proceed with surgery.

A 27-year-old used car salesman presents with his latest episode of palpitations and rapid heart action. He relates countless such episodes since his late teens and 1 episode of syncope. He felt as if he would lose consciousness with today's episode, but did not. You confirm a heart rate of nearly 220/min and succeed in obtaining a 12-lead ECG during tachycardia showing no R-R interval variability, narrow QRS complexes (100 msec), and discernible P waves hidden in the succeeding ST segments with a QP interval of 110 msec. The patient produces a previous routine tracing from his billfold; it reveals a PR interval of 0.07 sec (70 msec) and slurred sluggish upstrokes as the initial portion of the QRS complexes in multiple leads. Your best diagnosis is which of the following? A) Multifocal atrial tachycardia (MAT) B) WPW with orthodromic tachycardia (AVRT) C) First-degree AV block with Mobitz 1 progression (Wenckebach's) D) AV nodal re-entrant tachycardia (AVNRT)

Answer: B. WPW with orthodromic tachycardia (AVRT) Explanation The key to the correct diagnosis here is the noting of the QP interval (> 100 msec). In AVNRT, the P wave is hidden in the QRS complex. These are the patients with congenital dual AV pathways whose re-entry loop is restricted to the area surrounding the AV node. In the patient with AVRT, the normal impulse during tachycardia will travel antegradely down the normal conduction pathway and then will use the Kent pathway for the retrograde limb of the re-entry loop. What this means, however, is that there is a measurably longer distance and greater time needed for the advancing impulse to return to atrial myocardium in order to excite a P wave, the latter then being delayed and appearing "down the page" in the ensuing ST segment. Wenckebach (1st degree AV block with Mobitz 1 progression) would not explain tachycardia; WPW with antidromic tachycardia should be a wide complex tachycardia; and multifocal atrial tachycardia should be irregular. Some authors place the diagnostic threshold of the QP interval at shorter levels (e.g., 70 msec).

Brain natriuretic peptide (BNP) is a hormone released from cardiac tissue that acts on receptors in the kidneys, smooth muscles of the blood vessels, and the adrenal glands. The overall effect is to increase urine production and sodium excretion, reduce vascular resistance, and diminish the release of renin and aldosterone. For which of the following situations is measurement of BNP most useful? A) A patient with elevated JVP, pulmonary congestion, peripheral edema, and an S 3 on auscultation B) A patient with hypertension, a renal bruit, and evidence of fluid retention C) A patient with chronic COPD, basilar crepitus, and pedal edema presenting with increased shortness of breath and the etiology of the worsening symptoms is in question D) A patient with a transient ischemic attack, a right-sided carotid bruit, and a history of hyperlipidemia

Answer: C. A patient with chronic COPD, basilar crepitus, and pedal edema presenting with increased shortness of breath and the etiology of the worsening symptoms is in question Explanation BNP measurement can help elucidate the presence of congestive heart failure as a component of a patient's clinical status. In a patient with a classic presentation of CHF (elevated JVP, pulmonary congestion, peripheral edema, and an S 3 ), the BNP is expected to be elevated, and its presence adds nothing to the diagnosis or treatment. The use of the word "brain" in BNP is a historical misnomer, and measurement in the evaluation of a neurologic presentation without evidence of CHF is unnecessary. Likewise, a patient with evidence of renal artery stenosis unassociated with CHF is not an indication to measure BNP. Its greatest clinical utility is in cases where both cardiac and noncardiac causes can mimic the findings of CHF. Acute shortness of breath may occur with COPD, with CHF, or with both. A low level of BNP is consistent with a pulmonary etiology for the symptoms. An elevated level suggests a cardiac contribution to the presentation. The 2006 Heart Failure Society of America (HFSA) guidelines recommend that BNP or NT-proBNP levels be assessed in all patients suspected of having HF when the diagnosis is not certain.

A 43-year-old woman comes to your office complaining of 6 months of increasing dyspnea. Recently she has had to stop virtually all activities except moving about the house. In the course of her evaluation, an echocardiogram demonstrates a greatly enlarged right atrium and right ventricle with a pulmonary artery systolic pressure of 95 mmHg. The left heart has normal size and function. She is eventually diagnosed as having primary pulmonary hypertension. Which of the following would you expect to hear on auscultation of her second heart sound? A) A single second heart sound B) A soft P2 C) A widely split second heart sound D) Paradoxical splitting of the second heart sound E) A fixed split of the second heart sound

Answer: C. A widely split second heart sound Explanation This patient with pulmonary hypertension will have a delayed emptying of the right ventricle because of the severe increase in afterload from the pulmonary vessels. This will result in a widely split second heart sound. P2 will be loud in this patient. A second heart sound with only P2 is associated with severe aortic stenosis. Paradoxical splitting of the second heart sound occurs with severe aortic stenosis, permanent pacemaker, and LBBB. A truly fixed split of the second heart sound is the hallmark of an atrial septal defect.

A 63-year-old female who has been a patient in your clinic for many years returns for ongoing evaluation related to complaints of shortness of breath, which has been developing over several months. One week previously, you started an evaluation for symptoms and findings of congestive heart failure. Initial lab work was obtained, and the patient was scheduled for echocardiography. Remarkable among the results were significant protein in her urine and mild elevations of BUN and creatinine. Her chest x-ray showed an increase in the cardiothoracic ratio, with mildly increased pulmonary vasculature. No infiltrates were reported. Echocardiogram report noted no pericardial effusion, near-normal intraventricular dimensions, symmetric biventricular thickening, hypokinesis, and diffuse "granular sparkling." On examination, the patient is in no marked distress at rest. She has a small amount of petechiae around the eyes bilaterally, and there is a general fullness to the tongue. She has mild basilar rales that improve but do not resolve with deep breathing. Cardiac exam demonstrates a regular rate and rhythm without rub, gallop, or murmur. The abdomen is benign, and a normal liver span is noted. There is a mild amount of pitting edema bilaterally. No other prominent skin lesions or discolorations are noted. Neurologic examination was within normal limits. Which of the following is most likely to be the cause of this patient's cardiac condition? A) Hemochromatosis B) Sarcoidosis C) Amyloidosis D) Fabry disease

Answer: C. Amyloidosis Explanation Restrictive myocarditis arises in several pathological conditions when the heart is infiltrated with abnormal depositions. These include amyloidosis, Fabry disease, hemochromatosis, and sarcoidosis. Patients with sarcoidosis are uncommonly affected with cardiac manifestations (~ 5%), whereas lung involvement occurs in nearly 95% of sarcoid patients. A near-normal CXR makes sarcoidosis much less likely. Abnormal iron deposits are the hallmark of hemochromatosis. These deposits cause dysfunction of the involved organs, the liver is primarily affected, and hepatomegaly is very common. Also, iron pigments in the skin result in a bronzed appearance in the majority (~ 90%) of patients. Cardiac involvement is found in approximately 15% of hemochromatosis patients. Fabry disease is caused by the absence of an enzyme required for proper metabolism of lipids (ceramide trihexosidase). Lipids then accumulate in eyes, kidneys, autonomics, and cardiovascular system causing symptoms in adolescence and resulting in death by the fifth decade. Other characteristics include distal paresthesias, lymphedema of the legs, episodic diarrhea, eye opacities, and angiokeratosis. Renal failure often predominates. This patient lacks the neurologic, eye, and bowel problems, which make Fabry disease less likely. Amyloidosis is one of the more common causes of restrictive myocarditis. It often presents with unexplained proteinuria, CHF, and intermittent abdominal cramping and diarrhea. The findings on echocardiogram are typical of amyloidosis and are referred to as a speckled or diffuse granular pattern. The diagnosis of amyloidosis can be confirmed with biopsy of the oral mucosa or the abdominal subdermal tissue.

A 35-year-old woman presents to your office with chest pain for the past 6 months. She describes the pain as a non-radiating substernal pressure sensation, lasting from a few minutes to as long as 4 hours. She does not get the pain while walking her dog around a local soccer field every evening, but she occasionally has it when lying down at night. It has been occurring more frequently over the past 4 weeks. Her only other medical problems are moderate obesity and hypertension. She does not smoke and is not taking oral contraceptives. She is not aware of her cholesterol level, but she remembers being told that her blood sugar was previously normal. Her family history is unremarkable. On physical examination, her blood pressure was 140/90 mmHg and heart rate was 85. Other than obesity, the examination was normal. A complete blood count and chemistry panel were normal except for a cholesterol level of 220 mg/dL. An electrocardiogram showed normal sinus rhythm and nonspecific ST and T wave changes. Which of the following statements is true regarding this patient? A) If she has 2 mm of upsloping ST segment depression during a standard exercise stress test, she is extremely likely to have significant coronary disease. B) She has a < 5% chance of having a significant stenosis in 1 or more coronary arteries. C) An exercise capacity of 10 METs will risk stratify this patient to a very low risk class. D) A stress test using a myocardial perfusion imaging study (e.g. IV technetium 99m) will not improve the accuracy of a standard exercise stress test in this patient. E) Standard exercise stress testing will have a high degree of accuracy in determining if she has coronary artery disease.

Answer: C. An exercise capacity of 10 METs will risk stratify this patient to a very low risk class. Explanation This woman is having atypical chest pain, defined as such because the description of the chest pain is appropriate for angina but the precipitating factors are not. A 35-year-old woman with atypical chest pain has a 12% chance of having at least a 70% stenosis of 1 major coronary artery. Standard exercise stress testing will lack accuracy, according to Bayes theorem, because of the low prevalence of the disease in the population. Atypical chest pain occurring in a middle-aged man, on the other hand, would be associated with a very high degree of accuracy because of the 50% prevalence of coronary disease in that population. Upsloping ST segment changes during exercise, particularly in the presence of resting ST segment abnormalities, also lack specificity. An exercise technetium 99m imaging would likely improve the accuracy of the exercise stress test, particularly in this situation. An exercise capacity of 10 METs (indicating 10 times basal oxygen consumption) is consistent with an extremely low 5-year cardiac mortality in any patient (and less than 2% in this woman).

Erectile dysfunction is often associated with cardiovascular processes. Newer interventions, including medications, are being developed to improve erectile function. Among these are medications such as sildenafil (Viagra®), tadalafil (Cialis®), avanafil (Stendra®) and vardenafil (Levitra®, Staxyn®). Which of the following conditions is most likely to result in an adverse event when one of these agents is administered? A) Atrial fibrillation and warfarin B) Mitral valve prolapse and propranolol C) Angina and isosorbide dinitrate D) Hypertrophic cardiomyopathy and verapamil E) Congestive heart failure and digitalis

Answer: C. Angina and isosorbide dinitrate Explanation Significant hypotension is a serious complication that is recognized when sildenafil and the other agents are used in combination with nitrate medications. Due to the risk, this combination is to be actively avoided. Although the other conditions all have significant morbidity associated with them, the adverse event risk when combined with these agents is much less than that noted with nitrates.

A 26-year-old Caucasian female is referred by her gynecologist because of hypertension. The gynecologist noticed that her blood pressure was in the 160-170/95-105 range on several readings 3 months ago, so her birth control pills were discontinued. Her BP has remained elevated. Her medical history is otherwise negative, and she has no FH of hypertension. PE: BP 164/104 in both arms, funduscopic exam is normal, there is no S4 or bruits. Labs: Na 140, K 3.8, Cl 104, CO2 26, BUN 11, Cr 0.7. EKG, biochemistry profile, and U/A are normal. A renal angiogram shows the following: beaded appearance of renal arteries Which of the following agents would be the most appropriate initial therapy? A) Thiazide diuretic B) Amiloride C) Angiotensin converting enzyme inhibitor D) Spironolactone

Answer: C. Angiotensin converting enzyme inhibitor Explanation This young Caucasian female presents with sudden onset of hypertension. Her gynecologist correctly considered the possibility that her hypertension was secondary to oral contraceptives, but after 3 months with no improvement, one must suspect another etiology. The clinical presentation is the clue in that sudden onset of hypertension in a young, especially Caucasian, female should raise the possibility that this woman has renal artery stenosis secondary to fibromuscular dysplasia. Other clues to the diagnosis would be a renal (abdominal) bruit, but 30% of patients with fibromuscular dysplasia will not have a bruit. Secondly, hypokalemia may occur because hypertension in renal artery stenosis is mediated by stimulation of renin and therefore aldosterone. The diagnosis is confirmed by renal arteriography. The arteriogram demonstrates the characteristic "beaded" appearance of the renal artery. Patients should be placed on an ACE inhibitor (or angiotensin II receptor blocker), and then a decision regarding angioplasty or surgical bypass can be made. A thiazide would therefore not be appropriate, especially in light of the reduced serum potassium. Beta-blockers should be used as initial therapy in essential hypertension, particularly in younger Caucasian patients. Amiloride would be appropriate if the patient had hypertension and hypokalemia due to Liddle syndrome. Spironolactone would be appropriate therapy for hypertension and hypokalemia if it was secondary to primary hyperaldosteronism.

In a patient with cardiac involvement related to Lyme disease, the most common abnormality is: A) Pericardial inflammation B) Mitral and aortic valve dysfunction C) Atrioventricular conduction defects D) Coronary artery inflammation

Answer: C. Atrioventricular conduction defects Explanation Cardiac disease associated with Lyme disease is most commonly manifested with conduction abnormalities including first degree, Wenckebach, and complete heart blocks. Valvular disease is more common with rheumatic heart disease and endocarditis conditions. Coronary artery inflammation is linked with Kawasaki disease. Pericardial inflammation follows autoimmune and some infectious processes.

You are called to admit a 32-year-old male who presented to the emergency department with complaints of chest pain. He is admitted to a monitored floor and, during the course of hospitalization, he is shown to have experienced a myocardial infarction. A homocysteine level is found to be very high. Which of the following is not routinely included in the treatment of an acute MI in this young patient? A) Beta-blockers B) Aspirin C) B vitamin supplements D) Statins E) ACE inhibitors

Answer: C. B vitamin supplements Explanation Homocystinemia can arise in several situations including nutritional deficiencies, medication side effects, and as a genetically based condition. High levels of serum homocysteine were thought to have been linked to early-onset cardiovascular disease. Levels of homocysteine can be significantly improved with the use of B vitamin supplements, particularly B 12 , B 6 , and folate. However, recent data has shown that lowering elevated homocysteine levels does not improve mortality or morbidity in patients with an acute MI.

A 59-year-old man comes to your office because of a history of "congestive heart failure." He reports increasing dyspnea over the past 2 years to the point where he can now walk only about 50 feet before having to stop. He also complains of being somewhat lightheaded when he stands up. He denies chest pain, orthopnea, or paroxysmal nocturnal dyspnea. He shows you the pills he was prescribed by his previous physician and, after a quick consultation with the "Physician's Desk Reference," you conclude that they are digoxin and furosemide. His risk factors for coronary artery disease include age, male sex, and hypertension. On examination, his blood pressure is 180/110 mmHg and his heart rate is 85 while sitting. Upon standing, his blood pressure is 145/85 and his heart rate is 105. His neck veins are not elevated. His lungs have moderate bibasilar crackles. He has a very prominent apical impulse and a low-pitched sound heard best at the apex just before the 1st heart sound. The remainder of the examination is normal. Laboratory data are all normal except for a BUN of 38 mg/dL and a creatinine of 2.2 mg/dL. His chest x-ray shows cardiomegaly with moderate pulmonary vascular redistribution. His electrocardiogram shows sinus rhythm with voltage criteria for left ventricular hypertrophy. There are nonspecific ST-segment and T-wave changes, particularly in the precordial leads. For a better understanding of his heart failure, you obtain an echocardiogram. Which of the following is the most likely finding? A) Minimal left ventricular hypertrophy, four-chamber cardiac enlargement, and a left ventricular ejection fraction of 10% B) Concentric left ventricular hypertrophy and severe aortic stenosis C) Concentric left ventricular hypertrophy and a left ventricular ejection fraction of 80% D) Concentric left ventricular hypertrophy, segmental wall motion abnormalities, and a left ventricular ejection fraction of 15% E) Normal-sized, non-hypertrophied ventricles with a strange speckled pattern, huge atria, and a left ventricular ejection fraction of 45%

Answer: C. Concentric left ventricular hypertrophy and a left ventricular ejection fraction of 80% Explanation This case represents a common misunderstanding of the hemodynamics of heart failure. This man has severe hypertension but is being treated incorrectly. His chest x-ray indicates that he has a large heart and a high pulmonary capillary wedge pressure, but it does not explain the underlying pathophysiology. He has diastolic dysfunction because of the stiffness of his hypertrophied ventricle, but nothing about the physical examination suggests left heart systolic dysfunction or valvular heart disease. He has a prominent apical impulse, indicating an actively contracting left ventricle, and a 4th heart sound caused by his high left ventricular end-diastolic pressure. If the proper therapy is not used, he will eventually end up with poor left ventricular function.

dyspnea accompanied by a stabbing right thoracic pain that increases with each inspiration. The respiratory rate is 28/min; systemic arterial pressure is 100/84 mmHg. The jugular pulsations are noted to be elevated nearly to the earlobes with the patient sitting bolt upright; no clear pattern is discerned, although they seem to pulsate at a rate equal to the patient's arterial pulse of 110/min. A pleural rub is heard at the site of the chest discomfort. A right ventricular lift is apparent, and the pulmonary artery is palpable. S2 is split widely, and P2 is loud and palpable. The patient reports having just completed a nearly nonstop 4,300-mile round-trip solo drive of an 18-wheeler to another garage on the West Coast and back. He is also known to have recently diagnosed unresectable adenocarcinoma of the colon, for which he refused further treatment. The patient admits to smoking 2½ packs per day of cigarettes since he was 15 years old (i.e., > 100 pack years). This patient has an acute version of which of the following disorders? A) Tricuspid regurgitation due to ruptured chordal apparatus B) Tricuspid stenosis due to malignant carcinoid syndrome C) Cor pulmonale due to pulmonary thromboembolism D) Pericardial tamponade resulting from metastatic pericardial disease via bronchogenic carcinoma E) Pericardial constriction due to previous blunt chest trauma (steering wheel injury via motor vehicle accident)

Answer: C. Cor pulmonale due to pulmonary thromboembolism Explanation It is very important to appreciate the fact that acute cor pulmonale represents pulmonary thromboembolism almost by definition. This gentleman may represent another variation on the so-called "economy class syndrome" theme that has been a concern in the commercial air transportation community, although the nature of the association, if any, remains uncertain. Of far more importance in this particular patient may be the neoplasm, the association between a true hypercoagulable state and known cancer. This patient has all the features of (acute) pulmonary hypertension and right heart failure. The systemic venous hypertension, as reflected in his jugular venous pulsations, reminds us of the differential diagnosis of this important physical finding, which is: 1) Right heart failure 2) Tricuspid valve disease 3) Pericardial compressive physiology 4) Superior vena cava syndrome Note that the patient with superior vena cava syndrome would have distended, pulseless neck veins as well as the other features that so often accompany the Pancoast tumor—e.g., Horner syndrome, collar of Stokes, facial suffusion, and a characteristic chest x-ray. On the Board exam, look for patients with acute tricyclic overdose.

A 16-year-old high school student is referred by his high school wrestling coach to you for a physical examination. He is healthy and has no complaints. PAST MEDICAL HISTORY: No immunizations since age 12 (had Td booster then); has received 2 MMRs; doesn't know about hepatitis B vaccination SOCIAL HISTORY: C student Sleeps about 10 hours a night Smokes marijuana on occasion Smokes a cigarette on occasion Drinks beer on occasion Is not sexually active Doesn't wear seat belts FAMILY HISTORY: Mother 44, healthy Father 44, healthy Brother 16, healthy Brother, died suddenly while skiing at age 17; no autopsy obtained REVIEW OF SYSTEMS: Negative PHYSICAL EXAMINATION: 5' 11", 250 lbs BP 130/70, P 72, RR 18, Temp 98.7° F HEENT: PERRLA, EOMI TMs clear Throat clear Neck: Supple; no murmurs heard in neck Brisk carotid upstroke Heart: RRR with III/VI harsh systolic murmur Murmur increased with standing Murmur decreased with squatting Lungs: CTA Abdomen: Bowel sounds present; no hepatosplenomegaly Extremities: No cyanosis, clubbing, or edema GU: Tanner IV pattern hair growth and gonadal development Skin: Acne on face and back Which of the following diagnostic tests will confirm your diagnosis? A) Holter monitor B) ECG C) Echocardiogram D) Doppler ultrasound of his scrotum E) Treadmill stress test

Answer: C. Echocardiogram Explanation This patient has hypertrophic cardiomyopathy. All aspects of his physical examination correlate with this diagnosis: the harsh systolic murmur that does not radiate to the neck, increases with standing and decreases with squatting. The history of his brother dying of "sudden" death supports the familial form of this disease and increased risk for sudden cardiac death. An echocardiogram is diagnostic test of choice and will show you the asymmetric septal hyperthrophy with small left ventricular cavity and systolic anterior motion of the mitral valve. A treadmill stress test (blood pressure response) and Holter monitor (prolonged or repetitive episodes of nonsustained ventricular tachycardia) are often done as a part of risk stratification to estimate risk for sudden cardiac death; however, neither will confirm diagnosis. The ECG can be suggestive of hypertrophic cardiomyopathy. It may show Q waves in the inferior to lateral leads (from the hypertrophied septum) and have a normal-to-increased voltage (depending on the degree of LVH) with anterolateral T wave inversions consistent with repolarization abnormality. Athletes with HCM (with or without obstruction) should not participate in competitive sports.

Your next new patient admitted to your team on the Inpatient Service is a 62-year-old man who has suffered from congestive heart failure for the last 7 years. He has attempted to take his medications faithfully, but the complexity of his regimen and his failing eyesight have sometimes resulted in inadvertent errors. In the emergency department, he is noted to have a narrow complex tachycardia at a rate of 88/min with apparent retrograde P waves hidden in the ST segments. You recognize non-paroxysmal junctional tachycardia (NPJT) and understand this rhythm to reflect a nonspecific acceleration (to within the range of 70-130/min) of what would otherwise represent the normal rate range of the junctional tissues (40-60/min) in a resting adult. The best descriptor for this known mechanism of arrhythmia is which of the following? A) Delayed after-depolarizations B) Atrio-ventricular bypass (via a congenital anomalous pathway) C) Enhanced automaticity D) Reentry E) Early after-depolarizations

Answer: C. Enhanced automaticity Explanation Although not seen on a daily basis, the phenomenon of non-paroxysmal junctional tachycardia is exceedingly important both from a practical clinical point of view and a conceptual example. Arguably, this is the very best example of "enhanced automaticity," the junctional tissues having been accelerated nonspecifically by 1 or more of several factors such as the following: 1) Digitalis excess 4) Post-operative state 2) Hypokalemia 5) Post-prandial hypoglycemia 3) Post-infarction state 6) Other Magnesium sulfate may have a role in treatment, although the usual focus is the correction of the spectrum of reversible contributors operative in the specific patient. The rate range of 70-130/min is very important, particularly given the overlap with the normal range for the resting sinoatrial node. By way of review, the junctional rhythms are: ## Name Rate Range Significance 1 Junctional escape ("idionodal rhythm") 40-60/min Default pacemaker 2 Non-paroxysmal junctional tachycardia (NPJT) 70-130/min As above; never "normal" or "chronic" 3 Paroxysmal junctional tachycardia (PJT) 150-250/min Rare, rapid, perhaps "incessant"

A 24-year-old woman presents to your office complaining of palpitations. These occur most often when she drinks coffee. She has never had syncope. The palpitations can last up to 3-5 minutes and spontaneously resolve. PAST MEDICAL HISTORY: Negative SOCIAL HISTORY: Works as a cross-country truck driver Married with 1 child Smokes 2 packs/day of cigarettes Drinks a fifth of vodka on the weekends FAMILY HISTORY: Mother 60 with HTN Father 50 with HTN REVIEW OF SYSTEMS: Negative PHYSICAL EXAMINATION: BP 110/70, P 90, RR 16, Temp 98.8° F, Ht 6' 1", Wt 230 lbs HEENT: PERRLA, EOMI TMs clear Throat clear Neck: Supple Heart: RRR without murmurs, rubs, or gallops Lungs: CTA Abdomen: Benign Extremities: No cyanosis, clubbing, or edema GU: Normal female external genitalia Which of the following would be the next most useful test in evaluating this patient? A) Stress test B) Left heart catheterization C) Holter study D) Echocardiogram E) Right heart catheterization

Answer: C. Holter study Explanation She likely is having an arrhythmia (such as SVT) from her caffeine intake. The best test would be to monitor on a Holter and then see which arrhythmia is occurring. The other tests are not indicated at this time.

A 17-year-old senior is the starting guard on the state basketball championship team. He has been offered a full scholarship at a university after never missing a game in 4 seasons and setting the all-time school record in both total points and assists. Seated in the stands for the final game of the season, you are summoned when he collapses at mid-court after a thunderous game-ending dunk shot. You assist in his full resuscitation from ventricular fibrillation, accompanied in this effort by the local paramedics who were there in 3½ minutes. With an opportunity to examine him in the hospital, you are struck by his bifid carotid impulses—which are mirrored in his apex cardiogram, the latter of which you are able to both palpate and project via shadows on the bed clothing. He has a harsh holosystolic murmur at the lower left sternal border that accentuates with the upright posture as well as the Valsalva strain. Occasional premature contractions are followed by radial artery impulses, which are diminished relative to the apparent sinus cycle pulsations. You feel certain that the young man has which of the following? A) Acquired (muscular) ventricular septal defect (VSD) B) Myxomatous mitral valve prolapse (MVP) C) Hypertrophic obstructive cardiomyopathy (HOCM) D) Ostium secundum atrial septal defect (ASD) E) Ostium primum atrioventricular septal defect (AVSD

Answer: C. Hypertrophic obstructive cardiomyopathy (HOCM) Explanation The combination of: 1) "spike and dome" carotid pulse contour, 2) similar apex cardiogram, 3) holosystolic murmur, 4) murmur augmentation by standing, 5) murmur augmentation by Valsalva strain, 6) the Brockenbrough phenomenon (premature contractions followed by diminished radial artery impulses), and 7) sudden death (resuscitated) makes the diagnosis of hypertrophic (obstructive) cardiomyopathy inescapable here. The pure form of this disease is now known to be an autosomal dominant transmission, thus explaining the importance of investigating other members of the kindred, particularly in the type of scenario described. This entity is one of the several diagnoses that is customarily first considered when there is an acute event in an elite athlete. Other entities worth consideration in this regard are WPW, Marfan physiology, anomalous coronary disease, and long QTc syndrome. Appropriate associations with the other entities mentioned might include: Myxomatous mitral valve prolapse (MVP): mid-systolic click Ostium secundum atrial septal defect (ASD): fixed splitting of S2 Ostium primum atrioventricular septal defect (AVSD): RBBB/LAFB Acquired (muscular) ventricular septal defect (VSD): myocardial infarction Wolff-Parkinson-White syndrome (WPW): delta waves Congenital long QT syndrome: QTc > 0.43-0.44

You are a passenger on an Antarctic cruise. You have just attended a grand soirée in the main dining room, and you and your spouse have walked up to the front deck to catch a breath of fresh air. Immediately ahead of the mighty ship looms a towering white mass. The ship is slowly turning to the left, but strikes what is obviously an iceberg. A large chunk of ice breaks off and falls onto the chest of a young woman who is standing nearby. She crumples to the deck and you rush to assist her. One of the ship's crew runs over to help you, and together you move her unconscious form to Captain Smith's cabin. The ship's surgeon arrives moments later. He is immediately concerned upon palpating the woman's pulse. It seems that her blood pressure is very low. The surgeon examines her chest and notes a large mid-sternal hematoma. He palpates her sternum and tells you that it is obviously fractured. You, being the observant person you are, point out her extremely distended neck veins. The surgeon examines the neck closely and remarks about the single collapse of the veins during each heartbeat. He pulls out his stethoscope and listens to her heart and lungs. Captain Smith enters the cabin to inquire about the young lady's condition. He assures you that his initial impression is that the ship has received only minor damage from the collision. You continue to observe the actions of the surgeon. Which of the following is he most likely to do next? A)Apply a tight wrap to her chest to stabilize the sternum. B) Apply rotating tourniquets to her arms and legs. C) Insert a long, large-bore needle into her left chest toward her cardiac apex. D) Insert a large trocar through her left lateral chest wall. E) Call for the ship's chaplain to read the last rites to her.

Answer: C. Insert a long, large-bore needle into her left chest toward her cardiac apex. Explanation This poor woman has had severe chest trauma. Her physical neck veins are consistent with acute tamponade, certainly from a traumatic hemopericardium. A tension pneumothorax would produce distended pulseless neck veins and absent breath sounds on 1 side. Rotating tourniquets was an old therapy for heart failure, which she could have conceivably had from coronary artery trauma and myocardial infarction, but the treatment would not have been sufficient.

An 18-year-old woman is standing in line for college admission testing when she begins to feel lightheaded and falls to the ground. She is unresponsive. No seizure activity is noted. Her vital signs are stable. Though she looks somewhat pale, she regains consciousness within 3 minutes and within 15 minutes is back to her previous state. She says that she felt weak, and the room became "dark" to her just prior to "falling out." She says that she has not been ill and otherwise has had no medical problems. She does admit to frequently feeling lightheaded upon rising quickly from the supine or seated positions and says that she may have passed out once before in church. What is the most likely explanation for her syncopal episode? A) Hypertrophic cardiomyopathy B) Anomalous coronary artery C) Neurocardiogenic (vasovagal) syncope D) Long QT syndrome E) Severe aortic stenosis

Answer: C. Neurocardiogenic (vasovagal) syncope Explanation Neurocardiogenic (vasovagal) syncope is the most common cause of syncope in young adults. The history of feeling lightheaded, with tunnel vision, while standing, is typical. The other entities can also present with syncope but are far less common. As many as 16% of individuals will faint at least once in their lives.

A 50-year-old man presents to the emergency department with a history of crescendo-pattern chest pain and is admitted to the CCU. While in the CCU, he develops more chest pain, and the nursing staff records a 12-lead ECG that shows 15-mm ST-segment elevation in leads V3 and V5 with additional lesser elevations in leads I, aVL, and V6. These changes revert to normal almost immediately following the administration of sublingual nitroglycerin. Based on your clinical and ECG findings, which of the following medications would be contraindicated in this patient? A) Diltiazem B) Lovastatin C) Propranolol D) Nifedipine E) Isosorbide dinitrate

Answer: C. Propranolol Explanation Based on his clinical findings of crescendo type angina and the quickly reversible ECG findings of ST-segment elevation, he likely has variant angina (formerly known as Prinzmetal's). The use of the nonselective beta-blocker propranolol has been shown to actually prolong the duration of vasospasm in patients with variant angina. Nifedipine, sorbite dinitrate, and diltiazem would all be therapeutic and help prevent the spasms. Lovastatin would not help prevent the spasms but would not be contraindicated. Aspirin should be used with caution and is likely best avoided because aspirin inhibits prostacyclin production.

All of the following right heart sounds become louder on inspiration except which of the following? A) Pulmonic regurgitation murmur B) Tricuspid stenosis murmur C) Pulmonic ejection sound D) Tricuspid regurgitation murmur E) Right-sided 3rd heart sound

Answer: C. Pulmonic ejection sound Explanation Inspiration decreases intrathoracic pressure and increases venous return to the right heart. Right atrial and right ventricular volume are increased, resulting in all of the right heart sounds becoming louder except the pulmonic ejection sound. The latter is a sound produced by the tensing of a stenotic pulmonic valve in early systole in patients with congenital pulmonic stenosis. A larger right ventricular volume causes the pulmonic valve to begin its upward excursion in early systole from a higher position, resulting in less distance to be traveled before tensing occurs, and, therefore, a softer sound.

A 65-year-old man presents for surgical repair of bilateral inguinal hernias that he has had for over 3 years. He denies chest pain or dyspnea on exertion. He says he feels healthy as a horse. PAST MEDICAL HISTORY: Inferior myocardial infarction 3 months ago Hypertension for 15 years MEDICATIONS: Isosorbide dinitrate 20 mg tid Captopril 25 mg tid Atenolol 20 mg qd Enteric-coated aspirin qd SOCIAL HISTORY: Works as a carhop at a local drive-in Returned to work 6 weeks after his MI Rollerskates to work, which is about 3 miles away Mows his lawn on the weekend Quit smoking in 1972 2 glasses of red wine once a week FAMILY HISTORY: Father died of MI at age 70 Mother died of MI at age 67 Brother died of MI at age 56 REVIEW OF SYSTEMS: Negative PHYSICAL EXAMINATION: BP 120/70, P 60, RR 18, Temp 99.2° F HEENT: PERRLA, EOMI, developing cataract in left eye TMs clear Throat clear Neck: Supple, no bruit Heart: RRR without murmurs, rubs, or gallops Lungs: CTA Abdomen: Bowel sounds present, no hepatosplenomegaly Extremities: No cyanosis, clubbing, or edema Laboratory: CXR normal ECG: Q waves in leads II, III, and AVF; otherwise normal Which of the following is most appropriate at this point? A) Approve for surgery now, depending on results of echocardiogram. B) Wait 9 months before proceeding to surgery. C) Schedule surgery in 3 months, if clinically unchanged. D) Approve for surgery in 3 months, depending on the results of a radionucleotide stress test. E) Proceed to surgery now.

Answer: C. Schedule surgery in 3 months, if clinically unchanged. Explanation In the majority of cardiac patients having noncardiac surgery, the highest potential risk is from coronary artery disease. In this patient with a history of recent MI, the incidence of reinfarction during a major noncardiac surgery averages approximately 6%. The risk, however, is inversely proportional to the time since the MI. For example, within the first 3 months the risk is 37%; after 6 months, it falls to 4-5%. In this patient, it would be better to wait another 3 months before doing this elective surgery. Additionally, he is having exercise testing daily with his skating to work for 3 miles and his lawn mowing on the weekend. Studies have shown that an exercise capacity of 5-6 METs indicates that cardiac risk during noncardiac surgery is low. Mowing the lawn easily reaches this threshold; therefore, formal testing is not indicated and is just an added expense.

Which of the following conditions is most appropriate for pharmacologic (non-exercise) stress testing? A) Diabetes mellitus B) Cardiac pacemaker C) Severe claudication D) Digitalis therapy

Answer: C. Severe claudication Explanation Exercise stress testing is useful for identifying patients with impaired cardiac perfusion. Patients with ischemia exhibit typical ECG findings that aid in the diagnosis of coronary artery disease. Several conditions interfere with the ability of the test to demonstrate the usual changes in the ECG tracing. Among these are bundle-branch blocks, pacemaker-driven rhythms, and baseline ST interval changes. The effects of digoxin also interfere with the ECG identification of ischemia. These conditions often require the addition of perfusion or echocardiogram studies. Severe claudication often restricts individuals from achieving sufficient coronary activity to induce ischemia but does not interfere with the ECG findings. Pharmacologic stress testing can be performed in patients who cannot exercise adequately. A diabetic with no other complicating conditions should be able to perform a conventional exercise ECG study and should not require the use of a pharmacologic agent to stress the heart.

Which of the following statements regarding digoxin is true? A) The commonest arrhythmia produced by digoxin toxicity is atrial fibrillation. B) The effectiveness of digoxin antibody therapy for digoxin toxicity should be assessed by subsequent serum digoxin levels. C) Signs of digoxin toxicity may be present in a patient with serum levels in the therapeutic range. D) Toxic level of digoxin can be removed with hemodialysis. E) Digoxin has been shown to have an adverse effect on survival in patients with left ventricular ejection fractions less than 25%.

Answer: C. Signs of digoxin toxicity may be present in a patient with serum levels in the therapeutic range. Explanation Digoxin has no effect on survival in patients with heart failure but has been shown to improve symptoms and decrease readmission rates. Digoxin is not removed by dialysis. The commonest arrhythmia associated with digoxin toxicity is multiforme premature ventricular contractions. Ectopic atrial tachycardia, particularly with various amounts of AV block, and accelerated junctional rhythm are the two most specific arrhythmias associated with digoxin toxicity. Once digoxin antibodies have been administered, serum digoxin levels are meaningless, and the dose must be based on the initial level. Digoxin has a very narrow therapeutic to toxic range, and patients can exhibit signs of toxicity with serum levels in the "normal" range.

A 54-year-old male, who works as a postal carrier, presents to the clinic with complaints of bilateral lower leg pain. He normally drives a delivery truck on his mail route, but occasionally must walk to deliver items. He has noted pain in the lower legs. It occurs at about 200 meters of walking or after climbing two flights of stairs. The cramping pain improves after 3-5 minutes of rest, and then he can resume activity. He is a social drinker and smokes 1 pack/day. He denies any illicit drugs. On physical examination, there is significant difficulty palpating the dorsalis pedis and posterior tibialis pulses bilaterally. The capillary refill of the nail beds on the feet is mildly delayed. Which is the most appropriate next intervention? A) Institute dipyridamole therapy. B) Advise patient to restrict activity as much as possible. C) Smoking cessation. D) Prescribe support stockings. E) Refer for lower extremity angiography.

Answer: C. Smoking cessation. Explanation Claudication stems from peripheral vascular compromise. The use of dipyridamole has come under scrutiny, and its benefit at this time is very suspect. The only drug shown to improve symptoms consistently is cilostazol (Pletal®), but remember not to use it in patients with heart failure (black box warning!)—best therapy at this point would be aspirin and exercise with the smoking cessation—if this did not improve symptoms, then cilostazol would be the next step. Support stockings have no role in the treatment of claudication and may actually worsen symptoms. Referral for invasive procedures is more appropriate when surgical intervention is indicated. The indications for surgical intervention in lower extremity peripheral vascular disease include nonhealing ulcers, rest pain, and claudications that interfere with the requirements of daily living. At this point, the patient experiences symptoms but is still able to complete his duties. Rest is actually counterproductive, and patients are to be encouraged to exercise to the point of toleration to encourage the development of collateral circulation. The best intervention for this stage of disease is aggressive efforts toward smoking cessation, exercise, and daily aspirin.

A 76-year-old man was shooting pool when he collapsed to the floor. His friends report that he was lining up a shot when he suddenly lost consciousness, was unarousable for about 20 seconds, had no jerking movements, and did not lose control of his bowel or bladder. When he woke up, he was completely aware of his surroundings and was ready to resume his game, but his friends insisted that he come to your emergency department. Upon questioning, the man denies any cardiac symptoms or history, although he admits to having 3 previous episodes of syncope. His risk factors for coronary artery disease include age, male sex, and hypertension. On examination, his blood pressure is 110/65 mmHg, and heart rate is 165 beats/min. He has prominent regular cannon a waves in the neck veins. His cardiac and neurological examination is otherwise normal. His basic electrolytes, complete blood count, and cardiac markers are all normal. His electrocardiogram shows a narrow QRS tachycardia at a rate of 168. Since he is hemodynamically stable, you decide to admit him to the telemetry unit and observe his course. He does well, still at the same heart rate, until 2 hours later when he gets up to use the bedside commode. He then climbs back into the bed and faints. A rhythm strip of the whole event is captured by the monitoring equipment. Which of the following most likely describes the findings on the rhythm strip? A) A 30-second episode of 3rd degree AV block with a slow ventricular escape rhythm B) An acceleration of the ventricular rate to 330 from 165, indicating that the initial rhythm was atrial flutter with 2:1 AV block C) Sudden cessation of the narrow QRS complex tachycardia followed by a 15-second period of asystole D) A 30-second episode of torsade de pointes E) A 30-second episode of rapid ventricular tachycardia

Answer: C. Sudden cessation of the narrow QRS complex tachycardia followed by a 15-second period of asystole Explanation This man has the classic tachycardia-bradycardia syndrome in which a tachyarrhythmia, in this case the common form of PSVT (producing the regular cannon a waves in the neck), stops suddenly as a good reentrant arrhythmia should, and a long pause occurs before the sinus node begins capturing the atrium properly. Careful electrophysiological studies have shown that most of these pauses are caused by SA exit block; that is, the SA node is firing properly but not capturing the atrium. This occurs because of senile infiltration of the conduction system by fibrous tissue, the same cause of most AV nodal and bundle-branch blocks. The fibrous tissue is rendered refractory by a supraventricular tachycardia, most frequently atrial fibrillation and atrial flutter, and the patient experiences a long pause that can produce fainting. Treatment involves permanent pacemaker implantation to protect against the bradyarrhythmia, and then whatever treatment is appropriate for the tachyarrhythmia. There is no reason why this man, without evidence of structural heart disease, should have ventricular tachycardia. Third-degree AV block would be unlikely to occur suddenly upon termination of a supraventricular tachycardia. There is no reason to suspect torsade de pointes. An atrial rate of 330 bpm is very fast for atrial flutter, and this rhythm would not produce regular cannon a waves.

2 days ago, a 50-year-old man had an acute anterior STEMI. He is recovering after stent placement in his LAD coronary artery and is doing well. Past Medical History: Peptic ulcer disease 5 years ago; on no medications Diabetes mellitus for 20 years; on insulin Gout 2 years ago; on no medications currently Morbid obesity Only other medicine he is on is an antihistamine for allergies. Vital Signs: BP 130/70, P 90, RR 20, Temp 98° F Besides his morbid obesity and early diabetic retinopathy, he has no abnormalities on physical examination. Based on his history, which of the following is true? A) Lipid-lowering agents should be used only if his LDL is not below 150. B) An ACE inhibitor is contraindicated. C) The use of beta-blockers is recommended even though he has diabetes. D) Aspirin is not recommended. E) Short-acting calcium channel blockers have been shown to prolong survival post MI.

Answer: C. The use of beta-blockers is recommended even though he has diabetes. Explanation The use of beta-blockers in diabetics is not contraindicated. In someone who has had a STEMI, start oral beta-blockers (metoprolol tartrate, carvedilol) within 24 hours as long as there are no contraindications; e.g., heart failure/low-output state; increased risk for cardiogenic shock; PR interval > 0.24 seconds; 2nd or 3rd degree heart block without a pacemaker; active asthma/reactive airways disease (use diltiazem). ASA is begun and continued indefinitely. ACE inhibitors are indicated within 24 hours to all with anterior STEMI, heart failure, or EF ≤ 0.40 (unless contraindicated). High dose statins (e.g., atorvastatin 40-80 mg qd) should be started on all patients who have atherosclerotic cardiovascular disease regardless of their LDL level per the 2013 ACC/AHA lipid guidelines. Calcium channel blockers have not been shown to prolong survival post MI.

A 25-year-old man presents after having a syncopal episode while playing basketball this morning. He has no other symptoms and says that he feels fine now. Physical Examination: HEENT: Normal Neck: Carotid pulse has a rapid upstroke and is bifid Heart: RRR with a harsh, non-radiating midsystolic aortic murmur Lungs: Clear to auscultation Abdomen: Benign GU: Normal male genitalia Skin: No rash On further testing, you have him perform a Valsalva maneuver and his murmur increases in intensity. Which of the following statements is true? A) Several drugs have been shown to prolong survival in these patients. B) Beta-blockers and verapamil will not improve symptoms. C) This patient has a common cause of sudden death in exercising young people in the United States. D) Coronary artery disease is likely. E) The carotid pulse finding in this disorder is also seen in aortic stenosis.

Answer: C. This patient has a common cause of sudden death in exercising young people in the United States. Explanation This patient has hypertrophic cardiomyopathy (HCM), which is the most common cause of sudden death in exercising young people (<35 years of age) in the United States. Anomalous coronary arteries are the next most common cause of sudden death in exercising young people (<35 years of age). No medications have been shown to prolong survival in HCM—only Implantable Cardioverter Defibrillator (ICD) and heart transplant has. Beta-blockers and verapamil will improve symptoms, however.

A 54-year-old man is enjoying himself at a local restaurant when he develops the sudden onset of severe dyspnea. He has no previous cardiac history and has been in good health otherwise. His wife drives him to your emergency department, whereupon he has a respiratory arrest and requires intubation and mechanical ventilation. On physical examination, his heart rate is 120 and his blood pressure is 85/50 mmHg. His neck veins are elevated to the angle of the jaw in the sitting position. His lungs have diffuse crackles and wheezes. There is a rapid heart rate without obvious extra sounds or murmurs. His extremities are cold and diaphoretic. Initial laboratory data shows normal CK and troponin levels. An electrocardiogram shows sinus tachycardia. A chest x-ray shows a normal-sized heart and gross pulmonary edema. An urgent echocardiogram is obtained, which shows severe mitral regurgitation. Which of the following statements is true? A) If a murmur were to be heard in this patient, it would be holosystolic. B) This abnormality is more likely to occur in women. C) This patient should have an immediate cardiothoracic surgical consultation. D) Blood cultures are likely to be positive, but that does not affect immediate management. E) Transient ischemia of the posterior papillary muscle is the most likely cause of this condition.

Answer: C. This patient should have an immediate cardiothoracic surgical consultation. Explanation This patient with acute mitral regurgitation is in shock and needs to have urgent cardiac surgery. Death is virtually certain without surgical intervention. The sudden volume load on the previously normal left atrium causes severe elevation of pulmonary venous and capillary wedge pressures, resulting in massive pulmonary edema. The left ventricular forward stroke volume decreases drastically. If a murmur were heard, it would be early systolic because of the nondilated, noncompliant state of the normal left atrium. In chronic mitral regurgitation, the large, compliant left atrium allows the left ventricle to eject blood into it throughout systole, producing a holosystolic murmur. The most common cause of acute mitral regurgitation in middle-aged men is rupture of a myxomatous chordae. There is nothing to suggest endocarditis or ischemic heart disease in this patient's presentation. This abnormality and the severe form of mitral valve prolapse frequently associated with it are more common in men.

to share a few cold ones with the Friday crowd. After his 3rd mug of beer, each followed by a double shot of whiskey, he lurches off the back of the stool, landing on the floor, and appears unconscious. Trained in the basics of Advanced Cardiac Life Support (ACLS), the bartender quickly confirms that he indeed has a pulse with an apparent rate of 145/min. As the EMS team arrives on the scene, the patient has regained consciousness, though very intoxicated. They use their "quick-look" paddle electrodes and demonstrate clearly a wide complex tachycardia, which is quite regular and continues at the rate of 145/min. The QRS duration is 0.185 sec, and there appear to be P waves at a much slower rate. As the medics prepare to administer protocol-driven treatment, they call you in the emergency department describing all of the above. You tell them that this indeed is most likely which of the following? A) Ventricular fibrillation (VF) B) Multifocal atrial tachycardia (MAT) C) Ventricular tachycardia (VT) D) Sinus tachycardia with bundle-branch block (BBB) E) AV nodal reentry tachycardia (AVNRT)

Answer: C. Ventricular tachycardia (VT) Explanation From a test-taker's point of view, there are 8 key points in this case, as follows: Older adult Alcohol Syncope Tachycardia at a rate of about 150/min Regular rhythm Wide QRS complexes Atrioventricular dissociation Ventricular rate greater than atrial rate With that data base, one is searching for the answer that fits the ECG criteria, which is sufficient to cause syncope, and which the patient could survive long enough to permit the depiction shared by the paramedics. As such, ventricular tachycardia can be the only answer. No one "regains consciousness" during ventricular fibrillation. AV dissociation excludes AV nodal re-entry tachycardia and sinus tachycardia. Multifocal atrial tachycardia is not regular. This question in particular is included to demonstrate the process of diagnostic clinical reasoning. The association with alcohol intake is well documented.

A 41-year-old used car salesman comes to see you in the emergency department after developing one of his "spells," during which his heart feels as if it will "jump out of (his) chest." These episodes begin abruptly, particularly if he doubles his normal daily caffeine intake (which usually consists of 3 cups of coffee and 2-3 cans of cola). He is concerned that he will pass out with today's episode, although he has never done so before. You quickly establish that the patient's HR is 150/min and supine BP 110/70 mmHg. The patient is placed on a monitor, which confirms the presence of a wide complex tachycardia. A 12-lead ECG is obtained during tachycardia and reveals a precisely regular rhythm without discernible P waves, QRS-duration of 185 milliseconds, a mean electrical QRS axis of -110° in the frontal plane (toward the patient's right shoulder), and a rather monomorphic QRS configuration with large dominant R waves in V1. You would surmise which of the following? A) This could not be antidromic WPW because the QRS complexes are too wide. B) Either digitalis or adenosine IV may be given with impunity as the drugs most likely to convert the tachycardia. C) This could not be ventricular tachycardia with the apparent preservation of hemodynamic stability. D) A calcium antagonist given intravenously (IV) cannot be selected safely as the initial treatment. E) The patient must be cardioverted emergently.

Answer: D. A calcium antagonist given intravenously (IV) cannot be selected safely as the initial treatment. Explanation While absolute statements are seldom possible in medicine, one in which you may be confident is the following: "Calcium antagonism is never the treatment for a patient with wide complex tachycardia in the emergency setting." If the diagnosis were actually traditional ventricular tachycardia, the calcium antagonist would have no primary efficacy and would simply add the adverse effect of negative inotropicity. If the diagnosis were antidromic conduction of a supraventricular tachycardia in the setting of preexcitation physiology, the calcium antagonist would retard conduction at the atrioventricular (AV) junction and thereby enhance anomalous antegrade conduction. The majority of patients who present with ventricular tachycardia have underlying structural heart disease. However, there has been increasing appreciation of the existence of multiple forms of idiopathic ventricular tachycardia with distinct features and unique mechanisms. The most common form of idiopathic ventricular tachycardia originates from the right ventricular outflow tract, which is characterized by sensitivity to adenosine, and appears to be due to cyclic AMP-mediated triggered activity. Other forms of idiopathic ventricular tachycardia include intrafascicular left ventricular tachycardia due to reentry, which is sensitive to verapamil, and automatic, propranolol-sensitive ventricular tachycardia. Although the mechanism of verapamil-sensitive idiopathic left ventricular tachycardia (ILVT) is usually reentry, the actual reentrant circuit is not clearly understood. This latter entity would be the sole exception to the general rule stated above, one that would not apply directly to the acute scenario as described.

A 70-year-old man is admitted to another hospital with an acute anterolateral myocardial infarction and is transferred to your tertiary care medical center 2 days later for further management. He had coronary bypass surgery 8 years ago. Because he continues to have mild chest pain, you refer him to your colleague for coronary angiography. The procedure shows occlusion of the left anterior descending and the right coronary arteries and a high-grade stenosis in the proximal circumflex artery. His internal mammary artery is a small vessel that is grafted to the distal left anterior descending. There is very slow flow through the graft, and a 90% stenosis of the left anterior descending distal to the insertion of the graft. The saphenous vein graft to the obtuse marginal branch of the circumflex is widely patent. The vein graft to the distal right coronary has a 95% irregular stenosis in its mid portion. The cardiologist successfully deploys a stent in the right coronary graft. The patient recovers without further complications. He exercises for 7 METs on a treadmill prior to discharge without chest pain or ECG changes. He is placed on secondary prevention medications and discharged to home. Four days later, he is readmitted to your service complaining of atypical chest pain, nausea, vomiting, and diarrhea. His neck veins are not elevated; he has a few wheezes in both lungs, and some mild abdominal tenderness. His laboratory data show a white blood count of 15,800 and a bicarbonate of 20 mEq/L. You make a diagnosis of possible viral gastroenteritis and treat him with intravenous fluids. The next morning he is clearly worse, now with severe dyspnea, audible wheezing, and confusion. On examination, his blood pressure is 140/85 mmHg, heart rate is 125, and his temperature is 102.2° F. He has prominent a and v waves in the neck, and the veins are elevated to the jaw at 30°. He has diffuse rales and wheezing in both lungs, and he is using his accessory muscles of respiration. His heart is very difficult to hear through the respiratory sounds. He groans as you palpate his abdomen, but his belly is soft and there are a few scattered bowel sounds. His extremities are cold and somewhat mottled. There is a bluish-black appearance to the end of one of his toes. The impressive parts of his laboratory data are his metabolic acidosis, high white blood count, and an arterial blood gas that shows a pH of 7.15, pO2 of 50 mmHg, pCO2 of 40 mmHg, and a bicarbonate of 12 mEq/L. He is transferred to the critical care unit, where he is intubated and placed on mechanical ventilation. An emergency echocardiogram is obtained. Which of the following is the most likely finding? A) Rupture of the tip of the posterior papillary muscle and severe mitral regurgitation B) A ventricular septal defect C) Rupture of the anterior wall of the left ventricle with a large pericardial effusion and signs of tamponade D) A large, lobulated, mobile mass in the apex of the left ventricle E) Extremely poor left ventricular function with an ejection fraction of 10% and elevated right heart pressures

Answer: D. A large, lobulated, mobile mass in the apex of the left ventricle Explanation This poor man had a recent large myocardial infarction and suffered a severe complication. All of the findings point to a severe metabolic acidosis, probably from mesenteric infarction from an arterial embolism from the apical thrombus. His normal blood pressure alone rules out cardiogenic shock, acute mitral regurgitation, acute ventricular septal defect, and free-wall rupture—the 4 causes of sudden shock after myocardial infarction.

A 45-year-old Caucasian male CIA agent stationed in a Middle Eastern country develops the acute onset of severe dyspnea. He has no prior cardiac history and denies having chest pain. By the time he is taken to a hospital, he is in acute respiratory distress and requires intubation and mechanical ventilation. He has a very abnormal chest x-ray with total unilateral opacification. A preliminary diagnosis of acute pneumonia is made, blood cultures are drawn, and he is started on antibiotics. After 48 hours, however, he is hypoxic on 100% FiO2 and is becoming progressively hypotensive. You are immediately flown halfway around the world and are then escorted to his bedside. You learn fthat he has no known medical problems and does not smoke. He was completely well prior to the onset of symptoms. On examination, you find a fit-appearing man who, at the moment, is obviously extremely ill. His blood pressure is 70 mmHg systolic, his heart rate is 120 beats/minute, and he is afebrile. His neck veins are elevated to the jaw with his torso inclined to 30°. He has a very prominent apical impulse. He has a rapid rate with a soft apical S3. At times, through the respiratory noise, you think you hear a soft, early systolic decrescendo murmur near the apex. There are coarse breath sounds throughout the left lung and diminished breath sounds on the right. You confirm the chest x-ray findings. His white blood count is normal. The blood cultures are not growing anything, and tracheal cultures show normal flora. An ECG shows sinus tachycardia, but is otherwise normal. You have brought along a new miniaturized echocardiogram machine, but the echo is extremely difficult technically because of his supine position and being on a mechanical ventilator. Which of the following is the likely cause of this man's severe illness? A) Acute aortic regurgitation from infectious endocarditis; he is a closet IV-drug abuser B) An acute myocardial infarction with cardiogenic shock C) An acute myocardial infarction with a new ventricular septal defect D) Acute mitral regurgitation from rupture of a myxomatous chordae E) Unilateral pneumonia

Answer: D. Acute mitral regurgitation from rupture of a myxomatous chordae Explanation The patient has acute severe mitral regurgitation from rupture of a myxomatous chordae of the mitral valve. The clues are the sudden onset of symptoms in an otherwise healthy middle-aged man, the prominent apical impulse (which means this is not a left ventricular function problem), and the early systolic decrescendo murmur. Unilateral opacification on a chest x-ray is an uncommon but well-reported presentation of pulmonary edema. There is no evidence of a myocardial infarction, particularly with the impressive apical impulse and normal electrocardiogram. There cannot be a new VSD because the murmur would be hard to miss and, again, he has no evidence of an acute infarction. Acute aortic regurgitation could present exactly like this, except that it should produce an early diastolic murmur. Transesophageal echo, had it been available, would easily have made the correct diagnosis, since the echo image would be unaffected by his mechanical ventilation

You are seeing a new patient who is a 65-year-old man with a history of poorly controlled hypertension. Three months ago, he developed shortness of breath, increased fatigue, and two-pillow orthopnea. Last month while he was traveling in Eureka Springs, Arkansas, he consulted a physician about these symptoms and was started on digoxin (0.25 mg/day), hydrochlorothiazide (50 mg/day), and potassium supplementation. At that time, he weighed 190 lbs with a blood pressure of 150/80 mmHg. He said his doctor said something about "JVD" of 4 cm. He had swelling of his ankles at that time also. An ECG from the visit 1 month ago showed left ventricular hypertrophy. CXR at that time showed cardiomegaly without effusion. Routine lab then showed BUN of 28 mg/dL and serum creatinine of 1.4 mg/dL. PAST MEDICAL HISTORY: Poorly controlled hypertension SOCIAL HISTORY: Lives with wife and dog Smokes ½ pack/day of cigarettes Drinks 1 beer a week FAMILY HISTORY: Father died of MI at age 67 Mother died of MI at age 66 REVIEW OF SYSTEMS: No fever Morning cough—clears with persistent coughing Weight loss of 7 pounds Still fatigued but a little better PHYSICAL EXAMINATION: BP 130/79, P 96, RR 18, Temp 99° F, Weight 183 lbs HEENT: PERRLA, EOMI TMs clear Throat clear Neck: Supple; neck veins flat; no JVD Heart: RRR without murmurs, rubs, with an S3 gallop Lungs: Few scattered crackles in bases Abdomen: Bowel sounds present; no hepatosplenomegaly Extremities: No cyanosis, clubbing, or edema LABORATORY: BUN 45 mg/dL Serum creatinine 1.6 mg/dL Serum sodium 136 mEq/L Serum potassium 3.6 mEq/L Serum digoxin 1.8 ng/mL (therapeutic 1.0-2.0) Which of the following is the best course to take at this time? A) Add calcium channel blocker. B) Change hydrochlorothiazide to furosemide. C) Increase digoxin dose. D) Add an ACE inhibitor. E) Add nothing at this point and follow up in 1 month.

Answer: D. Add an ACE inhibitor. Explanation Clinical presentation is consistent with heart failure. Considering presence of S3 gallop, he likely has HF with reduced EF. He needs appropriate medical therapy and echocardiogram (the most useful diagnostic test in the evaluation of patients with heart failure). Adding the ACE inhibitor will increase cardiac output and decrease LV filling pressure due to the vasodilatory effect. ACE inhibitors block formation of angiotensin II, causing an increase in renin (from decreased negative feedback). They have been shown to reduce morbidity and mortality in patients with systolic HF. Increasing the digoxin dose would not be a good idea based on his current digoxin level (dose should actually be reduced to achieve serum levels between 0.5 and 1.0). There is no need to go to a loop diuretic at this point because the hydrochlorothiazide is working well. A calcium channel blocker would not be effective and could be harmful due to a possible negative inotropic effect. Beta-blocker is not given as an option but would obviously be necessary in the future, particularly if echocardiogram confirms reduced LV EF; beta blockers are generally added once a patient is euvolemic.

A 55-year-old man collapses while playing tennis and is brought to the emergency department. The EMTs found him in cardiopulmonary arrest, performed CPR, applied a 200-joule shock to his chest, and inserted an endotracheal tube and IV. He has been bagged, and chest compressions continued on the way to the emergency department. At the time you see him, you get a rhythm strip that shows ventricular tachycardia. He has received epinephrine 1 mg IV push. Which of the following antiarrhythmics would be reasonable to administer now? A) Procainamide B) Bretylium tosylate C) Phenytoin D) Amiodarone E) Sodium bicarbonate

Answer: D. Amiodarone Explanation The most appropriate management of cardiac arrest induced by ventricular tachycardia (which is what the rhythm strip shows) is administering an initial 200-joule defibrillation. Additional shocks at higher energies, up to a maximum of 360 joules, should be attempted if the initial shock is ineffective in reverting the ventricular tachycardia. Current ACLS guidelines support the use of amiodarone as adjunctive therapy in patients refractory to cardioversion. If amiodarone is not available, lidocaine is an alternative. Remember, in patients with unstable arrhythmias (e.g., chest pain, SOB, hypotension, mental status changes), immediate cardioversion is most appropriate.

You see a 30-year-old man in the emergency department who has been experiencing palpitations for the past 2 hours. He has had similar symptoms in the past, but the symptoms have not lasted as long as this episode. He is otherwise healthy. He takes no medications, has never smoked, drinks alcohol only on weekends, and has never used illicit drugs. There is a family history of premature coronary artery disease. (Both grandfathers died in their 50s from MI.) On physical exam, he appears nervous but healthy. Vital signs: BP 110/68 mmHg; pulse 155 bpm; respirations 12; afebrile The exam is unremarkable except for tachycardia. PMI is nondisplaced, and estimated JVP is 4 mmHg. An ECG is performed: Wide complex tachycardia Conscious sedation with synchronized cardioversion is the best treatment if available quickly. If conscious sedation with synchronized cardioversion is not available, which of the following is the most appropriate next step in the management of this patient? A) Metoprolol intravenously B) Verapamil intravenously C) Adenosine intravenously D) Amiodarone intravenously

Answer: D. Amiodarone intravenously Explanation In patients with underlying structural heart disease/MI, wide complex tachycardia (WCT) is almost always (> 98%) ventricular tachycardia (VT). However, especially in younger patients and/or patients without known heart disease (such as the patient in our question), SVT with aberrancy and WPW/preexcitation must be considered. In patients with VT, though, drugs that treat SVT may cause hemodynamic instability and ventricular fibrillation. Additionally, in patients with preexcitation and atrial fibrillation presenting as wide complex tachycardia, AV nodal blocking agents may cause ventricular fibrillation (by removing the AV nodal protective effect of inhibiting AV conduction, all atrial impulses may travel down the "slick" aberrant pathway and lead to rapid ventricular tachycardia fibrillation). In unstable WCT patients, cardioversion is the treatment of choice. This patient, while anxious, is not unstable. With WCT patients in whom the diagnosis is uncertain, conscious sedation with synchronized cardioversion is recommended by most. If this is not available, or if the patient has refractory or recurrent WCT, then IV amiodarone is the drug of choice. Alternative options include procainamide or lidocaine. The correct answer here is amiodarone because we cannot say with certainty that this is SVT; it may, in fact, be VT from undiagnosed CAD.

A 22-year-old woman who is in the third trimester of an otherwise unremarkable pregnancy presented to her obstetrician with the sudden onset of severe intrascapular pain and moderate dyspnea. On physical examination, she was reported to be exceedingly uncomfortable and could not remain still. Her heart rate was 110 and blood pressure 165/110 mmHg in the left arm. The office nurse cannot obtain a blood pressure reading in the right arm. She was hospitalized and you are consulted. On your examination, she has no pulses in the right arm and her right carotid pulse is weaker than the left. She also has a soft diastolic decrescendo murmur heard best at the lower left sternal border. Basic laboratory data is normal. An electrocardiogram shows sinus tachycardia. Which of the following statements is true? A) This patient is likely to have a DeBakey class III or a Stanford Type B dissection. B) The drug of choice for this condition is intravenous nitroprusside alone. C) An aortic dissection in this location can usually be treated conservatively. D) An MRI of the chest is an appropriate diagnostic test for this condition. E) An echocardiogram is not necessary.

Answer: D. An MRI of the chest is an appropriate diagnostic test for this condition. Explanation This patient presents with acute aortic dissection, which has been described in the third trimester. Her physical examination makes it virtually certain that the aneurysm involves the ascending aorta and the arch, therefore making it a DeBakey Class II or Stanford Type A dissection. An MRI or CT scan of the chest should be diagnostic. An echocardiogram is indicated because of the evidence of aortic regurgitation on physical examination. Emergency surgery is recommended for type A aortic dissection, whereas type B dissection is generally treated medically unless complications occur (Cleveland Clinic Journal of Medicine October 2011 vol. 78 10 685-696). Immediate medical management of aortic dissection includes stabilizing the patient with prompt attention to blood pressure reduction. β-Blockers are the first drugs of choice because of their mechanism of lowering the rate of rise of ventricular force (dP/dt) and shear stress on the aorta (Circulation. 2010; 122: 184-188.). Intravenous agents are chosen for rapid onset, and esmolol (Brevibloc) or labetalol (Normodyne) are usually chosen (Circulation. 2010; 122: 184-188.). Other agents may be used, including nicardipine and nifedipine. Sodium nitroprusside (Nitropress) given as an i.v. infusion at 0.25-5.0 µg/kg/min may be added to beta-blocker or calcium-blocker therapy for rapid blood pressure control in appropriate patients. Nitroprusside decreases arterial pressure but actually increases the rate of rise of the pressure (increasing the "shear" forces), a feature which can be blunted with beta-blockers or calcium-channel blockers. But be aware that prolonged treatment with sodium nitroprusside is associated with an increased risk of fetal cyanide poisoning as nitroprusside is metabolized into thiocyanate and excreted into urine (Eur Heart J. 2011 Dec;32(24):3147-97.). The patient may require multiple antihypertensive medications to obtain control of BP and HR (Cleveland Clinic Journal of Medicine October 2011 vol. 78 10 685-696).

A 30-year-old woman presents with complaints of bleeding gums and easy bruisability for the past 7 months. She notes that her menses have been heavier than usual during the past few months. Which of the following is appropriate in the initial workup? A) To be cost effective, start with a coagulation factor deficiency workup. B) CBC with platelet count, PT, PTT, bleeding time, and Factors VIII and IX levels. C) CBC with platelet count, PT, PTT, antiplatelet antibody, and Factor VIII level. D) CBC with platelet count, PT, PTT, and bleeding time.

Answer: D. CBC with platelet count, PT, PTT, and bleeding time. Explanation Note that coagulation factor deficiencies are more likely to present with complications such as hemarthrosis. Platelet disorders usually present with mucosal bleeding and a history of bleeding with light trauma. Knowing what the PTT, PT, and platelet counts are will help guide the direction in which you need to proceed. If the PT alone is elevated, then you know you are dealing with a deficiency/inhibitor of Factor VII, vitamin K deficiency, or liver disease. If the PTT alone is elevated, then you could be dealing with deficiency/inhibitors of Factors VIII, IX, or XI, or inhibitor of XII (deficiency of Factor XII does not cause a bleeding diathesis), von Willebrand disease, or lupus anticoagulant. Abnormal bleeding time would indicate von Willebrand disease or a platelet abnormality.

A 45-year-old man being evaluated for ischemic heart disease has had chest pain with exertion for about 5 months and finally came to see you this week for evaluation. He says the pain occurs only with exertion and is a pressure-like feeling in his chest. The pain does not radiate, and he has no sweating or shortness of breath with the pain. The pain mainly occurs when he is running on his treadmill, although lately he has had chest pain occur while having sexual intercourse. Pain subsides after resting for a few minutes. He does not have any chest pain except with exertion and has never had pain at rest. PAST MEDICAL HISTORY: Hypertension for 4 years treated with fosinopril 20 mg q day Hernia repair 10 years ago SOCIAL HISTORY: Lives with his wife of 2 years; his 3rd marriage Works as a librarian by day, bartender by night No children Doesn't smoke or drink FAMILY HISTORY: Father 70; S/P CABG at age 60 Mother 70; Alzheimer disease Brother 50; angioplasty a year ago for left main coronary artery disease REVIEW OF SYSTEMS: Negative for any other symptoms PHYSICAL EXAMINATION: Ht 5'10", Wt 180, BP 120/70, P 85, RR 14, Temp 98.8° F HEENT: PERRLA, EOMI TMs clear Throat clear Neck: Supple; no thyromegaly Heart: RRR without murmurs, rubs, or gallops Lungs: CTA Abdomen: Bowel sounds present; no masses; no hepatosplenomegaly Extremities: No cyanosis, clubbing, or edema Rectal: Heme-negative; no masses LABORATORY: Normal for age You decide to perform an exercise stress test on him. Which of the following should make you stop the test urgently? A) Sinus tachycardia of 130 bpm B) Mild cramping in his left calf C) Respiratory rate of 30 D) Decrease in systolic BP > 15 mmHg E) ST-segment depression of 1.5 mm

Answer: D. Decrease in systolic BP > 15 mmHg Explanation Other things that should make you stop the test quickly are: ST-segment depression greater than 2 mm, development of ventricular tachycardias, development of severe chest pain, shortness of breath, or lightheadedness. His target heart rate is ~ 148 bpm because that represents 85% of his maximum predicted heart rate of 175 bpm (220 - age). Remember that a positive test is indicated by finding ST depression of > 1 mm at the J + 80 millisecond point (i.e., at 80 milliseconds after the J point) in 2 contiguous leads. His prior probability of heart disease is very high based on his symptoms and family history, so a positive test in this patient is likely a true positive.

A 35-year-old Caucasian male living in northern Louisiana has been having sharp, pleuritic chest pains that are relieved by sitting upright. He has reported some fever for several days and a nonproductive cough. PAST MEDICAL HISTORY: Essentially negative; came in 3 years ago for URI SOCIAL HISTORY: Works as a policeman Doesn't smoke; drinks 2 beers on the weekends when he plays poker with his buddies FAMILY HISTORY: Mother with depression Wife is manic-depressive and often steals things from local department stores REVIEW OF SYSTEMS: No weight loss No chills No nausea or vomiting PHYSICAL EXAMINATION: Ht 6'1", Wt 190 lbs (very muscular), BP 120/70, RR 10 and splinting, P 110, T 100.5° F HEENT: PERRLA, EOMI TMs clear Throat clear Neck: Supple; no meningismus Heart: RRR without murmurs and gallops Lungs: CTA Abdomen: +BS, soft, no organomegaly Extremities: No cyanosis, clubbing, or edema Skin: No rashes Which of the following findings would least support a diagnosis of acute pericarditis? A) PR-segment depression, especially in lead II. B) A pericardial rub is not present. C) Serum creatine phosphokinase concentrations of 2x normal. D) Diffuse T wave inversion with ST-segment elevation. E) Frequent atrial premature beats.

Answer: D. Diffuse T wave inversion with ST-segment elevation. Explanation Another way to think of this question: Which of these can or is likely to occur in pericarditis? Then pick the one left that does not fit. Look for what does do it and then see what is left as the "except." Many of you probably went for the "no rub" answer. However, remember that a rub is actually not that common. So, if we read the question as, "Which of the following findings can occur in acute pericarditis?" then the finding of "no rub" is certainly a possibility. Now, if the rub is there, it is diagnostic; if not, then it really doesn't help you rule out the diagnosis. With a pericarditis, you would expect to see frequent atrial premature beats. Also, PR-segment depression, especially in lead II, would help you here. CPKs frequently will rise with pericarditis and in fact can be 2-3 times normal; but they are transiently elevated. The finding of diffuse T wave inversion with ST-segment elevation more likely means the poor guy had an MI, not pericarditis. These ECG findings would least support the diagnosis of acute pericarditis.

T.L., a 17-year-old female, presents to clinic for her routine sports physical. She is finishing her junior year in high school and has been receiving offers from several colleges for soccer. This is your first time to see her. She has been doing well with no real concerns, but the mother does mention that T.L. gets dizzy and lightheaded sometimes when playing soccer. It usually occurs after playing for 30 minutes or longer. She does not have chest pain with exercise but sometimes feels nauseated. She has never passed out. There is no family history of sudden death. PMH: Significant for migraines at age 6 but no longer has problems with that Immunizations: UTD Social: Lives in the city with her parents and 2 siblings; no pets; no smoking Physical Exam: Vitals: Weight 136 pounds (70th percentile), height 63.5 inches (45th percentile) BP 104/62; Pulse 52 HEENT: Within normal limits Heart: Regular rate and rhythm, no murmur, normal pulses in all limbs Lungs: CTA in all lung fields, no wheezing Abdomen: Non-tender, no masses, no HSM Extremities: Strong pulses, no edema GU: Normal female, Tanner 5 Skin: Warm, dry, no rash Lab: Hemoglobin 15.6 What would your next course of action be? A) Tell the patient she is not drinking enough and have her drink Gatorade 30 minutes prior to and during the game. B) Have the patient take iron supplements and recheck in 2 months. C) Reassurance to mother that the patient is just fine. D) ECG. E) EEG.

Answer: D. ECG. Explanation Always be suspicious if a patient has symptoms with exercise. This patient's ECG showed mild intraventricular conduction delay and an Rsr pattern in V1. There is also marked Q-T prolongation—corrected Q-T interval is 0.56. ST segments are normal, and there are no pathologic Q waves. Echocardiogram showed intraventricular septal hypertrophy that bulges into the left ventricular outflow tract. This patient had both hypertrophic cardiomyopathy as well as prolonged Q-T interval. She was started on a beta-blocker and, of course, was no longer able to participate in competitive sports.

A 73-year-old man with prosthetic aortic valve presents for primary care. He has been well controlled on warfarin 3 mg q hs for 6 years with INR 2.5-3.0. He is concerned about his prostate, memory problems, and heart disease, and would like to take supplements recommended by his niece who works at a health food store. Which of the following would you specifically recommend against? A) Cat's claw B) Folate C) Saw palmetto D) Gingko biloba

Answer: D. Gingko biloba Explanation Gingko biloba can increase hemorrhage risk in patients who take it. It can further increase the risk in anticoagulated patients. Another agent to avoid is St. John's wort, which may increase the metabolism of warfarin and, if a patient isn't monitored closely, could lead to under-anticoagulation. Folate, cat's claw, and saw palmetto do not affect warfarin metabolism.

A 56-year-old man comes to the emergency department complaining of six hours of constant dull substernal chest pain. He has had chest pain intermittently for the past year but has not sought medical attention. He has an episode of pain 2-3 times per week, occurring at rest and with exercise, and sometimes at night. The pain lasts from 20 minutes to several hours. He regularly cares for his three horses, lifting 100-pound bales of hay and raking out their stalls, and this activity is infrequently associated with chest pain. His cardiac risk factors include age, male sex, and a strong family history of early coronary disease. In fact, he sought medical attention on this occasion because his 43-year-old brother had a heart attack the previous week. His physical examination is unremarkable. Initial cardiac markers are negative, and his electrocardiogram is normal. Which of the following statements is true? A) He should be given a prescription for a beta-blocker and sublingual nitroglycerin and a follow-up appointment with his primary care provider should be arranged. B) He should have early cardiac catheterization and probably coronary intervention. C) He should be admitted to the hospital and started on aspirin, clopidogrel, and a low-molecular weight heparin. D) He should be observed for at least 8-12 hours after symptom onset and additional cardiac markers and electrocardiogram should be obtained. E) He has a noncardiac cause of chest pain; he should be discharged from the emergency department and a follow-up appointment with his primary care provider should be arranged.

Answer: D. He should be observed for at least 8-12 hours after symptom onset and additional cardiac markers and electrocardiogram should be obtained. Explanation 2012 ACCF/AHA guidelines suggest that patients with negative cardiac biomarkers within 6 hours of the onset of symptoms consistent with acute coronary syndrome should have biomarkers remeasured in the time frame of 8 to 12 hours after symptom onset. Also, an electrocardiogram should be repeated. If there is no recurrence of symptoms and the aforementioned study is negative, the patient should have a stress study. There is no indication for coronary angiography at this time.

A 72-year-old man presents with 3 hours of chest pain radiating down his left arm. He also has moderate dyspnea. He has a history of known coronary artery disease with coronary bypass graft surgery 5 years ago. His physical examination shows a heart rate of 85, a blood pressure of 145/95 mmHg, and moderate elevation of the jugular venous pressure. His first troponin I level is 13.6 ng/mL. An electrocardiogram shows ST segment depression in the lateral leads. Which of the following statements is true? A) This patient is at high risk and should receive immediate fibrinolytic therapy. B) Beta-blockers should be avoided because of his heart failure. C) A stress study should be performed the next day to determine if coronary angiography is necessary. D) He should receive aspirin, clopidogrel/ticagrelor/IV GP IIb/IIIa inhibitor, intravenous unfractionated heparin/low-molecular-weight heparin (or some other type of antithrombin therapy). E) Clopidogrel/ticagrelor should be continued for only 1 week after discharge.

Answer: D. He should receive aspirin, clopidogrel/ticagrelor/IV GP IIb/IIIa inhibitor, intravenous unfractionated heparin/low-molecular-weight heparin (or some other type of antithrombin therapy). Explanation This patient presents with a new non-elevated ST segment myocardial infarction (NSTEMI), a history of previous coronary bypass surgery, and evidence of heart failure. This is a high-risk situation, and he needs aggressive medical therapy and early coronary angiography. According to current ACC/AHA guidelines, he should have a combination of aspirin, clopidogrel/ticagrelor/IV GP IIb/IIIa inhibitor, and a low-molecular-weight heparin or other antithrombin therapy. A beta-blocker improves outcomes most in patients with mild to moderate heart failure associated with an acute infarction. Per the 2012 ACCF/AHA Focused Update Incorporated Into the ACCF/AHA 2007 Guidelines for the Management of Patients With Unstable Angina/Non-ST-Elevation Myocardial Infarction: A Report of the American College of Cardiology Foundation/American Heart Association Task Force on Practice Guidelines. 2007 WRITING COMMITTEE MEMBERS; Presidents and Staff; American College of Cardiology Foundation; American College of Cardiology Foundation/American Heart Association; American Heart Association. Circulation. 2013 Jun 11;127(23):e663-828. doi: 10.1161/CIR.0b013e31828478ac. Epub 2013 Apr 29. No abstract available. PMID: 23630129 [PubMed - in process] Patients with definite UA/NSTEMI at medium or high risk and in whom an initial invasive strategy is selected should receive dual antiplatelet therapy on presentation. Aspirin should be initiated on presentation. The choice of a second antiplatelet therapy to be added to aspirin on presentation includes 1 of the following (note that there are no data for therapy with 2 concurrent P2Y12 receptor inhibitors, and this is not recommended in the case of aspirin allergy): Before PCI: ● Clopidogrel; or ● Ticagrelor; or ● An IV GP IIb/IIIa inhibitor. IV eptifibatide and tirofiban are the preferred GP IIb/IIIa inhibitors. At the time of PCI: ● Clopidogrel if not started before PCI; or ● Prasugrel; or ● Ticagrelor; or ● An IV GP IIb/IIIa inhibitor. The duration and maintenance dose of P2Y12 receptor inhibitor therapy should be as follows: a. In UA/NSTEMI patients undergoing PCI, either clopidogrel 75 mg daily,prasugrel 10 mg daily,or ticagrelor 90 mg twice daily should be given for at least 12 months. b. If the risk of morbidity because of bleeding outweighs the anticipated benefits afforded by P2Y12 receptor inhibitor therapy, earlier discontinuation should be considered.

A 74-year-old lethargic man is brought to the emergency department complaining of nausea, vomiting, and abdominal pain. He has been having vague muscle and joint pains for several days prior to admission. On exam, he has hyporeflexia, and you notice that he has hypotonic weak muscles. An electrocardiogram shows a shortened QT interval. What is the most likely cause of his lethargy? A) Hypokalemia B) Hypocalcemia C) Carbon monoxide poisoning D) Hypercalcemia

Answer: D. Hypercalcemia Explanation Hypercalcemia is most commonly caused by hyperparathyroidism or malignancy (usually breast, lung cancer, or multiple myeloma), but can also be caused by sarcoidosis and other granulomatous diseases. When it is caused by hyperparathyroidism, it is usually a mild hypercalcemia; it is also known as primary hyperparathyroidism. This occurs more often in women, and its incidence increases with age. The increase in calcium is due to the hypersecretion of parathyroid hormone (PTH), which causes an increase in the absorption of calcium in the intestine as well as decreased excretion of calcium at the renal tubule. The osteoclasts in the bones are not involved in this process. When caused by cancer, the hypercalcemia is usually more severe. It is due to increased osteoclastic activity within the bone (non-PTH mediated). This affects men and women equally, and increases in incidence with increasing age (as does the occurrence of these tumors). Hypercalcemia affects nearly every organ system in the body but particularly the central nervous system and kidneys. Neurologic symptoms begin with fatigue, depression, weakness, and confusion and can progress to hallucination, disorientation, hypotonicity, seizures, and coma. Renal effects include nephrolithiasis; calcium deposition can lead to nephrogenic diabetes insipidus as well as acute kidney injury. Patients might also experience gastrointestinal manifestations, such as anorexia, nausea, vomiting, and constipation, as well as nonspecific musculoskeletal tenderness. The high levels of calcium ions decrease neuronal excitability, which leads to hypotonicity of smooth and striated muscle. This explains the fatigue, muscle weakness, low tone, and sluggish reflexes in muscle groups. Therefore, neuromuscular symptoms include weakness and diminished deep-tendon reflexes. However, cases of hyperreflexia and tongue fasciculations have been noted. Although, it appears reduced or depressed deep-tendon reflexes are more commonly manifested by hypercalcemic patients. Cardiovascular signs and symptoms of hypercalcemia vary. Calcium has a positive inotropic effect until levels reach more than 15 mg/dL, at which time myocardial depression ensues. The QT interval typically shortens, and the PR and QRS intervals are prolonged when the serum calcium concentration is more than 13 mg/dL. The treatment is directed to rapidly correct the hypercalcemia. The prognosis depends on the underlying cause. If a malignancy, the prognosis is usually poor.

A 25-year-old male presents after having a syncopal episode while playing basketball this morning. He has no other symptoms and says that he feels fine now. Physical Examination: HEENT: Normal Neck: Carotid pulse had a rapid upstroke and is bifid Heart: RRR with harsh, nonradiating midsystolic aortic murmur Lungs: Normal Abdomen: Benign GU: Normal male genitalia Skin: No rash On further testing, he performs a Valsalva maneuver, and his murmur increases in intensity. Which of the following is the most likely diagnosis? A) Mitral valve prolapse B) Abnormal coronary artery C) Coronary artery disease D) Hypertrophic cardiomyopathy E) Aortic stenosis

Answer: D. Hypertrophic cardiomyopathy Explanation This patient has hypertrophic cardiomyopathy (HCM), which is the most common cause of sudden death in exercising young people. Abnormal coronary arteries are the next most common disorder, but they do not demonstrate the abnormalities on physical exam as HCM. No medications have been shown to prolong survival—only heart transplant. Beta-blockers and verapamil will improve symptoms, however. The carotid pulse finding in this disorder is also seen in aortic stenosis, but the history and murmur are characteristic of HCM.

What are the 4 components of tetralogy of Fallot? A) ASD, right ventricular outflow tract obstruction, overriding aorta, RVH B) ASD, VSD, patent ductus arteriosus, LVH C) Large VSD, left ventricular outflow tract obstruction, overriding aorta, LVH D) Large VSD, right ventricular outflow tract obstruction, overriding aorta, RVH E) Large VSD, right ventricular outflow tract obstruction, overriding aorta, LVH

Answer: D. Large VSD, right ventricular outflow tract obstruction, overriding aorta, RVH Explanation This is just a simple memorization thing. It likely has appeared on past Board exams, so just memorize it. A mnemonic that helps me (but may not help you) is: "V-ROAR-V," where V stands for VSD, RO stands for right ventricular outflow obstruction, OA for overriding aorta, and RV for RVH. Like I said, it may or may not help you—my brain is sometimes weird in how it works.

A 38-year-old woman with recurrent hematuria is scheduled for cystoscopy. She is otherwise healthy, has never had surgery, takes no medications, does not smoke cigarettes, does not drink alcohol, and does not use IV drugs. Review of systems is negative. Physical exam is normal, except for a sharp, short sound heard in mid-systole followed by a grade II/VI murmur audible at the apex. What antibiotic prophylaxis for infective endocarditis should be given to this patient? A) One dose of oral amoxicillin prior to the procedure B) Intravenous ceftriaxone C) Intravenous ampicillin plus gentamicin perioperatively D) None of the choices

Answer: D. None of the choices Explanation The most recent updates on prophylaxis for endocarditis are from ACC/AHA 2008 Focused Update on Infective Endocarditis: SBE prophylaxis for dental procedures: Prosthetic cardiac valve Previous history of endocarditis Congenital heart disease: Unrepaired cyanotic Completely repaired with prosthetic material or device for ≤ 6 months postprocedure Cardiac transplant recipients who develop cardiac valvulopathy This woman likely has mitral valve prolapse with a murmur. Note that all of the valvular abnormalities have been taken out for needing antibiotic prophylaxis. For adults, the only reason you'll need to give antibiotic prophylaxis is for those with prosthetic valves or who have a history of endocarditis. Summary of major changes in updated guidelines: Limit SBE prophylaxis to only those conditions listed above. Antibiotic prophylaxis is no longer recommended for any other form of congenital heart disease. Antibiotic prophylaxis for procedures on respiratory tract or infected skin, etc. only for conditions listed above. No antibiotic prophylaxis for GU or GI procedures (regardless of cardiac valve abnormality)!

You see a 65-year-old female patient who is new to your practice. She tells you that her old doctor mentioned "something about aortic stenosis," but she did not fully understand and didn't ask any questions. Now she is concerned, although she reports no chest pain, weakness, or palpitations. Her exam is normal. Which of the following is most consistent with the diagnosis of aortic stenosis? A) A systolic thrill, if it occurs, is located in the right axilla. B) It is not a common congenital cardiac anomaly. C) Sudden death occurs in < 1% of patients, almost all of whom are symptomatic. D) Once symptoms develop with aortic stenosis, prognosis is poor. E) Patients with bicuspid aortic valves develop aortic stenosis 2 decades later than patients with trileaflet valves.

Answer: D. Once symptoms develop with aortic stenosis, prognosis is poor. Explanation A bicuspid aortic valve is very common, occurring in 1/50. AS develops 2 decades earlier in patients with bicuspid valves than those with tricuspid valves. Sudden death occurs in 10% of patients, almost all of whom are symptomatic. A systolic thrill, if it occurs, is located in the suprasternal notch or carotids.

What QT interval abnormality do you expect in these individuals? A) Can be either prolonged or short QT. B) Short QT. C) QT cannot be determined in these individuals. D) Prolonged QT. E) No change in the QT.

Answer: D. Prolonged QT. Explanation No Explanation provided.

You see a 49-year-old man in the emergency department for 30 minutes of crushing substernal chest pain. He has poorly controlled hypertension and a family history of premature coronary artery disease. He takes only HCTZ 25 mg daily and has no other medical problems. Vital signs are as follows: BP: 158/92 mmHg, pulse: 106, resp: 18, afebrile His exam is normal, with no evidence of congestive heart failure or murmurs. ECG shows 2- to 3-mm ST-segment depressions in leads I and aVL and leads V2 through V5. Initial CK is normal, but cardiac troponins are 3x normal. Cardiology is contacted and plans to take him to the cardiac catheterization lab within the next hour for delineation of his coronary anatomy and probable percutaneous coronary intervention (PCI). In addition to immediate aspirin and bivalirudin therapy, which of the following is the best treatment option prior to sending the patient for angiography/PCI? A) Immediate release nifedipine B) No other treatment prior to coronary angiography C) Thrombolytic therapy D) Ticagrelor (P2Y12 receptor blocker)

Answer: D. Ticagrelor (P2Y12 receptor blocker) Explanation In patients with non-ST elevation MI, thrombolytic therapy is not beneficial and should be avoided. Ticagrelor is the current P2Y12 receptor blocker recommended over clopidogrel, but clopidogrel is still acceptable in some centers. Most do not use prasugrel in this situation because of the risk of increased bleeding. Note that bivalirudin is the recommended anticoagulant therapy instead of heparin based on current studies. Immediate release nifedipine has no role in acute coronary syndromes and may be harmful because of reflex tachycardia and hypotension. GP IIb/IIIa inhibitors are not used as much anymore in patients with NSTEMIs who are going to the cath lab soon—especially if the patient is also on bivalirudin because of the increased risk of bleeding. If heparin is used instead of bivalirudin, and the patient has hemodynamic instability, markedly elevated troponins, or recurrent ischemia, then most add a GP IIb/IIIa inhibitor. So current bottom line (late 2012), before PCI in a NSTEMI: Use aspirin, bivalirudin, and ticagrelor as your antiplatelet and anticoagulation.

You are a physician moonlighting in a small town hospital. A severely ill man shows up at the front door. You determine that he has acute mitral regurgitation from rupture of a myxomatous chordae. Which of the following should you do as soon as possible? A) Change the antibiotic coverage to the broadest possible spectrum and add positive end-expiratory pressure to his ventilator settings. B) Transfer him emergently for aortic valve replacement. C) Transfer him via emergency air ambulance for urgent coronary angiography and probable coronary bypass grafting. D) Transfer him emergently for mitral valve repair or replacement. E) Start him on low-dose intravenous dopamine and, cautiously, on intravenous nitroprusside.

Answer: D. Transfer him emergently for mitral valve repair or replacement. Explanation This man needs urgent cardiothoracic surgical intervention for the mitral valve. His mortality risk without immediate surgery is obviously very high. The other choices either are for incorrect diagnoses or delayed mitral valve surgery.

A 45-year-old woman comes to your emergency department with a complaint of 2 hours of severe anterior chest pain. She has no history of cardiac disease. She describes a sharp, stabbing pain that radiates to her left arm and neck, worsened somewhat by respiration. She also has moderate dyspnea. Her cardiac risk factors include current smoking and hypertension. On physical examination, her heart rate is 95 and blood pressure 165/110 mmHg. The remainder of the examination is normal. Her laboratory data is all normal except a white blood count of 11.6 x 109/L. Cardiac markers are negative. A chest x-ray is normal. Her electrocardiogram follows: diffuse ST segment elevation Which of the following statements is true? A) This patient should be transferred immediately to a tertiary care center for percutaneous coronary intervention. B) An echocardiogram is likely to show a segmental wall motion abnormality of the left ventricle. C) Abdominojugular reflux is usually present in this condition. D) Treatment with a nonsteroidal antiinflammatory agent is indicated. E) Fibrinolytic therapy with intravenous tissue plasminogen activator is indicated.

Answer: D. Treatment with a nonsteroidal antiinflammatory agent is indicated. Explanation This history and electrocardiogram of diffuse concave-up ST-segment elevation are consistent with acute inflammatory pericarditis. Physical examination frequently reveals a pericardial friction rub, but this finding is notoriously evanescent. Coronary intervention is unnecessary and fibrinolytic therapy is contraindicated, since the latter may produce hemorrhagic pericarditis and tamponade. An echocardiogram may show a small pericardial effusion. Abdominojugular reflux is a rather nonspecific finding of right heart dysfunction. Treatment of acute pericarditis is with nonsteroidal agents. Patients who respond slowly or inadequately to nonsteroidal agents may require a course of colchicine. In unresponsive cases, a short course of corticosteroids might be considered.

A 70-year-old male, status post AVR replacement 2 years ago for aortic stenosis, presents with widespread ecchymosis on his back and legs and some bruising on the back of both hands. His last INR was 3 weeks ago and was 3. He states he saw a doctor 6 days ago for a cough and was put on a medication described as a "white tablet." His chronic medications include: warfarin 5 mg qd, albuterol inhaler 2 puffs 4 times a day, and nortriptyline 25 mg q hs. Which of the following medications was he placed on? A) Cefixime B) Codeine C) Amoxicillin D) Trimethoprim/sulfamethoxazole

Answer: D. Trimethoprim/sulfamethoxazole Explanation The combination of warfarin and TMP/SMX can be a very deadly one. It is the most common warfarin interaction leading to hospitalization in the U.S. Amoxicillin, codeine, and cefixime do not interact with warfarin. Azithromycin only rarely causes an increase in INR in patients on warfarin

A 63-year-old, retired, second grade English teacher presents to the emergency department with the local EMS after she suffered 40 minutes of crushing chest pain at home. You note her ashen skin color with moist clammy features and a BP of 80/60 mmHg with a heart rate of 122/min. Her systemic venous pressure appears to be elevated, and there is no murmur. Her ECG reveals 6-8 mm ST-segment elevation in leads I, aVL, and V1-V6 with hyperacute T-wave changes in the same leads and developing deep Q waves. Without an apparent mechanical complication of her acute event, you know that her prognosis may be improved by which of the following? A) Surgical resection of the myocardial infarction segment B) Chronic subcutaneous home infusion of dobutamine C) Immediate high-dose adrenolytic therapy (e.g., propranolol 80 mg every 6 hr) D) Urgent revascularization E) Acute mitral valvuloplasty

Answer: D. Urgent revascularization Explanation The prognosis for the patient with cardiogenic shock in the setting of myocardial infarction is limited and is a function of the amount of lost myocardium. The threshold value is usually stated to be 40%; losses above that amount generally are those that account for shock. The patient with acute papillary muscle disruption or perforation of the interventricular septum represents special, uniquely surgical challenges. The patient described here used to be seen as nearly hopeless until the observation that some may improve dramatically if given a successful catheter revascularization approach. Getting her to the cath lab as quickly as possible is the best chance for recovering/maintaining viable myocardium. If the cath lab is not available (e.g., rural community), then fibrinolytics are indicated.

Which of the following patients is likely to benefit the most from thrombolytic therapy? A) A 60-year-old woman with a recent onset of mild right hemiparesis and 2 hours of chest pain with an ECG showing no Q waves and 4 mm of ST-segment elevation from leads V1 to V6, and 2 mm of ST-segment elevation in leads I and aVF B) A 68-year-old woman with the onset of chest pain 2 hours ago with tall R waves and ST-segment depression in leads V1 and V2 C) A 60-year-old man with two previous coronary bypass surgeries and 2 hours of chest pain who has 4 mm of ST-segment depression and new T wave inversion in leads V1 to V4 D) A 40-year-old man with the onset of chest pain 4 hours ago who has small Q waves and 3 mm of ST-segment depression in leads II, III, and aVF E) A 50-year-old man with the onset of chest pain 2 hours ago and a new left bundle-branch block

Answer: E. A 50-year-old man with the onset of chest pain 2 hours ago and a new left bundle-branch block Explanation Patients with new left bundle-branch block (LBBB) in the clinical setting of an acute ischemic event have the most mortality benefit from thrombolytic therapy because of the amount of myocardium in jeopardy. Patients with anterior ST-segment elevation benefit (but less than a new LBBB). Patients with inferior ST-segment elevation benefit much less and are actually harmed if they have ST-segment depression. ST-segment depression results in increased mortality if thrombolytics are given! The 60-year-old woman has a strong indication for thrombolytic therapy except for the recent onset of a neurological deficit, which is an absolute contraindication to thrombolytics. The 68-year-old woman with a tall R wave in leads V1 and V2 is having an acute posterior MI and would benefit from thrombolytic therapy, but the man with the LBBB has had more myocardium become jeopardized and would benefit more from thrombolytics.

A 17-year-old Asian student is playing basketball when he collapses and becomes unresponsive. An initial examination by the trainer, who was at courtside, reveals that he is not breathing and does not have a palpable pulse. The school has a defibrillator at hand for all games, and 300 joules of energy are delivered to the student's chest. He recovers quickly and is immediately brought to your hospital and admitted to your coronary care unit. By the time you examine him, he is alert and his vital signs are stable. He has no previous history of heart disease or any other medical problem, and he denies illegal drug use. There is no history of sudden death in younger members of his family. His physical examination is entirely normal. Basic laboratory data show a normal complete blood count and chemistry panel. A chest x-ray shows moderate cardiomegaly. An electrocardiogram shows sinus rhythm with very large R waves in I, aVL, V5, and V6. Which of the following statements is true? A) The most common cause of sudden death in this condition is associated with anomalous origin of a coronary artery. B) Amiodarone has been shown to prolong survival in these patients. C) An echocardiogram will show asymmetric septal hypertrophy and a pressure gradient across the aortic outflow tract. D) Standing will produce a systolic murmur. E) Atrial fibrillation is a frequent complication of this condition.

Answer: E. Atrial fibrillation is a frequent complication of this condition. Explanation This patient has the apical form of hypertrophic cardiomyopathy, a form well documented in Asians. Apical hypertrophic cardiomyopathy is a rare form of hypertrophic cardiomyopathy which usually involves the apex of the left ventricle and rarely involves the right ventricular apex or both. Because apical hypertrophic cardiomyopathy involves hypertrophy—which usually involves the apex of the left ventricle and rarely involves the right ventricular apex or both—the septum and remaining walls of the left ventricle are not abnormally thickened. Therefore, there is no asymmetrical septal hypertrophy (as is seen in the more common forms of hypertrophic cardiomyopathy), so there is no abnormal anterior septal motion of the anterior mitral leaflet into the left ventricular outflow tract during systole. Thus, an echocardiogram will not show asymmetric septal hypertrophy and a pressure gradient across the aortic outflow tract. Rather, an echocardiogram will show hypertrophy of the left ventricular apex giving it a "spade-like" appearance. Hypertrophic cardiomyopathy is the most common abnormality found at the time of autopsy in a young person who dies during activity. It is usually inherited as an autosomal dominant, but new mutations are not uncommon. The apical form is not associated with obstruction and usually does not produce a murmur with Valsalva or standing. The electrocardiogram frequently shows prominent voltage in the anterior or lateral leads. Echocardiography is diagnostic. No medication has been shown to prolong survival in hypertrophic cardiomyopathy. Atrial fibrillation is a common problem because of high left ventricular end diastolic pressure. Anomalous origin of a coronary artery is the 2nd most common cause of death in exercising young people. Apical hypertrophic cardiomyopathy may manifest as morbid events such as atrial fibrillation, myocardial infarction, embolic events, ventricular fibrillation (which may lead to sudden cardiac death) and congestive heart failure.

A 17-year-old woman with diagnosis of coarctation of the aorta by a local cardiologist is following up with you for further treatment. She has not had menses yet. Otherwise, she is healthy and did not have any problems until her family noted that she was not growing properly. PAST MEDICAL HISTORY: Negative SOCIAL HISTORY: Lives with mom and dad in an apartment A senior in high school Denies smoking or drinking FAMILY HISTORY: Grandfather with hypertension Grandmother with hypothyroidism REVIEW OF SYSTEMS: No menses PHYSICAL EXAMINATION: BP 110/70 upper extremities; 80/50 lower extremities; P 90; RR 18; Temp 98.6° F HEENT: PERRLA, EOMI TMs clear; low-set ears Low posterior hairline High-arched palate Neck: Marked webbing of the neck Heart: RRR without murmurs, rubs, or gallops Lungs: CTA Abdomen: Bowel sounds present, no hepatosplenomegaly Extremities: Short digits; no cyanosis or clubbing; hands look edematous GU: Minimal breast development Normal external genitalia No secondary hair development Which of the following other cardiac abnormalities is associated with the presence of her coarctation of the aorta? A) Sinus venosus B) Mitral stenosis C) Ventricular septal defect D) Patent ductus arteriosus E) Bicuspid aortic valve

Answer: E. Bicuspid aortic valve Explanation 70% of people with coarctation will also have a bicuspid aortic valve. Usually, signs such as murmurs don't occur until later in life in the mid-30s. None of the other conditions are commonly associated with coarctation. She does have Turner syndrome. Note also that aneurysms of the circle of Willis occur with coarctation.

A 45-year-old man with diabetes and severe knee arthritis is referred to you by his rheumatologist. He has chest pain with typical and atypical features for angina. Sometimes the pain is a "pressure" in his mid-chest, but mostly he describes sharp stabbing pain at the right upper sternal border. He denies any lung disease. He cannot walk very far due to his arthritis. PAST MEDICAL HISTORY: Severe arthritis; takes ibuprofen 800 mg 3 times daily; occasionally has to take narcotic agents to relieve the pain Diabetes for 10 years—takes insulin 30 U NPH in a.m. SOCIAL HISTORY: Lives alone in an apartment with modern conveniences Smokes ½ pack of cigarettes daily Doesn't drink alcohol FAMILY HISTORY: Father 70 with HTN Mother died at age 72 of stroke Brother 42 with diabetes REVIEW OF SYSTEMS: No radiation of pain No shortness of breath with pain No headaches No dyspnea on exertion—but can't walk so difficult to assess PHYSICAL EXAMINATION: BP 120/80, RR 20, Temp 98.7° F, P 88 HEENT: PERRLA, EOMI TMs clear Throat clear; poor dentition Neck: Supple Heart: RRR without murmurs, rubs, or gallops Lungs: CTA Abdomen: Benign Extremities: Severe osteoarthritis of the knees Which of the following is the most appropriate next step to evaluate this patient? A) Treat the patient for an ulcer because of all the nonsteroidals he is taking. B) Electrophysiologic study. C) Proceed to left heart catheterization. D) Routine stress test. E) Dobutamine stress echo.

Answer: E. Dobutamine stress echo. Explanation This patient is diabetic and has some symptoms suggestive of coronary artery disease. He cannot perform a routine stress test due to his arthritis. An electrophysiologic study is not going to tell us anything. This could be an ulcer, but the pressing issue is whether or not he has coronary artery disease. Proceeding to left heart catheterization is invasive and not necessary at this point. Pharmacological type of stress test (dobutamine stress echo or adenosine/regadenoson nuclear stress test) should be helpful in determining if he has significant coronary disease—if so, then a left heart cath would be indicated

35-year-old female comes with several family members to your clinic to discuss results from a recent out-of-area hospitalization. She was visiting family at that time and was prompted to visit the area's emergency department when she experienced chest pain. She was admitted at that time for evaluation. During her workup, cardiac enzymes were normal; exercise tolerance testing demonstrated no abnormalities, and an echocardiogram was remarkable only for mitral valve prolapse (MVP) without regurgitation or valvular thickening. She was discharged on a beta-blocker. The patient now wants your opinion as to the long-term prognosis of her condition. Which of the following would you tell her about her MVP? A) Left ventricular failure is a common consequence. B) She should receive antibiotics before dental procedures. C) It increases her risk for sudden death. D) It increases her risk of systemic emboli. E) Her current condition is benign.

Answer: E. Her current condition is benign. Explanation Prolapse of the mitral valve is a relatively common finding that is associated with several conditions and can vary significantly in severity. Significant mitral regurgitation can result in left ventricular enlargement, but hypertrophy of the left ventricle and sudden death are much more commonly associated with aortic stenosis. MVP with normal leaflets and no regurgitation is most commonly associated with a benign course and may respond symptomatically to beta-blocker therapy. It is not an indication for prophylactic antibiotics. Note the ECG changes in II, III, and aVF. Septal MI = changes in V1-V2 Anterior MI = changes in V3-V4 Anteroseptal MI = V1-V4 Lateral MI = I, aVL, V6 Anterolateral = I, aVL, V3-V6 (extensive will involve V1-V6!) Inferior = II, III, aVF Apical = II, III, aVL and any of V1-V4 Posterior = Tall R in V1-V2 High lateral = I, aVL Criteria for right bundle-branch block (RBBB) include: QRS > 120 ms (3 small squares) Depolarization of the right ventricle is delayed resulting in an RSR' ("rabbit ears") or RR' in V1 and often a slurred S wave in V5-V6. Flipped T waves in V1, sometimes V2.

A 28-year-old internal medicine resident presents with complaint of chest pain. He describes the pain as sharp, stabbing, intermittent, and radiating to his left chest. On occasion, he also has some dysphagia. SOCIAL HISTORY: Currently on cardiology rotation; under lots of stress Recently separated from his wife Mother died last month FAMILY HISTORY: Mother with MI last month at age of 70 Father with MI 2 years ago at age 68 Brother healthy Sister healthy PHYSICAL EXAMINATION: HEENT: Normal Neck: Supple, no masses Heart: RRR with I/VI flow murmur Lungs: Clear to auscultation Abdomen: + BS, benign Extremities: No rashes He went to his fellow residents on various rotations and got the following tests done: Stress test: Normal EGD: Normal Esophageal motility: Mostly normal peristalsis, but with 37% simultaneous contractions On further questioning, he tells you his dysphagia is for solids and liquids, but especially cold liquids. Based on his personal history, family history, physical findings, and laboratory studies, which of the following is true? A) Botox therapy is recommended. B) Atypical angina is a likely diagnosis. C) He has Plummer-Vinson syndrome. D) He should have a workup for scleroderma. E) If a barium swallow is done, it usually is normal but can show a "corkscrew" pattern for this diagnosis.

Answer: E. If a barium swallow is done, it usually is normal but can show a "corkscrew" pattern for this diagnosis. Explanation He has diffuse esophageal spasm. It most commonly causes an intermittent dysphagia for both solids and liquids, but especially for cold liquids. Barium swallow, if done, is frequently normal but may show a "corkscrew" pattern. Symptomatic therapy with calcium channel blockers or antispasmodics may be helpful. Usually, it is self-limited and reassurance is the key. Plummer-Vinson syndrome occurs in middle-aged-to-elderly females and is associated with iron deficiency, cervical esophageal web, and an increased risk of cancer.

A 35-year-old woman with systemic lupus erythematosus complains of fatigue and vague abdominal pain. She has vomited twice today. Her blood pressure during inspiration is 70/40—during expiration, it increases to 90/40. Neck vein distension is noted on exam. Heart tones are muffled. ECG findings would be most likely to include: A) Prolonged QT interval B) Shortened PR interval C) Complete right bundle-branch block D) Bradycardia E) Low voltage of the QRS complexes

Answer: E. Low voltage of the QRS complexes Explanation The patient has a pericardial effusion, as evidenced by muffled heart tones, distended neck veins, and increased pulsus paradoxus—the difference between systolic pressure on inspiration and expiration. A difference of 20 mmHg or more suggests that the amount of pericardial fluid is sufficient enough to cause cardiac tamponade. Low voltage of the QRS complexes is often associated with tachycardia, ST-segment elevation, and T wave changes. An elderly man's elevated neck veins are easily seen on exam. He has known advanced left and right heart failure from previous myocardial infarctions.

A 25-year-old pregnant woman at 25-weeks gestation presents for evaluation. The patient says that she thinks she had rheumatic fever as a child. She presents today in atrial fibrillation with pulmonary edema. You start therapy and order an echocardiogram. Which of the following is the most likely etiology for her atrial fibrillation and pulmonary edema? A) Coronary aneurysm B) VSD C) Atrial stenosis D) IHHS E) Mitral stenosis

Answer: E. Mitral stenosis Explanation A pregnant patient who presents with new-onset atrial fibrillation and pulmonary edema is likely due to either mitral stenosis or secundum ASD. Knowing that she possibly had rheumatic fever makes mitral stenosis very likely. The other etiologies would be unusual to cause atrial fibrillation and/or pulmonary edema.

A 35-year-old woman presents with sudden onset of shortness of breath. She has atrial fibrillation on the monitor. The physical examination shows an opening snap at the apex, and the S 1 appears to be loud. You hear a low-pitched diastolic rumble at the apex. ECG shows atrial fibrillation with left atrial enlargement. Which of the following cardiac abnormalities is most likely? A) Aortic regurgitation B) Aortic stenosis C) Mitral regurgitation D) Hypertrophic cardiomyopathy E) Mitral stenosis

Answer: E. Mitral stenosis Explanation Her physical findings are classic for mitral stenosis. Remember that the symptoms may not show up for many years and frequently are heralded with atrial fibrillation. On the Board exam, also look out for the pregnant woman with mitral stenosis who presents with pulmonary edema, bloody frothy sputum, and onset of atrial fibrillation.

A 55-year-old man and his wife arrive at your hospital to visit a friend who is an inpatient. While waiting for an elevator, the man suddenly grabs his chest, hollers out in pain, and then falls against the back wall. A code is called and you respond. His wife reports that for 3 months, he has been having occasional episodes of pain in his chest that awakens him from sleep in the early morning. He never gets pain while doing chores at home, such as mowing the grass. He recently underwent an ECG and exercise stress test at his internist's office and was told his results were "normal." He is otherwise healthy. His only medicine is an aspirin a day that he self-prescribed at the onset of his chest pains. Family history is unremarkable. He does not smoke or use drugs. Vital signs: HR: 120 Respirations: 18 BP: 110/65 mmHg, afebrile He is slightly confused. The physical exam is otherwise normal. There is no nuchal rigidity. Within 15 minutes, he has regained normal mentation. ECG during this episode: Sinus tachycardia with ST-segment depression in leads V1-V4. ECG performed 15 minutes after recovery: Normal Urgent echocardiogram shows normal LV wall motion with a normal estimated ejection fraction Laboratory: cTn-I 30 minutes after the episode: < 0.01 ng/mL Cardiac catheterization shows normal coronary arteries. Which of the following is the most appropriate pharmacotherapy? A) Aspirin B) Lidocaine C) Aspirin, propranolol, enalapril, and PRN nitrates D) Aspirin, propranolol, enalapril, clopidogrel, and tirofiban E) Nitrates and calcium channel blockers

Answer: E. Nitrates and calcium channel blockers Explanation This is a case of variant angina. In support of this diagnosis: history of rest pain, ST depression that rapidly resolves and is not associated with the release of cardiac enzymes, and normal coronary arteries. No arrhythmia was noted during the event, so lidocaine is not justified. However, syncope associated with variant angina is often due to occasional ventricular arrhythmias, so careful monitoring is warranted. The normal cardiac catheterization excludes unstable angina, so treatment for acute coronary syndromes is unnecessary. Nitrates and calcium channel blockers are effective in reducing the numbers of events of vasospasm in cases of variant angina. Propranolol actually increases the risk of vasospasm. Aspirin is generally not used in these patients because it inhibits prostacyclin and will increase risk of spasm.

Your next patient is a 32-year-old African-American attorney who has been under extraordinary stress in the midst of a high-profile courtroom drama. He asked to see you because of chest pains. Your review of his ECG shows ST-segment elevations in multiple leads with prominent J-points and T-wave magnitudes that exceed the magnitude of the ST-segment elevations in those leads bearing the ST abnormalities. The patient's BP is (and has been) normal, as is his physical examination. Fortunately, the patient's life insurance evaluations had included an ECG tracing that you are able to obtain for comparison, and you note no significant changes between the 2 ECGs. You know that the most likely explanation of the ST-segment findings is which of the following? A) Recording artifact B) Chronic pericarditis C) Prinzmetal's variant angina pectoris D) Hypothyroidism (and hypothermia) E) Normal repolarization variant

Answer: E. Normal repolarization variant Explanation Early repolarization (ER), also known as benign early repolarization (BER) or normal variant, is noted in approximately 1% to 2 % of the population and in up to 48% of patients with chest pain who are seen in the emergency department. The electrocardiographic characteristics of ER include widespread ST-segment elevation, upward concavity of the initial portion of the ST segment, notching of the terminal QRS complex, and concordant T waves of large amplitude. The incidence of 1 to 2% is found equally common in all races. The findings are known to regress during exercise testing. The nearly universal impression of an increased frequency in African-American men is not well documented in the literature, although there are very thoughtful statements as to the significance with respect to both thrombolysis criteria and cocaine usage. This patient's chest pain may be GI (stress → GERD), as well as anxiety over the case he is working on.

A 67-year-old man was admitted to the CCU with an acute ST-segment elevation MI. He was stabilized and initially received aspirin, thrombolytic therapy, heparin, nitrates, and beta-blockers. On day 2, he has multiple runs of ventricular tachycardia that are asymptomatic and last only a few seconds. Eventually, his thrombolytics and anticoagulants are discontinued. On day 5, he becomes acutely short of breath. Vital signs: BP 96/50 mmHg; pulse 110 bpm; respiration 24; afebrile On exam, there is a systolic murmur that begins in midsystole and extends to the second heart sound. It is audible over the entire chest. Which of the following is the most appropriate next step in patient care? A) Prescribe massive volume resuscitation for presumed right ventricular infarction. B) Order an immediate surgical consultation for repair of a ventricular septal defect (VSD). C) Immediately aspirate the pericardial sac for presumed tamponade. D) Re-institute thrombolytic therapy for a recurrent MI. E) Order an immediate surgical consultation for repair of a ruptured papillary muscle.

Answer: E. Order an immediate surgical consultation for repair of a ruptured papillary muscle. Explanation Mechanical complications of acute MI include ventricular free wall rupture (< 1%), intraventricular septal rupture (2%), and acute mitral regurgitation. This patient's exam is consistent with mitral regurgitation. Causes of acute mitral regurgitation include LV dilatation, papillary muscle ischemia, and papillary muscle chordae rupture. Timing of papillary muscle/chordae rupture is usually 2-7 days post-MI, and it presents with hypotension, pulmonary edema, and late holo- or midsystolic murmur (a new murmur in the setting of acute MI). Emergent surgical intervention is necessary, because perioperative mortality is 27%. Repeating thrombolytic therapy in this patient is not warranted unless a diagnosis is confirmed. This patient's exam is not consistent with pericardial tamponade or a VSD. RV infarction may present with hypotension but would not lead to pulmonary edema or the murmur described.

A 60-year-old man with negative past medical history presents for his annual physical examination. He reports no problems. PAST MEDICAL HISTORY: Really, really negative! SOCIAL HISTORY: Works as a minister in local community church Not married Doesn't smoke Doesn't drink FAMILY HISTORY: Father died at age 78 of MI Mother died at age 55 of MI REVIEW OF SYSTEMS: Negative PHYSICAL EXAMINATION: BP 130/76, P 110, RR 18, Temp 99° F HEENT: PERRLA, EOMI TMs clear Throat clear Heart: RRR without murmurs, rubs, or gallops Lungs: CTA Abdomen: Bowel sounds present, no hepatosplenomegaly Extremities: No cyanosis, clubbing, or edema LABORATORY: CBC normal; no anemia Lytes normal ECG:12 lead tracing; shows 2 premature ventricular complexes (PVCs) Which of the following statements about premature ventricular complexes is true? A) His PVCs predict a higher incidence of cardiac mortality. B) The frequency of isolated PVCs decreases with age. C) Less than 40% of males have PVCs on a 24-hour Holter monitor. D) The frequency and nature of PVCs cannot be correlated with increased mortality in patients with known coronary artery disease. E) PVCs such as these can cause symptoms.

Answer: E. PVCs such as these can cause symptoms. Explanation Palpitations are the most common symptoms that people will perceive from them. Symptoms can also occur if the stroke volume is decreased by decreasing overall ventricular filling. PVCs are common and occur in nearly 60% of men who undergo Holter monitoring. In patients who have known coronary artery disease, the frequency and nature of the PVCs do matter; generally, > 30/hour is associated with increased mortality. PVCs increase in frequency with aging, not decrease.

Your new patient today is a 64-year-old retired coal miner who presents with progressive dyspnea and peripheral edema. He has sought compensation for "black lung" and has smoked non-filter cigarettes up to 3½ packs daily for 52 years. He begins each day with a 90-minute coughing session productive of "from ½ to ¾ of a coffee cup" of dark sputum ("almost like tar"). His PaO2 on room air is 53 mmHg with a PaCO2 of 52 mmHg, HCO3 of 32 mmol/L and pH of 7.37. His ECG reveals maximum positive-amplitude R waves in limb lead III and isoelectric QRS complexes in lead aVR. The most likely cause of QRS frontal plane axis deviation in this patient is which of the following? A) Coarctation of the thoracic aorta, post-ductal (adult type) B) Severe non-calcific valvular pulmonic stenosis, congenital C) Severe calcific valvular aortic stenosis, acquired D) Systemic arterial hypertension, "essential" of long-standing E) Pulmonary arterial hypertension, acquired via cor pulmonale

Answer: E. Pulmonary arterial hypertension, acquired via cor pulmonale Explanation In contrast to the patient with pulmonary thromboembolism and acute cor pulmonale, here is the typical picture of chronic cor pulmonale heart disease in the setting of what is no doubt chronic bronchitis. This is the traditional association of the classic "blue bloater" and the development of pulmonary hypertension, no doubt as a function of the chronic arterial hypoxemia for which they are so well known, although the mechanism is the same in cystic fibrosis.

A 33-year-old man presents with increasing shortness of breath and dyspnea on exertion. He says he had a heart murmur as a child but never had it evaluated. Physical Examination: He is acyanotic and in no distress at rest. He is afebrile and has a heart rate of 90, BP is 120/70, and RR is 26. His chest is clear to auscultation. His heart exam shows a right ventricular lift. S 1 is normal and S 2 is split with a decreased pulmonary component. There is a systolic click at the left upper sternal border. He has a 3/6 systolic murmur at the left upper sternal border as well. No diastolic murmur is present. CXR: Mild right ventricular enlargement with post-stenotic dilatation of the main pulmonary artery and normal peripheral pulmonary arteries and lung fields. ECG shows RAD and RVH. An echocardiogram is ordered. Which of the following is the most likely diagnosis? A) Aortic valve stenosis (AS) B) Coarctation of the aorta C) Ventricular septal defect (VSD) D) Atrial septal defect (ASD) E) Pulmonic valve stenosis (PS)

Answer: E. Pulmonic valve stenosis (PS) Explanation He has findings of pulmonic valve obstruction with symptoms of dyspnea and fatigue; the murmur findings listed in his physical examination and his ECG are consistent with PS. Many patients with moderately severe pulmonary valve stenosis will be asymptomatic in childhood. However, by the time they reach their 20s or 30s, most patients will be symptomatic with dyspnea, shortness of breath, and increased fatigue. Pulmonic valve obstruction needs to be relieved or the symptoms will be progressive.

A 52-year-old CEO sits down for dinner with his family but is reluctant to eat because of an overpowering feeling of "indigestion." As a registered nurse, his wife is insistent on a 911 call, and the patient is transported to your emergency department. His BP is 90/70 mmHg, HR 50/min, and his general appearance is characterized by sweating and ashen pallor. The ECG shows 2-3 mm ST-segment elevation in leads II, III, and aVF accompanied by "hyperacute" (tall peaked) T waves but no Q waves at this point. Which of the patient's coronary arteries listed below is likely occluded? A) Left anterior descending (LAD) B) Diagonal branch of the LAD C) Left circumflex (LCX) D) Right ventricular branch of the RCA E) Right coronary artery (RCA)

Answer: E. Right coronary artery (RCA) Explanation This is the classic picture of inferior myocardial infarction and should correlate with occlusion of a right coronary artery.

A 60-year-old man presents with a large anterior myocardial infarction. He is hypotensive and tachycardic. Vital signs are tenuous at best with a BP of 90/50 mmHg and a heart rate of 120. He begins to become unresponsive and appears to be deteriorating on arrival to the emergency department. Which of the following treatments would not be appropriate? A) Getting the patient to the cardiac cath lab immediately B) Placement of an intraaortic balloon pump C) Placement of a Swan-Ganz catheter to monitor pressures and output D) Starting inotropic agents (dopamine or dobutamine) E) Starting nitroprusside or nitroglycerin IV for afterload reduction

Answer: E. Starting nitroprusside or nitroglycerin IV for afterload reduction Explanation That would be the one thing here you would not want to do. His blood pressure is already low, and he is in cardiogenic shock. You do not want afterload reduction—this will make his hypotension acutely worse. All of the other choices are appropriate, depending on the hospital setting you are in. Obviously, if a cath lab was not available, management should include fibrinolytic therapy, inotropic agents, and placement of an intraaortic ballon pump (if possible), followed by immediate transfer to a PCI (percutaneous coronary intervention) facility.

A 45-year-old man presents with a large anterior myocardial infarction. He is hypotensive and tachycardic. Vital signs are seriously affected with a BP of 85/50 and a heart rate of 120. He is becoming unresponsive and appears to be deteriorating on arrival to the emergency department. Which of the following treatments is inappropriate at this time? A) Placement of an intraaortic balloon pump. B) Emergency revascularization with either PCI or CABG is recommended in suitable patients with cardiogenic shock due to pump failure after STEMI irrespective of the time delay from MI onset. C) In the absence of contraindications, fibrinolytic therapy should be administered to patients with STEMI and cardiogenic shock who are unsuitable candidates for either PCI or CABG. D) Starting inotropic agents (dopamine or dobutamine). E) Starting nitroprusside or nitroglycerin IV for afterload reduction.

Answer: E. Starting nitroprusside or nitroglycerin IV for afterload reduction. Explanation Starting nitroprusside or nitroglycerin IV for afterload reduction would be the one thing here you would not want to do. His blood pressure is already low, and he is in cardiogenic shock. You do not want afterload reduction—this will make his hypotension acutely worse. All of the other choices are appropriate depending on the hospital setting you are in. According to the 2013 ACCF/AHA Guideline for the Management of ST-Elevation Myocardial Infarction: Executive Summary, treatment of cardiogenic shock, Class I includes: 1. Emergency revascularization with either PCI or CABG is recommended in suitable patients with cardiogenic shock due to pump failure after STEMI irrespective of the time delay from MI onset. 2. In the absence of contraindications, fibrinolytic therapy should be administered to patients with STEMI and cardiogenic shock who are unsuitable candidates for either PCI or CABG.

What MI location is the MOST COMMON?

Anterior - LAD

If a pt is post MI w/ a second or third degree heart block, what is the appropriate treatment measures if it is an anterior MI? Inferior MI?

Anterior MI: pacemaker (temp then permanent) - worse prognosis Inferior MI: atropine IV, if not controlled then temporary pacement (better prognosis)

Name the leads and corresponding artery for which ST-Elevation/Depression can be seen for the following infarctions: Anterior MI? Lateral MI? Inferior MI? Posterior MI?

Anterior: V1-V4 - LAD Lateral: I, avL, V5, V6 - Circumflex Inferior: II, III, avF - RCA Posterior Large R wave in V1/V2 - RCA

What drugs are C/I to use with statins?

Antifungal (-azoles) and Abs (Erythromycin, Clarithromycin)...results in myosite and rhabdomyositis

In the CHD "transposition of the great arteries", what arteries are switched?

Aorta and Pulmonary Artery are switched

What does a bifid uvula suggest?

Aortic Aneurysm and Loeys-Dietz syndrome (is a newly recognised disorder of connective tissue which shares overlapping features with Marfan syndrome (MFS) and the vascular type of Ehlers-Danlos syndrome, including aortic root dilatation and skin abnormalities)

What is the *GOLD STD* for determining the extent of dissection for surgery?

Aortic Angiography

What is the common causes for AR?

Aortic Leaflet Abnormality: • *MC: Infective Endocarditis* • Bicuspid Aorta • Rheumatic heart Disease • Anorexigenic drugs Aortic Route Abnormality: • Marfaran Syn • HTN • Aortic Dissection • Syphillis • Ankylosing spondylitis • Psoratic arthritis • Lyme disease

Pt presents to the clinic with a *harsh cresendo-descendo systolic murmur* heard at the 2nd ICS RUSB that *radiates to the carotids*. What murmur is being described?

Aortic Stenosis

What is the *TOC* for AS?

Aortic valve replacemnt is the *TOC* - it is indicated in symptomatic patients

What is the *GOLD STD* for diagnosing and locating PVD?

Arteriography (contrast in vessels and RAD) - ONLY needed however, if surgery (revascularization) is being considered

After acute event, what tx are used for USA?

Aspirin, BB, nitrates

What are the sx of MS?

Asx until MV reaches <1.5cm2 then the sx include DOE, orthopnea, PND, palpitations, CP, hemoptysis, Thromboembolism, if RVF occurs then ascities and edema may develop.

When diagnosing HTN in a patient, how many in office BP readings are required?

At least *2 readings over a span of 4+ weeks* The only exception is if the pt has severe HTN or the evidence of end-organ damage - then you diagnosis it on 1 BP reading

What is the *MOST COMMON* primary cardiac neoplasm?

Atrial Myxoma - benign gelatinous growth, usually pedunculated and usually arising from the interatrial septum of the heart in the region of the fossa ovalis. Most are sporaticaly inherited.

HCM is what type of genetic disorder?

Autosomal Dominant

How long before a BP reading does a pt need to stop smoking or using caffine? A. 15 min B. 30 min C. 1 hour D. 90 min

B. 30 min • Make sure the patient has not ingested caffeine or smoked cigarettes in the past 30 minutes (both elevate BP temporarily).

Name the NYHA described..."symptoms occur w/ *prolonged or moderate exertion*, such as climbing a flight of stairs or carrying heavy packages. *Slight limitation* in activity"? A. I B. II C. III D. IV

B. II

Define the following arrhythmia description: "P wave fails to conduct suddenly, without a preceding PR interval prolongation therefore, the QRS drops suddenly". A. Sinus brady B. Mobitz type II C. Winkebach D. First defree AV block

B. Mobitz Type II (second degree heart block) Often progress to complete heart block Site of block is within the *His-Purkinje System* Tx: Pacemaker implementation is generally indicated

It a pt comes in with a BP of 130/88, what is their classification of HTN? A. Normal B. PreHTN C. HTN Stage 1 D. HTN Stage 2

B. PreHTN (SBP: 120-139, DBP: 80-89) Tx: Lifestyle modification

What is the most likely murmur, based upon he sx listed below: Systolic Minor turbulence of right ventricular outflow and pulmonary artery Grade 1-3/6 in intensity Blowing in quality Loudest: LUSB Medium pitch Decreased with inspiration, sitting up, standing up A. Still's Murmur B. Pulmonary ejection murmur C. Pulmonary flow murmur D. Venous hum E. Carotid Bruit

B. Pulmonary Ejection murmur

If pt presents with the classic symptoms of shock + fever, possible site of infxn, what type of shock is most likely? A. Hypovolemic B. Septic C. Neurogenic D. Cardiogenic

B. Septic

What type of CAD is being described: CP that lasts < 10/15 min that is described as heaviness/pressure that is brought on by exertion, and relieved by rest or NTG? A. Unstable Angina B. Stable Angina C. Prinzmetal's Angina

B. Stable Angina

Which form of endocarditis is associated w/ a "damaged heart valve"? A. Acute B. Subacute

B. Subacute

The veins in the leg consits of superficial, deep, and perforating veins. The perforating veins connet the superficial and deep systems. Valves from from ____ to _____, and *not* visa versa. A. Deep, Superficial B. Superficial, Deep

B. Superficial to deep ALWAYS never the oppositie

What dyslipidemia medications are indicated in children, pregnant, and breastfeeding pts?

BAS

What is the definition for HTN emergency?

BP: >220/120, in addition to end-organ damage-immediate treatment is indicated

What is the characteristic findings on CXR for constrictive pericarditis?

BQ - CXR shows *calcification ring* ECHO can also show pericardial thickening - CT/MRI ae more accurate in diagnosis pericardial thickening

What is the treatment for TOGA?

Balloon atrial septostomy, if inadequate shunting (emergency)

Are atrial myxoma malignant or benign?

Benign - however they can embolize, leading to metastic disease or cause relative valvular dysfunction

What HTN treatment is indicated for post-myocardial infarction?

Beta blocker, ACE-I

What is the *MOST COMMON CAUSE* of secondary HTN in young women?

Birth Control Pills

What is the *MOST COMMON* age for a pulmonary flow murmur?

Birth to infancy: disappears early in infancy (3-6 mo) Common in pre-term neonate

What is the treatment for tetrology of fallot?

Blalock-Taussig procedure

What is the tx for tricupsid atresia?

Blalock-Taussig shunt or Prostaglandin

Femoral and popliteal PVD causes _____ claudication? A. buttox B. Thigh C. Calf D. Hip

C. Calf Claudication

If a pt comes in with a BP of 145/93, what is their classification of HTN? A. Normal B. PreHTN C. HTN Stage 1 D. HTN Stage 2

C. HTN Stage 1 (SBP: 140-159; DBP: 90-99) Tx: Lifestyle Modification + Drug therapy

Name the NYHA described...."symptoms occurs w/ *usual activities of daily living*, such as walking across the room or getting dressed. *Marketedly Limited*"? A. I B. II C. III D. IV

C. III

What is CAD risk primarily due to: A. VLDL B. HDL C. LDL D. Total Cholesterol

C. LDL (esp levels >160) Calculate LDL: Total Chol - HDL - TG/5

Define the following arrhythmia description: "progressive prolongation of the PR interval until a P-wave fails to conduct" A. Sinus brady B. Mobitz type II C. Winkebach D. First defree AV block

C. Mobitz I - Winkebach (second degree heart block) -Site of block usually @ AV node -Benign condition that does not require tx

If pt presents with the classic sx of shock + SC injury or neurologic deficits, what type of shock is most likely? A. Hypovolemic B. Septic C. Neurogenic D. Cardiogenic

C. Neurogenic

What form of angina is characterized by *"transient coronary vasospasm"*, the episodes occur at rest, and ass w/ ventricular dysfunction? A. Stable Angina B. Unstable Angina C. Prinzmetal's Angina

C. Prinzmetal's Angina

What is the TOC is severe CAD?

CABG

What procedures can be used with "reversible" ischemia?

CABG and PTCA

What is the *MOST COMMON CAUSE* of V. Tach?

CAD w/ Prior MI = MCC OTher: active ichemia, hypotsn, CM, CHD, prolonged QT syndrome, drug toxicity

What is the *GOLD STANDARD* for MI diagnosis?

CARDIAC ENZYMES (<-- Per Stepup to Medicine) ANGIOGRAPHY (<-- Per Ovalle)

What is the MCC of heart failure due to fluid overload in infants and toddlers?

CHD such as PDA and VSD.

What cardiac enzyme increases w/in 4-8 hour - peaks at 24 hours - and returns to nl at 48-72 hours?

CK-MB (best when measured 24-36 hours) Measure @ admission - every 8 hours for first 24 hours

What is the MCC of multifocal atrial tachycardia?

COPD

What is the *MOST COMMON CAUSE* of A. Flutter?

COPD = MCC Other: rheumatic heart disease, CAD, CHF, ASD

What is the *DEFINITVE TEST* for prinzmetal's angina?

CORONARY ANGIOGRAPHY - displays coronary vasopasm when the pt is given "IV ERGONOVINE"

What are the indications for stents and thrombolytics?

CP 30-6 hours: ST elevation of 1mm or greater in 2 leads in STEMI --> use thrombolytics o CP < 30 min → STENT in STEMI/NSTEMI o CP > 6 hours - risk to benefit ratio in STEMI NEVER USE USE THROMBOLYTICS IN NSTEMI*** TPA - most expensive and most effective Streptokinase: least expensive an less effective

What test is 100% sensitive in dectecting AAA, but should only be used in hemodynamically stable pt?

CT (you *cannot* use plain radiography to r/o AAA)

What are the characteristic CXR findings for pericardial effusion?

CXR shows enlargement of cardiac silhouette when *>250 mL* of fluid has accumulated Cardiac silhouette may be have prototypical *"water bottle"* appearance. An enlarged heart w/o pulmonary vascular congestion suggests pericardial effusion

What are the CXR and EKG findings for hypoplastic left heart syndrome?

CXR: Enlarged heart with increased PVM and PV congestion ECG: RVH

What diagnostic tests can you order for AR?

CXR: LVH, dilated aorta ECG: LVH Echocardiogram-perform serially in chronic, stable patients to assess need for surgery to Assess LV size and function and Look for dilated aortic root and reversal of blood flow in aorta. Also in acute aortic regurgitation, look for early closure of mitral valve. Cardiac catheterization: to assesses severity of aortic regurgitation and degree of LV dysfunction

What are the CXR, EKG, and ECHO for AS?

CXR: calcific aortic valve, enlarged LV/LA (late) ECG: LVH, LA abnormality Echocardiogram-diagnostic in most cases. Findings include LVH; thickened, immobile aortic valve; and dilated aortic root

What findings can be seen for MR on CXR and ECHO?

CXR: dilated LV, pulmonary edema Echocardiogram: MR; dilated LA and LV; decreased LV function

What findings are seen on CXR and EKG for ASD?

CXR: large pulmonary arteries; increased pulmonary markings (vascularity) ECG: right bundle branch block and right axis deviation; atrial abnormalities can also be seen (e.g., fibrillation, flutter)

What diagnostic tests can be ordered and what are the results in MS?

CXR: left atrial enlargement (early) Echocardiogram-*most important test* in confirming diagnosis a. Left atrial enlargement b. Thick, calcified mitral valve c. Narrow, "fish-mouth"-shaped orifice d. Signs of RVF if advanced disease

Defined Cardiac Arrest and Sudden Cardiac Death.

Cardiac Arrest: sudden loss of CO; potentially reversible if circulation and O2 delivery are promptly restored Sudden Cardiac Death: unexpected death w/in 1 hour of sx onset secondary to a cardiac cause

What is the definitive diagnostic test for AS?

Cardiac Cath Can measure valve gradient and calculate valve area-< 0.8 cm2 indicates severe stenosis; normal aortic valve is 3 to 4 cm2 c. Useful in symptomatic patients before surgery

What diagnostic procedure is required for constrictive pericarditis?

Cardiac Cath - elevated and equal diastolic pressure in all chambers - ventricular pressure tracing shows a rapid "y" descent, which has been described as a dip, plateau, or a "square root sign"

NSTEM tends to be smaller than a STEMI and presents similar to USA, how can you differentiate the two?

Cardiac Enzymes

What are the complications of infective endocarditis?

Cardiac Failure Myocardial Abscess Various solid organ damage from showed emboli Glomerulonephritis

What complications account for the majority of deaths in hypertensive patients?

Cardiac complications - CAD, CHF w/ LVH, stroke, renal failure, PVD, and Aortic Dissection MOST deaths due to HTN are ultimately due to *MI/CHF*

Describe the changes in CO, SVR, and PCWP in patients with... (1) Cardiogenic shock (2) Hypovolemic Shock (3) Neurogenic Shock (4) Septic Shock

Cardiogenic: CO: decreased SVR: increased PCWP: *increased* <-- NOTE! Hypovolemic: CO: decreased SVR: increased PCWP: decreased Neurogenic: CO: decreased SVR: decreased PCWR: decreased Septic CO: *increased* <-- NOTE! SVR: decreased <-- Severely decreased PCWR: decreased

What are the CXR findings for CHF?

Cardiomegaly *KERLY B LINES* Prominent interstitial markings Pleural Effusion

The coronary cath looks at what? Coronary angiography looks at what?

Cath --> determining a cardiac diagnosis Angiography --> presence and severity of CAD; looks at delineating coronary anatomy

What is the pathophysiology behind HCM?

Characterized by *thickened left ventricular walls* in the *absense* of valvular obstruction or systemic HTN, LV is decreased in size, Normal or increased systolic contraction 2 types: non-obstructive and obstructive

What syndrome is caused by "showers" of cholesterol crystals originating from a peripheral source (atherosclerotic plaque)?

Cholesterol Embolization Syndrome

Pt presents to the clinic and describes the following S&S, what is the most likley dx? Sx: aching and tightness feeling of the R leg (involved); often worse at the end of the day and leg elevation *improves* sx.

Chronic Venous Insuff - classic presentation Contrast to Arterial insuff: leg elevation *does not* improve sx.

What are the symptoms of cardiogenic shock?

Classic symptoms: hypotension, oliguia, tachy, AMS PLUS enlarged neck veins and pulmonary congestion

What are the *CLASSIC* S&S for DVT? Are these typically seen in patients?

Classical symptoms: a. Lower-extremity pain and swelling (worse with dependency/walking, better with elevation/rest) b. Homans' sign (calf pain on ankle dorsiflexion) c. Palpable cord d. Fever These are not typically see in pts NOTE: Remember the *HOMANS SIGNS* can always be performed when looking for a DVT, however it is not a reliable test.

A 5 mo pt presents the pediatric clinic, on exam you find the following results: Bounding radial pulses, Diminished femoral pulses, BP elevated in arms/decreased in legs. What is your *MOST LIKELY* diagnosis?

Coarc of the Aorta Sx: SEM loudest at ULSB, *Bounding radial pulses, Diminished femoral pulses, BP elevated in arms/decreased in leg*, Intermittent claudication, HA, epistaxis, paresthesias, CHF

What are the EKG findings for RCM?

Conduction disturbancees and low voltage QRS complexes are common w/ amyloidosis and sarcoidosis - RBBB/LBBB may be present

What management should be taken for intermittent claudication?

Conservative *#1 = STOP SMOKING* 2) Graduated exercise program - walk to pt of claudication, rest, then cont walking 3) Foot care 4) Atherosclerotic risk factor reduction (hyperlipidemia, HTN, wt, DM...) 5) avoid extremes fo temp

What is the treatment for both stable&asx patients and symptomatic patients?

Conservative if stable and asymptomatic: salt restriction, diuretics, vasodilators, digoxin, afterload reduction (i.e. , ACE inhibitors or arterial dilators) , and restriction on strenuous activity *Definitive treatment* is surgery (aortic valve replacement). This should be considered in symptomatic patients, or in those with significant LV dysfunction on echocardiogram. Acute AR (e.g., post-MI): *Medical emergency-Perform emergent aortic valve replacement* Endocarditis prophylaxis before dental, GI, GU procedures

What condition is defined by *fibrous scarring ofthe pericardium* that leads to thickening of the pericardium. with obliteration of the pericardial cavity?

Constrictive Pericarditis

What is the *GOLD STANDARD* diagnostic procedure for CAD?

Coronary Angiography

Define the following terms in regards to PVCs: Couplet? Bigeminy? Trigeminy?

Couplet: two successive PVCs Bigeminy: Sinus beat followed by a PVC Trigeminy: sinus beat followed by two PVCs

If pt presents w/ the classic symtpoms of shock + history of MI, angina, or heart disease or if JVD is presents, what type of shock is most likely? A. Hypovolemic B. Septic C. Neurogenic D. Cardiogenic

D. Cardiogenic

Define the following arrhythmia description: (1) PR Interval is prolonged (>20 seconds) (2) A QRS follows each P waves A. Sinus brady B. Mobitz type II C. Winkebach D. First defree AV block

D. First degree AV block - Delay is usually @ the AV node -Benign condition that DOES NOT require tx

If a pt comes in witha BP of 172/110, what is the their classification of HTN? A. Normal B. PreHTN C. HTN Stage 1 D. HTN Stage 2

D. HTN Stage 2 Tx: lifestyle mod w/ 2 drug combination theapy

Name the NYHA described...."symptoms occur *at rest*. Incapacitating.? A. I B. II C. III D. IV

D. IV

What is the arrhythmia that is defined as *"rapid and repetitive firing of 3+ PVCs in a row, at a rate of 150 - 250 bpm? A. A. Fib B. V. Fib C. Trigeminy D. V. Tach

D. V. Tach

Based on the following description, what is the most likely murmur? Turbulence in Superior vena cava Continuous murmur Grade 1-3/6 in intensity Loudest: left infra-clavicular area, especially with sitting/standing disappears while lying supine Intensity changes with rotation of head and disappears with compression of the jugular vein A. Still's Murmur B. Pulmonary ejection murmur C. Pulmonary flow murmur D. Venous hum E. Carotid Bruit

D. Venous Hum

What pt has the highest amputation risk?

DM

What are the risk factors for PVD?

DM, Smoking, CAD, hyperlipidemia, HTN, hyperhomocystinemia

What is the underlying cause for chronic venous insuff in many cases?

DVT Leads to ambulatory venous HTN, which result in edema, extravastion of plasma & RBCs into SQ resulting in brawing induration and brown-black color of skin. Eventually leads to reduced local capillary blood flow and hypoxia of tissues, mild trauma can precipitate tissue death.

What is definition of cardiogenic shock?

Defined as *SBP <90* w/ *urine output <20 ml/hr* and adequate LV filling Occurs when the heart is unable to generate a CO sufficient for tissue perfusion

In CHF there is systolic and diastolic dysfunction...what are they defined by and what is the *MOST COMMON CAUSE* of each?

Defined by *Ejection fraction* Systolic: <50% or decreased EF *MCC* - post MI Other: DCM/Myocarditis Diastolic: Nl/increased EF *MCC* = HTN leading to myocardial hypertrophy Other: AS, MS, AR, RCM/HCM

What fluids CANNOT be used in acute shock?

Dextrose (D5W) - the body cannot use it!

What pateints are niacin c/i in?

Diabetic patients (may worsen glycemic control)

HCM is which type of dysfunction: A. Systolic B. Diastolic

Diastolic (filling defect)

What are the common RF for dyslipidemia?

Diet - high in sat fat, choleserol, and calories, alcohol Age - increase until 65 y/o Inactive lifestyle, abdominal obesity Fmhx Male Meds (1) Thiazides - increases LDL, total, TG (2) BB - increase TG and lower HDL (3) Estrogen - TG increase w/ underlying dyslipidemia (4) Steroids and HIV drugs - increase serum lipids

What is the *MOST COMMON* cardiomyopathy?

Dilated CM

What is the *ONLY* CXR finding for AS in children?

Dilation of AA is only radiographic sign in children

What are the signs of PVD?

Diminished or absent pulses, muscular atrophy, decreased hair growth, thick toenails, and decreased skin temperature Ischemic ulceration (usually on the toes) • Localized skin necrosis • Secondary to local trauma that does not heal (due to ischemic limb) • Tissue infarction/gangrene in end-stage disease Pallor of elevation and rubor of dependency (in advanced disease)

What are the signs of left sided heart failure?

Dislaced PMI to the left Pathologic S3 (ventricular gallop) - S4 can be present Crackles/rales at lung bases (pulmonary edema, moderate severity of left ventricular HF) Dullness to percusssion and decreased tactile fremitus of lower lung feilds increased pulmonic component of heart sounds indicates pulmonary HTN

What is the tx for isolated systolic HTN?

Diuretic (preferred), calcium channel blocker (alternative therapy)

What is the initial test for DVT?

Doppler anlaysis and u/s -noninvasive but high operator dependent -high sensitive and specificity for proximal thrombi (popliteal and femoral), not so for distal (calf vein) thrombi

What are some drugs that can cause DCM?

Doxirubicin and Adriamycin

What is the name of the "postmyocardial infarction syndrome" that is immunologically based and consists of fever, malaise, pericarditis, leukocytosis, and pleuritis occuring weeks to months post-MI?

Dresslers Syndrome TOC = ASA

A very tall, thin young man comes to your clinic complaining of mid-back pain. He says 3 days ago, he began having a sharp, non-radiating, continuous pain between his scapulae. The pain has been severe enough that he has been unable to sleep. He has not had any similar symptoms previously, and has no other known medical problems. His examination is remarkable for his 6' 8" height; long, spindly fingers; and pectus excavatum. There is no palpable tenderness in his back. His femoral pulses seem somewhat diminished. On neurological examination, he has some mild weakness to dorsiflexion of his right foot. Expecting bad things, you consult your cardiologist colleague, and she elects to perform a transesophageal echocardiogram. Which of the following statements concerning this disorder is true? A) Lowering the aortic systolic pressure is the most important aspect of medical therapy. B) Appropriate medical therapy for an aortic dissection is intravenous nitroprusside alone. C) All descending thoracic aortic dissections require immediate surgery. D) Dissection of the ascending aorta should be treated with aggressive medical therapy and close observation. E) Aortic dissection can occur in the 3rd trimester of pregnancy without any obvious predisposing factors.

E. Aortic dissection can occur in the 3rd trimester of pregnancy without any obvious predisposing factors. Explanation Aortic dissections that involve the ascending aorta should always be treated surgically. Appropriate medical therapy is both beta-blockers and nitroprusside. The hemodynamic parameter that is most associated with extension of the dissection is the rate of rise of the aortic pressure. Nitroprusside decreases the aortic systolic pressure, but actually increases the rate of rise because of reflex tachycardia. Beta-blockers and nitroprusside together decrease the rate of rise. Most descending thoracic dissections can be treated medically. Aortic dissection is an uncommon complication of pregnancy, usually occurring in the 3rd trimester. Dissection of the coronary arteries may also occur in pregnant women.

Based on the folllowing description, what murmur is being described: Turbulence in carotid/subclavian artery Systolic ejection murmur Sometimes heard at base of neck Disappears with hyperextension of shoulders Grade 2-3/6 in intensity Occasional thrill over carotid A. Still's Murmur B. Pulmonary ejection murmur C. Pulmonary flow murmur D. Venous hum E. Carotid Bruit

E. Carotid Bruit

What is the *SOC* for pericardial effusion?

ECHO -confirms the presence or absense of a significant effusion - can show as little as 20 ml of fluid

Wha studies can be ordered to evaluate for TR?

ECHO EKG (RVH, RAE)

What is the *SOC* for all valvular heart diseases?

ECHO (If you want to visualize the left side of the heart i.e. the mitral valve, you can get a *TEE*)

What is the *SOC* (both sensitive, specific, and diagnostic) for cardiac tamponade?

ECHO - *MUST* be performed if suspicion of tamponade exists

What diagnostic tests can be used in cardiogenic shock?

EKG - looks for STEMI, NSTEMI, Arrthythmias ECHO - dx mechanical complications Swan Gann Catheter (looks at PCWP - Normal LA Pressure -10 mmHg; Abnormal pressure = 15 mmHg)

What is the EKG and CXR findings for PDA?

EKG: LVH, LAE, LVH + RVH CXR: incraesed pulmonary artery vascularity, vein size

Explain the diagnostic findings of EKG, CXR, and ECHO for VSD.

EKG: LVH; LAE,+/-RVH CXR: Cardiomegaly; left atrial enlargement; incr. Pulmonary vasculature ECHO: Shows septal Defect

What is shown on an EKG for pericarditis?

EKG: ST SEGEMENT ELEVATION IN ALL LEADS EXCEPT AVR AND V1 - PR INTERVAL DEPRESSED BELOW BASELINE

What are the characteristic findings for PACs? What management would you recommend?

EKG: early p-waves that differ in morphology from the normal sinus P-wave - QRS is normal Management: can occur in healthy individuals and some patients can be asx so they *do not require treatement * If palpatations - prescribe *BB*

What is the *INITIAL TEST OF CHOICE*/*GOLD STD* for CHF?

Echo

What is the *SOC* for HCM?

Echo

What is the *SOC* for MVP?

Echo

What is the *SOC* to evaluate DCM?

Echo

What ECG findings suggests a massive pericardial effusion and tamponade?

Electrial Alternans

What lab is *pathognomic* for CHF?

Elevated *BNP*

What is the definition for HTN urgency?

Elevated BP levels along *w/o end organ damage* These rarely require emergency therapy and can be managed by attempts to lower BP over a periods of 24 hours.

If the TRIAD is met for a ruptured AAA, what is indicated *EMERGENTLY*?

Emergent Laparotomy

What is the treatment for endocarditis?

Empirical until blood culture are final → Nafcillin + PCN + Genta/ VANCO (MRSA/PCN allergy) Strep: PCN G/Cephtriaxone Staph: Nafcillin + PCN + gentamycine; oxacillin; Vanco (MRSA) Enterococci: PCN + streptomycin/gentamycin

what are the complications of VSD?

Endocarditis Progressive AR Heart Failure Pulmonary HTN and shunt reversal (*Eisenmenger's*)

What is the tx for VSD?

Endocarditis prophylaxis is important. Surgical repair is indicated if the pulmonary flow to systemic flow ratio is greater than 1.5:l or 2:l. For the asymptomatic patient with a small defect, surgery is not indicated.

What are the other causes of secondary HTN?

Endocrine causes-hyperaldosteronism, thyroid or parathyroid disease, Cushing's syndrome, pheochromocytoma, hyperthyroidism, acromegaly Medications-oral contraceptives, decongestants, estrogen, appetite suppressants, chronic steroids, tricyclic antidepressants (TCAs), nonsteroidal anti- inflammatory drugs (NSAlDs) Coarctation of the aorta Cocaine, other stimulants Sleep apnea

What diagnostic technique can be "diagnostic" in RCM?

Endomyocardial bx

What is the *MOST COMMON CAUSE* of HTN?

Essential/primary HTN (i.e. there is no identifiable cause)

What is the tx for AS in children?

Exercise restrictions (avoid contact sports), Surgery, Balloon Valvuloplasty in children, Valve replacement in adults

What diagnostic procedures can be used as a screening tool for CAD?

Exercise stress ECHO/EKG

A 22-year-old female Russian immigrant is hospitalized for evaluation of a swollen, warm, and tender ankle and knee. She reports that her joint pain has responded to self-treatment with aspirin. After arrival to her hospital room, she develops fever and an erythematous macular rash associated with pale centers. The rash is located on the trunk and extremities but is not pruritic. A picture is shown here: erythema marginatum Which of the following results is most likely to be associated with this patient? A) Prolonged PR interval on ECG B) Positive LE cell preparation C) Elevated antibody titers to Borrelia burgdorferi D) Blood culture positive for Salmonella E) Positive culture for Staphylococcus aureus following joint aspiration

Explanation She has rheumatic fever presenting with polyarthritis and erythema marginatum. The diagnosis is suggested by evident improvement of pain with salicylates treatment and rash that is accentuated by fever. Prolonged PR interval is a minor manifestation of rheumatic fever, according to the Jones criteria. Major manifestations include carditis, polyarthritis, chorea, erythema marginatum (the rash in the picture), and subcutaneous nodules. Minor manifestations include fever, arthralgia, elevated ESR and/or CRP, and prolonged PR interval. The patient must also have supporting evidence of antecedent group A streptococcal infection by positive throat culture, rapid antigen detection test, or elevated or increasing streptococcal antibody titer. Increased incidence has occurred in the former Soviet Union.

Which of the following patterns would these neck veins most likely have? A) Large a waves and slow y descents B) Distended neck veins with no pulsatile activity C) Rapid x and y descents D) Large a waves and very large v waves E) Regular cannon a waves

Explanation This person has advanced left and right heart failure from previous myocardial infarctions. Large a and very large v waves would be typical of this condition. Large a waves and slow y descents are associated with tricuspid stenosis. Rapid x and y descents are found in constrictive pericarditis. A cannon a wave occurs with atrio-ventricular dissociation and RA contraction against a closed tricuspid valve (found in ventricular tachycardia). Distended nonpulsatile neck veins would be indicative of superior vena caval obstruction, most likely from cancer.

What systems should be assessed if a pt has a markedly elevated BP?

Eyes: papilledema CNS • Altered mental status or intracranial hemorrhage • Hypertensive encephalopathy may develop (suspect when BP is markedly ele- vated: 240/140 or higher, along with neurologic findings such as confusion). Kidneys: renal failure or hematuria Heart: unstable angina, MI, CHF with pulmonary edema, aortic dissection Lungs: pulmonary edema Kidneys: renal failure or hematuria

What are the clinical features of A. Fib?

Fatigue and Exertional Dyspnea Palpitations, dizziness, angina, or syncope Irregular, irregular pulse Blood stasis leads to formation of intramural thrombi, which can embolize to the brain Absense of an "*A Wave*" in JVP measurement.

What is the major SE of niacin?

Flushing

What is the *MOST COMMON CAUSE* of endocarditis in IVDU?

Frequently presents with right-sided endocarditis S. aureus is the *most common cause* Other organisms include enterococci and streptococci. Fungi (mostly Candida) and gram-negative rods (mostly Pseudomonas) are less common causes.

What are the AE for BAS?

GI effects

What is the *GOLD STD* for diagnosis of myocarditis? What is the tx?

GOLD STANDARD - BX OF MYOCARDIUM Tx: Supportive

What is the management for TR?

Geared more towards *MEDICAL MANAGEMENT* - tx the underlying cause (1) Tx LHF, endocarditis, or pulmonary HTN Severe regurg can be tx surgically is pulm HTN is NOT present - native valve repair surgery, valuloplasty of triscupsid ring, valve replacement surgery (Rare)

What is Rheumatic Fever?

General: Post infectious inflammatory dz Incidence: Common in age 5-15, Lower socioeconomic areas, 2-5 weeks latency Patho: Immune rxn to untreated group A β-hemolytic strep

What are the SE of fibrates?

Gl side effects (mild) Mild abnormalities in LFTs Gynecomastia, gallstones, weight gain, and myopathies are other side effects.

Duke's minor criteria mentions several immunological signs, differentiate the following: (1) Glomerulonephritis (2) Osler's Nodes (3) Janeway Lesions (4) Roth's Spots

Glomerulonephritis - post untx strep infxn - toxins attach the GBM and cause leakages (Sx: RBC Casts) Osler Nodes: small PAINFUL nodules on fingers/toes Janeway Lesions: small hemorrhages on palms and soles - NOT PAINFUL Roth Spots: fundus

General Approach to a pt in shock...

Goal: stabalize pt and determine cause of shock *#1: ABC!!!!* a. Establish two large-bore venous catheters, a central line, and an arterial line. b. A fluid bolus (500 to 1,000 m l of normal saline or lactated Ringer's solution) should be given in most cases. c. Draw blood: CBC, electrolytes, renal function, PT/PTT d. ECG, CXR e. Continuous pulse oximetry f. Vasopressors (dopamine or norepinephrine) may be given if the patient remains hypotensive despite fluids. g. If the diagnosis is still in question after the above tests, a pulmonary artery cath/echo can aid in diagnosis

The End....

Good Luck

Which lipid has a protective effect by removing xs cholesterol from arterial walls and has at least as strong as the atherogenic effect of LDL?

HDL - every 10 mg/dl increase decreases CAD risk by 50% <30 - independent risk factor for CAD >60 - "negative risk factor" (counteracts)

What is the *TOC* for lowering LDL levels?

HMG CoA reductase (Statins) Effects: *LOWER LDL*, min effect on HDL and TG Comment: have been shown to *reduce mortality from CV events significantly and reduce total mortality* SE: monitor LFTs (monthly for 3 mo, then every 3-6 mo); harmless elevation in CPK can occur

Describe the CV exam findings for VSD.

Harsh, blowing *holosystolic* murmur with thrill At fourth left intercostal space (LLSB) Murmur *decreases* with Valsalva and handgrip The smaller the defect, the louder the holosystolic murmur Sternal lift (RV enlargement) As PVR increases, the pulmonary component of 52 increases in intensity Aortic regurgitation may be seen in some patients.

What diagnostic procedures should be utilitzed for USA?

Higher risk for AE w/ Stress test sooo.... 1) Stablize w/ medical management before stress test or 2) undergo cardiac cath initially

What device can be used to detect silent ischemia and silent arrhythmias not assable by an in office EKG?

Holter Monitor

What is *BECKS TRIAD*?

Hypotension, Muffled Heart Sounds, JVD **Caracteristic for Cardiac Tamponade**

What are the grades for murmurs (I-VI)?

I = Very soft, *not immediately* apparent II = Soft, but *immediately audible* III =Loud, but *no palpable thrill* IV =Audible *with a palpable thrill* V =Can be heard with stethoscope *barely touching* the skin, with a thrill VI =Can be heard with stethoscope *not even touching* the skin, with a thrill

In regards to VT as a whole what is the *FIRST LINE THERAPY*?

ICD Amiodarone is considered the *BEST 2ND LINE TX*

As a general rule, what form of management should be given to all patient who are dx w/ sustained VT?

ICD Exception - EF is normal, then you can consider amiodarone

What is the acute management for a RCA infarction?

IV NL Saline + MOA (Morphine, O2, ASA) *NITRO IS C/I IN A RIGHT SIDED MI*

What is the *TOC* for HTN Emergency in the setting of an MI?

IV NTG and BB (esmolol/labetalol)

What is the *TOC* for HTN emergency in the setting of aortic dissection?

IV Nitroprusside and BB (esmolol/labetalol)

What is the *TOC* for HTN emergency?

IV Nitropusside

In refractory V. Fib, what are the ACLS guidelines...

IV amiodarone followed by shock *sodium bicarb is NO longer recommended*

What is the tx for MVP?

If patient is asymptomatic: reassurance If patient has systolic murmur or thickened valve: antibiotic prophylaxis for dental procedures to prevent infective endocarditis For chest pain B-blockers have been useful, but they are unlikely to be required. Surgery rarely is required. The condition is generally benign.

In a hemodynamically *unstable* patient in A. Fib what manaement would you take?

Immediate electrical cardioversion to sinus rhythm

What is pericarditis and what is the MCC?

Inflammation of pericardium MCC: virus/ BQ: Dresslers syndrome

What is myocarditis and what is the etiology?

Inflammation of the myocardium Etiology: viral infection - coxsackieb, echovirus, and adenovirus Mechanism of injury: o INVASION OF MYOCARDIUM: VIRUS** o PRODUCTION OF TOXIN: DIPTHERIA** o AI: SLE/RHEUMATIC FEVER**

What are the sx and signs for constrictive pericarditis?

Initial manifestations are secondary to systemic venous pressure elevation: edema, ascites, hepatic congestion. Later manifestations are due to elevation of left-sided intracardiac pressures: pulmonary congestion-cough, exertional dyspnea, and orthopnea. Signs include: a. Jugular venous distention (JVD)-*most prominent physical finding*; central venous pressure is elevated and displays prominent x and y descents b. *Kussmauls sign*-JVD (venous pressure) fails to decrease during inspiration c. *Pericardia! knock*-corresponding to the abrupt cessation of ventricular filling d. Ascites e. Dependent edema

What is the treatment for aortic dissection?

Initiate medical therapy immediately. a. lV ,B-blockers to lower heart rate and diminish the force of left ventricular ejection b. IV sodium nitroprusside to lower SBP below 120 mm Hg Type A dissections-surgical management Type B dissections-medical management

How to interpret diagnostic tests in a DVT?

Intermediate-to-high pretest probability of DVT • If Doppler ultrasound is positive, begin anticoagulation. • If Doppler ultrasound is nondiagnostic, repeat ultrasound every 2 to 3 days for up to 2 weeks. Low to intermediate probability of DVT • If Doppler ultrasound is negative, there is no need for anticoagulation; obser- vation is sufficient. • Repeat ultrasound in 2 days.

________ __________ = cramping leg pain that is reliably reproducable by the same walking distance (distance is very constant and reproducible) and pain is completely relieved by rest.

Intermittent Claudication

Recent studies show that what device increases survival in pts w/ cardiogenic shock?

Intra-aortic balloon pump (IABP) - placed in descending thoracic aorta for mechanical support (1) decreases afterload (2) increases CO (3) Decreases Myocardial demand

What is the *MOST COMMON CAUSE* of V.Fib?

Ischemic Heart Disease = MCC Other: Prolonged QT (drugs or torsades de pointe) or A.Fib w/ rapid ventricular rate in pts w/ WPW.

What is the often the trigger for cholesterol emboli syndrome and what is the S&S?

It is often triggered by a surgical or radiographic intervention (e.g., arteriogram), or by thrombolytic therapy. It presents with small, discrete areas of tissue ischemia, resulting in blue/black toes, renal insufficiency, ancl!or abdominal pain or bleeding (the latter is due to intestinal

What is very important about the ECHO and CHF?

It measures *EJECTION FRACTION* <40% = systolic dysfunction (*MOST COMMON*) >40% = diastolic dysfunction

What is the diagnostic criteria for Rheumatic Fever?

Jones Criteria: 2 major OR 1 major + 1 minor ("Jonesin for the Fever") *Major Criteria: (J<3NES)* J - Joints (Migratory Polyarthritis), <3 - Cardiac involvement (pericarditis, CHF, MS) N - Nodules (subcutaneous) E - Erythema marginatum, S - Sydenham's chorea *Minor Criteria: (FEAR 1 P)* F - Fever, E - Elevate phase reactants (ESR, CRP), A - Arthralgia (poly) R - Rheumatic Heart Dz PmHx or prev strep infxn 1 - 1st degree heart block P - Prolonged PR interval

What vessel corresponds to a poor prognosis in regards to CAD?

LAD - b/c it covers 2/3 of the heart

For patients that have CHD or CHD risk equivalents, what is the following: (1) LDL goal (2) Initial TLC (3) Consider Drug therapy

LDL goal: <100 Initiate TLC: 100 (*ALL* pts regardless of LDL) Consider drug therapy: >130

For patients that have NO CHD and >2 risk factors, what is the following: (1) LDL goal (2) Initial TLC (3) Consider drug therapy

LDL goal: <130 Initiate TLC: > 130 (*ALL* pt regardless of LDL) Consider Drug Therapy: >130

For patients that have No CHD but 2 risk factors, what is the following: (1) LDL goal (2) Initial TLC (3) Consider drug therapy

LDL goal: <130 Initiate TLC: >130 Consider drug therary: >160

For patients that have NO CHD and 0-1 risk factors, what is the following: (1) LDL goal (2) Initiate TLC (3) Consider Drug therapy

LDL goal: <160 Initiate TLC: >160 Consider drug therapy: >190

Before starting a patient on statins or fibrates, what lab must your order?

LFTs - can induce transient elevation in serum transaminases

What is the *acute tx* for CHF?

LMNOP L - Lasics M - Morphine N - Nitro O - O2 P - Position

What is initiated in all patients with an MI to prevent the progreessio of thrombus, however has NOT been shown to decrease mortality?

LMWH (enoxaparin)

What are the EKG findings for DCM?

LVH, LAE, Q waves, poor R wave progression, A.Fib LVH --> R in V5 (or V6) + S in V1 (or V2) > 35 mm LAE --> Lead II - M or noted appearance, V1 - biphasic

What labs are appropriate to order for Rheumatic heart disease?

Labs: ASO and Anti-DNase B titers, Acute phase reactants

What are the clinical sx for MR?

Left Sided HF Sx Palpitations Pulmonary Edema

What are the sx of left-sided heart failure? What is the MCC of left-sided HF?

Left sided heart failure - dyspnea, orthopnea, paraxoysmal noctural dyspnea, nocturnal cought (nonproductive) Adv CHF - confusion and memory impairment NYHA Class IV - diaphoresis and cool extremeities at rest MCC = CAD/HTN

What is Leriche's syndrome?

Leriche's syndrome: atheromatous occlusian of distal aorta just above bifurcation causing bilateral claudication, impotence, and absent/ diminished femoral pulses

What are the long term and short term goals for treatment in dyslipidemia?

Long term: reduce coronary heart disease Short Term: reduce LDL levels

What are the EKG findings for constrictive pericarditis?

Low QRS voltages, generalized T wave flattening or inversion, left atrial abnormalities Atrial fibrillation occurs in fewer than half of all patients.

What is the MOA for fibrates (gemfibrozil)?

Lowers VLDL and TG, increses HDL Used if Statins, Niacin, and BAS dont work

Although Amiodarone is used to control numerous rhythms (A.Fib,V. Fib, V. Tach), why do most professionals call it "Ami-terrible"?

MAJOR AE = TORADES DE POINTE (B/C OF PROLONGED QT) Called "Amiterrible" b/c it has multiple AE that can affect multiple organs: Cardiac (bradycardia, prolonged QT, torsades); Heptatic toxicity, Thyroid (hyper and hypo); Pulmonary toxicity; Dermatolgic; Neuro; and Opthalmic (corneal deposits and optic neuritis); GI upset

What is the MCC of Restrictive CM?

MCC = Amyloidosis Other common causes: Sarcoidosis, hemochromatosis, scleroderma, carcinoid syndrome, idiopathic Overall is caused by an infiltrating disease

What is the *MOST COMMON CAUSE* of DCM?

MCC = CAD w/ prior MI (from Stepup to Med) and Alcohol (from Ovalle)

What is the *MOST COMMON CAUSES* of MR?

MCC = MVP (according to ovalle)

What is the acute management for a NSTEMI/Non-Q wave MI?

MONA (THROMBOLYTICS ARE C/I) BB/CCB/STENT

What is the initial treatment of an MI upon admission?

MONA - Morphine, Oxygen, Nitro, Aspirin

In a papillary muscle infact, what is the common resulting murmur and what is the management

MR ECHO - Emergeny MV replacement - afterload reduction w/ nitroprusside or intra-aortic balloon pump.

Name the holosystolic murmurs.

MR/TR/VSD

What is the pathophysiology behind MVP?

MVP is defined as the presence of excessive or redundant mitral leaflet tissue due to myxomatous degeneration of mitral valve leaflets and/or chordae tendineae. The redundant leaflet(s) prolapse toward the left atrium in systole, which results in the auscultated click and murmur.

If cardioverion is sucessful in V.Fib, what is your following management options for this patient.

Maintain continuous IV infusion of the effective antiarrhythmic agent. IV amiodarone has been shown to be the most effective. Implantable defibrillators have become the *MAINSTAY OF CHRONIC THERAPY* in patients at continued risk for VF. Long-term amiodarone therapy is an alternative.

What patient population is is common to find MVP?

Marfan's syndrome, osteogenesis imperfecta, and Ehlers-Danlos syndrome. Presentation: "Tall, slender, young pt, most are ax but can have CP, angina, palpitations, syncope"

What methods are used for prophylaxis post-surgery?

Mechanical • Leg elevation, graduated compression stockings, early ambulation • Pneumatic compression boots-intermittently inflate and deflate, causing compression of the limb, usually the calves; very effective Pharmacologic • Heparin or LMWH: Unfractionated heparin or LMWH postoperatively until patient is ambulatory • Combination of pneumatic compression devices and pharmacologic prophylaxis may provide the greatest protection

Venous ulcers are *MOST COMMONLY* located where?

Medially from the instep of the foot to above the ankle, overlying the incompetent perforator vein

What are the surgical and medical management options for MS?

Medical a. Diuretics-for pulmonary congestion and edema b. Infective endocarditis prophylaxis c. Chronic anticoagulation with warfarin is indicated (especially if patient has AFib) Surgical (for severe disease) a. *Percutaneous balloon valvuloplasty* usually produces excellent results. b. Open commissurotomy and mitral valve replacement are other options if valvotomy is contraindicated.

What is the tx for truncus arteriosus?

Medical: digitalis/diuretics; Surgical: Remove the PA from the TA, Replace truncal valve, Close VSD if was open

What is the *MCC* of neoplasms of the heart?

Mets from other primary tumors - lung, breast, skin, kidney, lymphoma, Karposi sarcoma (AIDS)

If a patient presents superficial thrombophlebitis that occurs in different locations over a short periods of time, what condition should you be thinking of?

Migratory Superifical Thrombophlebitis - secondary to occult malignancy (*MC* the pancreas)

What is the initial LT treatment for mild CHF (I/II)?

Mild restriction of sodium intake (<4 g Na+) Physical activity Loop Diuretic ACEi Loop and ACEi are the *FIRST LINE TREATMENTS*

What is the *MOST COMMON* associated vavular abnormality associated w/ Rheumatic Fever?

Mitral Stenosis

What two forms of heart block require a pacemaker?

Mobitz II (second degree heart block) 3rd degree heart block (complete)

Describe the clinical features of MVP?

Most are *ASX* - palpitations and atypical CP can occur and TIAs from MV emboli (VERY RARE)

What are the sx of hyperlipidemia?

Most are Asx Severe: (1) Xanthelasma - yellow plaques on eyelids (2) Xanthoma - hard, yellowish masses found on tendons (finger extensors, achilles tendon, plantar tendon) (3) Pancreatitis w/ severe TG elevation

Describe the patho behind V. Fib?

Multiple foci in the ventricles fire rapidly, leading to chaotic quivering of the ventricles and NO CO. Reoccurance is high if *unrelate* to an MI and has a worse prognosis --> Pt will need a ICD/Amiodarone Fatal if untreated can result in "SUDDEN CARDIAC DEATH"

What are the S&S of Dig toxicity?

N/V Anorexia Cardiac: ectopic ventricular beats, AV block, Afib CNS: green/yellow vision, disorientation

What is the tx for superfiical thrombophlebitis?

NO anticoagulants needs Localized - Analgesic and activity Severe (w/ pain and cellultiis) - bed rest, elevation, hot compression. When sx resolve ambulation w/ elastic stockings. Abx are *NOT* neeed unless supporitive and I&D is needed.

In cardiac tamponade, what findings can be found for their pulse?

Narrow pulse pressure (due to decrease SV) *PULSUS PARADOXUS (PATHOGNEUOMIC)* - exaggerated decrease in artierial pressure during inspiration (>10 mm Hg drop); can be detected by a decrease in the amplitude of the femoral/carotid pulse during inspiration - pulse gets strong during expiration and weak during inspiration

What is the management option for hemodynamically stable sustained VT pts w/ SBP of > 90?

New ACLS says IV Amiodarone, IV procaimamide, or IV sotaolol

What is the *TOC* for the TG>500 mg/dl?

Niacin is the *first line* drug for hypertriglyceridemia 2nd line: Gemfibrozil

What management can be used for prinzmetal's angina?

Nitrates, CCB

what are the possible clinical findings for myocarditis?

Non-specific viral syndrome - pleuritic chest pain Dx: tachy, friction rub, S3 gallop, MR/TR, nonspecific T-wave changes, elevated troponin I and CK-MB w/ myocardial necrosis

What is the normal ejection fraction? What is the ejection fraction value that corresponds to increased mortality?

Normal = > 50% <50% is associated w/ mortality

What are the CXR findings for tricupsid atresia?

Normal size, "Boot-shaped", Concave ht border; ↓ pulm vasculature.

What is the *MOST COMMON* age to see still's murmur?

Occurrence: 3-6 y.o.; occasionally in infancy

What is the *MOST COMMON* age to see a pulmonary ejection murmur?

Occurrence: 8-14 y.o. (school age) "Think - school aged kid getting "ejected" from school"

What is the *MOST COMMON* type of ASD?

Ostium Secundum (*MC*) - occurs in the CENTRAL portion of the interarterial septum Osteum primum - occurs LOW in the septum Sinus venosus defects - occurs HIGH in the septum

Name the continuous murmurs.

PDA & Venous hum

What is the classic symtpoms for superficial thrombophlebitis?

Pain, tenderness, induration, and erythema along the course of the vein AND a *tender palapable cord*

What are the signs of MS?

Parasternal life Murmor: • The *opening snap* is followed by a low-pitched diastolic rumble and presys- tolic accentuation. This murmur increases in length as the disease worsens. • Heard best with bell of stethoscope in left lateral decubitus position

What is the cause of *eisenmenger's syndrome* and what are the sx?

Patho: Complication of uncorrected congenital ht. anomalies produces L-to-R shunting, Increased pulm resist. over time reverses shunt to a R-to-L (causing Pulm HTN), Deoxygenated blood enters systemic circ → hypoxia S&S: Cyanosis, Clubbing, Polycythemia

What is the patho and S&S of hypoplastic left heart syndrome?

Patho: Hypoplasia and underdevelopment of LV S&S: MV/AV are hypoplastic, Discrete coarc or aorta present, Peripheral pulses weak/absent

What is the patho underlying VSD?

Patho: L-to-R shunt, 2x blood in LA/LV, RV hypertrophy due to ↑ pulm resist and ↓ blood flow in lungs

What is the pathophysiology behind PDA?

Patho: Pulm resistance ↓ → L-to-R shunt, ↑ pulm blood flow → ↑ workload on L side of ht and ↑ venous return to LA

Name the EKG findings that are characteristic of VT.

Pathogneuomic = *WIDE AND BIZARRE QRS COMPLEXES* QRS can be monomorphic or polymorphic *MONOMORPHIC* - all QRS complexes are *identical* *POLYMORPHIC* - the QRS complexes are *different* - beat to beat variation

Name the causes of cardiac tamponade.

Penetrating (less commonly blunt) trauma to the thorax, such as gunshot and stab wound Iatrogenic: central line placement, pacemaker insertion, pericardiocentesis, etc Pericarditis Post MI w/ free wall rupture

________is defined as any cause of acute pericarditis that can lead to exudation of fluid into the pericardial space - can occur in associateon w/ ascites and pleural effusion in salt and water retention states such as CHF, cirrhosis, and nephrotic syndrome.

Pericardial effusion

What are the S&S of right sided heart failure? What is the MCC?

Peripheral Pitting Edema (secondary to venous insuff) Nocturia (due to increased venous return from leg elevation) MCC = Left sided HF or Cor pulmonale (pure) JVD Hepatomegaly/Hepatojugular reflex Ascites Right ventricular heave

If a patient is unable to exercise, what procedure can you use as a screening test for CAD?

Pharmacological Stress Test w/ IV adenosine, dipyramidole, dobutamine

Pt presents with HTN that is not resolved by TLC or pharmacological therapy...what 2 conditions should ALWAYS be on your differential diagnosis?

Pheochomocytoma Renal Artery Stenosis

MI is associated with 30% mortality, 1/2 of the deaths are (pre/post) hospital?

Pre

Before instituting drug therapy for HTN in a female what must you *ALWAYS* do?

Pregnancy Test Thiazides, ACEi, CCB, & ARBs are *CONTRAINDICATED* in pregnancy BB and hydralazine are safe!

What is the treatment for hypoplastic left heart syndrome?

Prostaglandin, Diuretics, Surgery, *Cardiac Transplantation*

What is the treatment for COA?

Prostaglandins → reverses closing of PDA temporarily, Surgery or cath techniques provide *definitive* repair

What clinical couse does ASD follow?

Pts are usually asx until middle age (~40 y/o) >40 - sx may include DOE, fatigue, and exercise intolerance Mild = normal lifespan

what is the pathphysiology underlying tetrology of fallot?

Pulm stenosis ↓ blood to lungs/amount of blood to L ht, R-to-L blood thru VSD allows deoxygenated blood to mix w/ oxygenated blood → hypoxemia, If ductus arteriosus remains open then pulm blood flow is adequate

If the diagnosis is unclear from the pts VS and clinical presentation, how can you diagnose hypovolemic shock?

Pulmonary Artery Cath

Based on the listed sx, what murmur is being described: Systolic Turbulence of left or right pulmonary artery branch Medium pitch Loudest: RUSB/LUSB Transmits well to left and right chest, axillae & back A. Still's Murmur B. Pulmonary ejection murmur C. Pulmonary flow murmur D. Venous hum E. Carotid Bruit

Pulmonary Flow murmur

What is the *MOST COMMON CAUSE* of inpatient mortality post-MI?

Pump Failure (CHF) Mild: tx w/ ACEi and Diuretic Severe: can lead to cardiogenic shock - hemodynamic monitoring

What can be used to precisely measure the EF and left ventricular function?

Radionucleotide ventriculography using technetium-99m

What is more important for cardiac tampondae - Rate or Amount?

Rate 200 mL of fluid that develops rapidly can cause cardiac tamponade 2L of fluid may accumulate slowly before cardiac tamponade occurs. When fluid accumulates slowly, the pericardium has the opportunity to stretch and adapt to the increased volume

In a hemodynamically stable patient, what management steps should you take?

Rate Control w/ CCB Duration of A.Fib <48 hours --> Cardioversion Duration of A.Fib > 48 hours --> Anticoagulate x 3 weeks w/ Warfarin (INR 2-3) then cardiovert or Duration of A.Fib > 48 hours --> get a TEE --> If thrombosis in left atrium then anticoagulate x 3 weeks and then cardiovert; If no thrombis in left atrium then cardiovert immediatly.

What is the *MOST COMMON CAUSE* of MS?

Rheumatic Heart Disease Rheumatic heart disease causes an immune mediated damage to the MV which leads to scarring and narrowing of the MV orifice - additionally anything that increases flow across the MV exacerbates pulmonary venous HTN and ass sx

Describe the pathophys behind Restrictive CM.

Rigid and non-complient ventricular walls w/ normal systolic fnx, ventricle cavities are small, *diastolic dysfunction*, elevated ventricular filling pressure, LEAST COMMON

What are the EKG findings for "Torsades de Pointe"?

Rotation of the heart's electrical axis by at least 180° Prolonged QT interval (LQTS) Preceded by long and short RR-intervals Triggered by an early premature ventricular contraction (R-on-T PVC)

What are the S&S, EKG, CXR findings for pulmonary stenosis?

S&S: Asx, Acyantoic, SEM loudest on expiration at ULSB + ejection click, S2 split, Seen in *Noonan syndrome* EKG: RAD, RAH, RVH CXR: Normal heart size, Post-stenotic dilitation of pulmonary artery

What is the *MOST COMMON CAUSE* of native valve endocarditis?

S. Viridians is the *MOST COMMON*organism in native endocarditis Other common organisms = S. Epidermidis, and enterococci *HACEK* group of organisms - Haemophillus, Actinobacillus, Cardiobacterim, Eikenella, Kingella

Explain the systemic inflammatory response syndrome (SIRS) and its progression.

SIRS characterized by *two or more* of the following: • Fever(>38°C) or hypothermia(<36°C) •Hyperventilation (rate>20bpm) or Paco2 <32 mmHg • Tachycardia (> 90 bpm) • Increased WBC count (>12,000cell/hpf <4,000cells/hp for >10% band forms) Sepsis • When blood cultures are positive and SIRS is present • Blood cultures: Obtain two sets from two different sites (each set should have aerobic and anaerobic bottles). *Draw blood before antibiotic administration* Septic Shock • Hypotension induced by sepsis persisting despite adequate fluid resuscitation Multiple Organ Dysfunction Syndrome (MODS) • Altered organ function in an acutely ill patient usually leading to death

(NSTEMI/STEMI) is larger and more devestating to the patient's health?

STEMI

What is the *MOST COMMON CAUSE* of death in the ICU?

Septic Shock

What are the S&S of HTN emergency?

Severe HA Visual Disturbances Alt Mentation

What are the clinical features of TR?

Signs and symptoms of *RVF* (ascites, hepatomegaly, edema,JVD) *Pulsatile liver* Prominent *V waves* in jugular venous pulse with rapid y descent *Inspiratory S3* along LLSB may be present *Blowing holosystolic murmur at LLSB, Intensified with inspiration; reduced during expiration or the Valsalva maneuver* Right ventricular pulsation along LLSB AFib is usually present.

70 y/o pt presents with sx of dizziness, confusion, syncope, fatigue, and CHF. You order an EKG and it reveals sinus node dysfunction characterized by a persistant spontaneous sinus bradycardia. What is the most likely diagnosis? Management?

Sinus node dysfunction characterized by a persistent spontaneous sinus bradycardia Symptoms include dizziness, confusion, syncope, fatigue, and CHF. Tx: Pacemaker implantation may be required.

Define Sinus Bradycardia.

Sinus rate <60 bpm: clinically significant when rate is persistently <45 bpm Causes include ischemia, increased vagal tone, antiarrhythmic drugs; may be a normal finding in trained athletes. Can be asymptomatic; patients may complain of fatigue, inability to exercise, angina, or syncope Atropine can elevate the sinus rate by blocking vagal stimulation to the sinoatrial node. A cardiac pacemaker may be required if bradycardia persists.

Describe the skin changes that are associated w/ chronic venous insuff.

Skin becomes thin, atrophic, shiny, and cyanotic Brawny induration develops w/ chronicity

What is the treatment for nonhemorrhagic tamponade?

Stable - monitor w/ EKG, ECHO, CXR Non-stable: *PERICARDIOCENTESIS* - if no improvement then can do fluid challenge

The pathophysiology of stable vs unstable angina?

Stable: due to increased demand Unstable: reduced resting coronary flow

What is the *MOST COMMON CAUSE* of prosthetic valve endocarditis?

Staphylococci are the *most common causes* of early-onset endocarditis; symptoms appear within 60 days of surgery (S. epidermidis *more commonly* than S. aureus). Streptococci are the *most common cause* of late-onset endocarditis; symptoms appear 60 days after surgery.

What is the initial LT treatment for mild-mod CHF (II/III)?

Start w/ a loop diuretic and ACEi Add a Beta Blocker if moderate disease is present.

Pt presents with the following sx, what is the murmur? Systolic Low-frequency Twanging string Groaning, squeaking, or musical Vibratory/musical Loudest: LLSB and apex or MLSB

Still's Murmur Grade 2-3/6 in intensity No referral to cardiologist is needed

Name the innocent heart murmurs.

Still's Murmur Pulmonary ejection murmur Pulmonary flow murmur Venous hum Carotid Bruit

What is the *MOST COMMON CAUSE* for subacute endocarditis?

Strep Viridians (MCC) Other: Enterococci

Pts w/ repetitive PVCs are at increased risk for what?

Sudden Death by V. Fib (Esp when >10/hour)

What are the tx options fo eisenmengers?

Supportive, Heart/lung transplant, Endocarditis prophylaxis

What is the treatment for atrial myxoma?

Surgical Excision

What is the tx for ASD?

Surgical repair when pulmonary-to-systemic blood flow ratio is greater than 1.5:1 or 2:1 or if patient is symptomatic.

Define *sustained V.Tach*. What are the associated risks?

Sustained VT lasts *longer than 30 secondas and is almost always symptomatic*! Ass w/ MI/Hypotsn Can be a life-treatening arrhythmia and can lead to V. Fib :0

What are the S&S of HCM?

Sx: DOE, CP, Syncope (post exertion/valsalva), palpitations, arrhythmias (AFib, ventricular), cardiac failure due to increased diastolic stiffness. *SUDDEN DEATH IN A YOUNG ATHLETE CAN BE THE FIRST SX OF THE DISEASE :(* Signs: Sustained PMI Loud S4 (corresponds to diastolic issue - remember diastole is 4 syllables) *Systolic Ejection Murmur - best heard at the left sternal border - increased w/ valsalva/standing/post-PVC and decreased w/ squatting/BB* Rapidly increasing carotid pulse w/ 2 upstrokes (*bisferious pulse*)

What are the S&S of RCM?

Sx: Fatigue, dyspnea, *RIGHT SIDED HEART FAILURE* sx - leg edema/hepatomegaly, ascites, distended neck veins Signs: S3/S4 and A.Fib

What are the pathognomic S&S for pericarditis?

Sx: PLEURITIC CP THAT IS RELIEVED BY SITTING UP AND LEARNING FORWARD - "POSITIONAL" PE: PATHONGMONIC - FRICTION RUN - SCRATCHY, CREAKY LEATHER (Although not always present)

What are the S&S of AR?

Symptoms a. Dyspnea on exertion, PND, orthopnea b. Palpitations c. Angina d. Cyanosis and shock in acute aortic regurgitation (medical emergency) Physical examination a. *WIDENED PULSE PRESSURE*-markedly increased systolic BP, with decreased diastolic BP b. Diastolic decrescendo murmur best heard at left sternal border c. *Corrigan's pulse* (water-hammer pulse)-rapidly increasing pulse that collapses suddenly as arterial pressure decreases rapidly in late systole and diastole; can be palpated at wrist or femoral arteries d. *Austin-Flint murmur*-low-pitched diastolic rumble due to narrowing of mitral valve orifice by aortic regurgitation, resulting in relative mitral stenosis e. Displaced PMI (down and to the left) and S3 may also be present.

In pts with chronic venous insuff, symptoms of leg swelling are worsened by periods of (rest/inactivity)?

Symptoms are worsened by periods of sitting or inactive standing

What are the clinical features of DCM?

Systolic Heart Failure Left Sided Heart Failure sx S3 Gallop (Sytolic Dysfunction <-- 3 syllables in systole) MR/TR

What is the *SOC* for endocarditis?

TEE - Tranesophageal (NOT transthoracic) ECHO!

What are the perferred tests for aortic dissection?

TEE and CT Transesophageal echocardiogram (TEE) has a very high sensitivity and specificity; ~ it is noninvasive and *can be performed at the bedside* (good in the unstable pt) CT scan and MRI are both highly accurate (MRI more so); MRI takes longer to perform, making it less ideal in the acute setting.

What is the *SOC* for diagnosis of ASD?

TEE is Diagnostic!

What is the *INITIAL TOC* for dyslipidemia?

TLC (1) Diet - high in omega-3s; <30% of total calories from dat, <10% from sat fat, <300 mg/dl of chol (2) Exercise: increase HDL and decraese CHD risk

What is a nonpharmacological treatment that can be used for the tx of CAD?

TLC - EXERCISE is crucial and DIET modification (decrease fat, cholesterol)

What pharmacological therapies are first line for all CAD patients?

TLC, Aspirin, BB, Nitrates (2nd line: add CCB)

What is the *TOC* and second line tx for rheumatic heart disease? What is the prophylaxis?

TOC = PCN (If allergic Azithro/Erythromycin) ARF requires NSAIDS and C-reactive protein monitoring Always prophylax for dental/GI/GU procedures w/ Amoxicillin or erythromycin Tx valvular pathology if present.

What is the *INITIAL TOC* for HTN?

TOC = Thiazides (HCTZ) 2nd line diuretic = Loop

What are the causes for TR?

TR is usually secondary to RV dilatation. Any cause of RV dilatation can result in enlargement of the tricuspid orifice. • Left ventricular failure is the *MOST COMMON CAUSE* (according to *STEP UP*). • Right ventricular infarction • Inferior wall MI • Cor pulmonale, secondary to pulmonary HTN b. Tricuspid endocarditis-seen in IV drug users (staph aureus) (*MOST COMMON CAUSE* according to Ovalle) c. May be secondary to rheumatic heart disease; usually accompanied by mitral and aortic valve disease d. Epstein's anomaly-congenital malformation of tricuspid valve in which there is downward displacement of the valve into the RV e. Other causes include carcinoid syndrome, SLE, and myxomatous valve degeneration.

What is the treatment for pulmonary stenosis?

TX: Mild=no tx, Mod-severe=balloon dilation via cardiac cath

What is the tx for pericarditis?

TX: SUPPORTIVE - AVOID ANTICOAGS NSAIDS are the *MAINSTAY* to treatment (Glucocorticoids if non-responsive to NSAIDS)

Pt with tetrology of fallot will have classically turn cyanotic when crying, what is the phenomenon referred to as?

Tet spells (hypoxic episodes)

What determines the mortality and morbitdity rate in hypovolemic shock?

The *rate of volume loss* - the slower the volume loss the better the prognosis b/c the compensatory mechanisms can respond.

What will an ECHO show for RCM?

Thickened myocardium Increased LA/RA size with normal LV/RV size *Amylodosis - myocardium will be brighter and have a sparkling appearance*

What cyanotic CHD is associated w/ the following pathophys and sx Patho: Common pulm veins don't form into post. wall of LA but in other places, PV most commonly arises from SVC, Oxygenated blood mixes w/ deoxygenated blood in R side of ht S&S: Cyanosis, SEM at LLSB, Diastolic murmur, split S2?

Total Anomalous Pulmonary Venous Connection (TAPVC) (cyanotic)

Name the components of *VIRCHOW'S TRIAD*:

Trauma (endothelial injury) Stasis (venous) hypercoagulability

What is the management for cholesterol embolization syndrome?

Treatment is supportive. *Do not anticoagulate*. Control BP. Amputation or surgical resection is only needed in extreme cases

What is the *TRIAD* associated w/ AAA rupture?

Triad of *abdominal pain, hypotension, and pulsatile abdominal mass*

What is the*MOST COMMON* age to find a carotid bruit?

Trick question - it can occur at any age

What CHD is assocaiated w/ following clinical signs: Cyanosis, Systolic murmur, Absent tricuspid valve, ASD/PDA/VSD?

Triscupsid Atresia

What is the *MOST IMPORTANT* cardiac enzyme?

Troponin (I&T) <-- greater sensitivity and specificity than CK-MB Increase w/in 3-5 hours - peak at 24-48 hours - return to nl at 5-14 days Obtain levels on admission - every 8 hours for 24 hours

Troponin (I/T) is falsely elevated in patients w/ renal failure?

Troponin I

What cyanotic CHD is described as follows: Single arterial trunk arises from normal ventricles, Pulm arteries originate from common arterial trunk, CHF develops due to pulm overload. S&S: Cyanosis, Pansystolic murmur at LLSB?

Truncus Arteriosus

What is the management for DCM?

Tx acutely like CHF --> LMNOP - lasics, morphine, nitro, O2, position LT: ACEi, BB, Diuretics, Chronic Anticoag (increased risk for emboli), AVOID alcohol, transplant (last resort)

What is the management for RCM?

Tx the underlying disorder, prevent arrhythmia, control HR, tx sx of HF Hemochromatosis - phlebotomy/deferoxamine Sarcodosis - glucocorticoids Amyloidosis - no tx (avoid dig b/c increased dig toxicity)

What is the treatment for PDA?

Tx: Indomethacin/Ibuprofen, Ligation

The daily sanford classification has type A and type B - explain each.

Type A: *proximal* - involves the *ascending aorta* (includes retrograde extension from descending aorta) - *MOST COMMON* Type B: *distal* - limited to the descending aorta

what labs do you need to order in a pt with a potential dx of HTN?

UA, Chem Panel (K+, BUN/Cr), Fasting glucose, lipid panal, ECG

In superficial thromboplebitits, what is the *MOST COMMON* location in the UE and LE?

UE: *MC* in the site of IV infusion LE: *MC* in varicose veins (great *saphenous* system) secondary to static blood flow NOTE: Virchow Triad applies to thrombophlebitis!

How can you differentiate VT from PVST?

Unlike PVST, VT *DOES NOT* respond to vagal maneuvars or adenosine

Explain the CHADS score for A. Fib?

Used to assess patients risk and when to anticoagulate w/ "Warfarin" verus "electric shock" DO NOT ANTIVERT PATIENTS WITH HIGH RISK! (CHAD SCORE >/= 2+ ALWAYS ANTICOAGULATE CHADS 2+ PATIENT Criteria for CHADS Score - BQ *C*ongestive Heart Failure (1 point) *H*ypertension (1 point) *A*ge over 75 years (1 point) *D*iabetes Mellitus (1 point) *S*troke or TIA history (2 points) Mitral Stenosis or prosthetic heart valve carry similar risk and also indicate Warfarin Interpretation CHADS Score >2 (CVA risk >5% per year): Warfarin with goal INR 2.0 to 3.0 CHADS Score 1 (CVA risk >4% per year): Warfarin or Aspirin CHADS Score 0: Aspirin 81 to 325 mg daily **NEW ONSET A.FIB W/ NO PRIOR DOCUMENTATION - CARDIOVERT***

In the case of a MI w/ sudden cardiac death, what is the *MOST COMMON CAUSE*?

V. Fib

What is the *MOST COMMON* CHD?

VSD

Define *non-sustained VT*.

VT that lasts *less than 30 seconds* - brief, asx Worse prognosis when CAD/LV dysfunction are present.

What is the treatment for AVRT?

Vagal Maneuvars IV Adenosine = TOC Prevention: DIG = TOC Recurrent/Symptomatic = radiofreqencu catheter abalation of either the AV node or accessory tract

What is the *MOST ACCURATE* diagnosis of DVT for the calf veins?

Venography • Invasive and infrequently used • Allows visualization of the deep and superficial venous systems, and allows assessment of patency and valvular competence

Pt presents with an ulcer that is *NOT* very painful (3/10) and located just above the medial malleolus and has rapidly recurred for the 3rd time in 6 mo. Is this ulcer, most likely venous or arterial?

Venous ulcer Classic presentation (1) less pain than arterial (2) located over medial malleolus (3) rapidly recur *Ulcer formation is directly proportional to the amount of swelling present*

What are the EKG findings for PVCs? Tx?

Wide, Bizzare QRS complexes followed by compensatory pause are seen; a P-wave is not usually seen because it is buried within the wide QRS complex. Increased Incidence w/ Age >10 PVC/hour --> Pathologic Exercise should improve PVCs, if not then pathologic Tx: Symptomatic - BB

What are the MCC for AS?

Young Pt: Bicuspid instead of tricuspid Aortic Valve Elderly pt: calcifications of the aortic valve

What are the risk factors for a DVT?

a. Age>60 b. Malignancy c.Prior history of DVT, PE, or varicose veins d. Hereditary hypercoagulable states (factor V Leiden, protein C and S deficiency, antithrombin III deficiency) e. Prolonged immobilization or bed rest f. Cardiac disease, especially CHF g. Obesity h. Major surgery, especially surgery of the pelvis (orthopedic procedures) i. Major trauma j. Pregnancy, estrogen use

What can pulse volume recordings be used to look at in PVD?

a. Excellent assessment of segmental limb perfusion b. Pulse wave forms represent the volume of blood per heart beat at sequential sites down the leg. c. A large wave form indicates good collateral blood flow. d. Noninvasive using pressure cuffs

What is the *MOST COMMON* location for cholesterol emboli to come from?

abdominal aorta, iliac, and femoral arteries

What is "postphbleitic syndrome"?

another name for *chronic venous insuff* - venous stasis disease

For any level of BP elevation, the descision to start medical therpay is dependent on what?

based on the pts total cardiovascular risk and not just the elevation of BP - estimation of the overall risk depends on CV and clinical risk factors, AKA "*The BIG 6 - FSH DAD*"

Aortoilliac occlusive disease cuases _____ and _____ claudication (in addition to the calves)

buttox and hip

What are the CXR for truncus arteriosus?

cardiomegaly and increased pulm vasculature

Septic shock is defined by what?

defined as hypotension induced by sepsis that persists despite adequate fluid resuscitation. This results in hypoperfusion and can ultimately lead to multiple organ system failure and death.

Pt presents with a new heart mumur and an unexplained fever, what should ALWAYS be in your ddx?

endocarditis

What is seen on a CXR for cardiac tamponade?

enlargement of cardiac silhouette when > 250 mL has accumulated - clear lung feilds

What is the phenotypic presentation of a pt w/ atrial myxoma?

fatigue, fever, syncope, palpitations, malaise, and a *low- pitched diastolic murmur that changes character with changing body positions (diastolic plop)*

What is the *MOST COMMON* source of emboli that cause acute arterial occlusion?

heart Other locations: Aneurysms and atheromatous plaque

What are the components of metabolic syndrome?

hypercholesterolemia hypertriglyceridemia impaired glucose tolerance diabetes hyperuricemia HTN

What is the most important and modifiable risk factors for CAD?

hyperlipidemia

What are the classical symtoms for all patients in shock?

hypotension oliguria tachycardia alt mental status

What diagnostic techniques can be used for acute arterial occlusion?

l. Arteriogram to define site of occlusion 2. ECG to look for MI, AFib 3. Echocardiogram for evaluation of valves, clot, MI

What is the treatment for septic shock?

l. IV antibiotics (broad-spectrum) at maximum dosages 2. Surgical drainage if necessary 3. Fluid administration to increase mean BP 4. Vasopressors may be used if hypotension persists despite aggressive IV fluid resus- citation . a. Dopamine is typically the *INITAL TOC* b. If dopamine does not increase the BP, norepinephrine may be given.

What is the diagnostic procedure for hyperlipidemia?

l. Lipid screening -Measure total cholesterol and HDL levels (nonfasting is acceptable). If either is abnormal, then order a full fasting lipid profile. 2. A full fasting lipid profile includes TG levels and calculation of LDL levels. 3. Consider checking laboratory tests to exclude secondary causes of hyperlipidemia. a. TSH (hypothyroidism) b. LFTs (chronic liver disease) c. BUN and Cr, urinary proteins (nephrotic syndrome) d. Glucose levels (diabetes)

Describe the findings on CV exam for ASD.

l. Mild systolic ejection murmur at pulmonary area secondary (ULSB) to increased pulmonary blood flow 2. Wide, fixed splitting of S2 3. Diastolic flow "rumble" murmur across tricuspid valve area (LLSB) secondary to increased blood flow 4. In advanced disease, signs of RVF may be seen.

What are the causes for HTN Emergency?

l. Noncompliance with antihypertensive therapy 2. Cushing's syndrome 3. Drugs such as cocaine, LSD, methamphetamines 4. Hyperaldosteronism 5. Eclampsia 6. Vasculitis 7. Alcohol withdrawal 8. Pheochromocytoma 9. Noncompliance with dialysis

What are the clinical features of VT?

l. Palpitations, dyspnea, lightheadedness, angina, impaired consciousness (syncope or near-syncope) 2. May present with sudden cardiac death 3. Signs of cardiogenic shock may be present. 4. May be asymptomatic if rate is slow 5. Physical findings include *cannon a waves* in the neck (secondary to AV dissociation, which results in atrial contraction during ventricular contraction) and an Sl that varies in intensity.

What are the complications of ASD?

l. Pulmonary HTN-does not occur before 20 years of age, but is a common finding in patients over 40 2. *Eisenmenger's disease* is a late complication seen in a minority of patients, in which irreversible pulmonary HTN leads to reversal of shunt, heart failure, and cyanosis . 3. Right heart failure 4. Atrial arrhythmias, especially *AFib* 5. Stroke can result from paradoxical emboli or AFib.

What are the clinical features of neurogenic shock?

l. Warm, well-perfused skin 2. Urine output low or normal 3. Bradycardia and hypotension (but tachycardia can occur) 4. Cardiac output normal, SVR low, PCWP low to normal

What is the treatment for neurogenic shock?

l. judicious use of IV fluids as the *MAINSTAY* 2. Vasoconstrictors to restore venous tone, but cautiously 3. Supine or Trendelenburg position 4. Maintain body temperature.

Do you want your total cholesterol:HDL ratio to be higher or lower?

lower Ratio of 5.0 is avg (<4.5 is ideal) Ratio of 10.0 is double the risk Ratio of 20.0 is triple the risk

What is the MOA for BAS?

lowers LDL; *INCREASES* TG!!! Effective when used in combination w/ statins or niacin to tx severe disease in high risk pts - third line agent for lowering LDL

What are the clinical featues of pericardial effusion?

muffled heart sounds soft PMI Dullness at left lung base +/- pericardial friction rub

What are the indication for surgical treatment and what are the surgical options?

ndications: rest pain, ischemic ulcerations (tissue necrosis), severe symptoms refractory to conservative treatment that affects quality of life or work Options Surgical bypass grafting-this is the *most common procedure* and has a 5-year patency rate of 70% (immediate success rate is 80% to 90%). Angioplasty-balloon dilatation

Explain the presentation for a cocaine induced MI? What is the *MOST COMMON CAUSE* of death?

o Tachycardia, HTN, vascular constriction o Sudden death due to *V.Fib*

What agents can be used in less severe emergencies?

oral agents - clonidine, captopril, and nifedipine.

What are the common cuases of septic shock?

pneumonia, pyelonephritis, meningitis, abscess formation, cholangitis, cellulitis, and peritonitis (however it is not limited to the ones listed)

Long standing MS can result in what complications?

pulmonary HTN, RVH, and A.Fib

PVD can be diagnosed by the *Ankle-to-Brachial Index (ABI)*....what does the ABI measure?

ratio of SBP at the ankle to the SBP in the arm Normal ABI: =/> 1 Claudication ABI: <0.7 Rest pain ABI: <0.4 False ABI can be seen in DM

How can a physician know if a patient is having a recurrent infaraction?

repeat *ST-Elevation* on EKG post 24 hours of the MI

Cardiac cath will show what in cardiac tamponade?

show equalization of pressures in all chambers of the heart shows elevated right atrial pressure w/ loss of y descent

What is management based on for AAA?

size

A 67-year-old woman presents with 4 hours of substernal chest pain. An electrocardiogram shows ST-segment elevation in leads V1-V5. She has no contraindications to fibrinolytic therapy. The hospital she presents to does not offer PCI, and it is anticipated that primary PCI cannot be performed within 120 minutes of first medical contact. You plan to give her fibrinolytic therapy. Which of the following statements is true? A) Streptokinase plus heparin has been shown to be better at reestablishing coronary blood flow than fibrin-specific fibrinolytic agents. B) Fibrin-specific fibrinolytic agents are preferred over streptokinase (non-fibrin-specific) when available. C) An absolute contraindication to thrombolytic agents is menses. D) Subcutaneous heparin has been shown to be just as effective in decreasing mortality as intravenous heparin. E) You should treat her differently if she presented instead with her symptoms and a new LBBB on her electrocardiogram.

specific) when available. Explanation Fibrin-specific agents are preferred when available. They include: tenecteplase (TNK-tPA) single IV weight-based bolus, reteplase (rPA) 10-U IV boluses given 30 minutes apart, and alteplase (tPA) 90-minute weight-based infusion. Non-fibrin-specific fibrinolytic agents include: streptokinase 1.5 million units IV given over 30-60 minutes. Streptokinase is no longer marketed in the U.S., but is available in other countries. Streptokinase is highly antigenic and absolutely contraindicated within 6 months of previous exposure because of the potential for serious allergic reaction. One of the contraindications to thrombolytic agents is active bleeding or bleeding diathesis (excluding menses); i.e., you can and should treat a woman presenting with an acute STEMI/new LBBB with thrombolytics if PCI is not available, and they have no other contraindications.

In patient with Class IV sx who are still symptomatic despite the previously mentioned treatment, what can you add?

spironolactone

If a pt has a rupture AAA what treatment is indicated?

surgery - all of these pts are unstable

Which murmurs are most commonly benign...sytolic/diastolic?

systolic

Pts with A. Fib are at increased risk for what?

thromboembolism and hemodynamic compromise

What is the *HALLMARK* of prinzmetal's angina?

transient *S-T ELEVATION* on ECG during chest pain, which respresents transmural ischemia

What can you monitor for the successfulness of tx in cardiogenic shock?

urine output

In what patient population is hypothermia seen in septic shock?

very young, elderly, debilitated, and immunocompromised

What is the *TOC* for TG < 500 mg/dl?

weight loss, diet, exercise (wt loss is the *primary goal*)

HCM is *MOST COMMON* in what patient population?

young atheletes

What effect does HTN have on the kidney?

• Arteriosclerosis of afferent and efferent arterioles and glomerulus-called *nephrosclerosis* • Decreased GFR and dysfunction of tubules-with eventual *renal failure*

Explain *"Nonbacterial Thromboltic Endocarditis (Marantic Endocarditis).

• Associated with debilitating illnesses such as metastatic cancer (found in up to 20% of cancer patients) • Sterile deposits of fibrin and platelets form along the closure line of cardiac valve leaflets. • Vegetations can embolize to the brain or periphery. • Although the use of heparin may be appropriate, no studies have confirmed its efficacy.

What effect does HTN have on the eyes?

• Early changes-*Arteriovenous nicking* (discontinuity in the retinal vein secondary to thickened arterial walls) and *cotton wool spots* (infarction of the nerve fiber layer in the retina) can cause visual disturbances and scotomata • More serious disease-hemorrhages and exudates • Papilledema-an ominous finding seen with severely elevated BP (indicated HTN Emergency)

Explain the D-dimer in DVT.

• Has a very high sensitivity (95%), but low specificity (50%); can be used to rule out DVT when combined with Doppler and clinical suspicion According to ULM: D-Dimer less than 500 - strong evidence against thromboembolism. But if >500 you MUST w/u patient!!!!!

What effect does HTN have on the CNS?

• Increased incidence of *intracerebral hemorrhage* • Increased incidence of other stroke subtypes as well (transient ischemic attacks [TIAs] , ischemic stroke, and lacunar stroke) • Hypertensive encephalopathy when BP is severely elevated (uncommon)

Explain Leutic Heart:

• Luetic heart is a complication of *syphilitic aortitis*, usually affecting men in their fourth to fifth decade of life. Aneurysm of the aortic arch with retrograde extension extends backward to cause aortic regurgitation and stenosis of aortic branches, most commonly the coronary arteries. • Treatment: IV penicillin and surgical repair

What is the correct procedure for blood presssure readings?

• The arm should be at heart level, and the patient should be seated comfortably. • Have the patient sit quietly for at least 5 minutes before measuring BP • Use a cuff of adequate size (a cuff that is too small can falsely elevate BP readings). The bladder within the cuff should encircle at least 80% of the arm.

Explain *Nonbacterial Verrucous Endocarditis (Libman-Sacks Endocarditis)*

• Typically involves the *aortic valves* in individuals with *SLE* • Characterized by the formation of small warty vegetations on *both sides* of valve leaflets and may present with regurgitant murmurs • Rarely gives rise to infective endocarditis, but can be a source of systematic embolization • Treat underlying SLE and anticoagulate.

What is the symptoms for "rest/continuous pain"in PVD?

• Usually felt over the distal metatarsals • Often *prominent at night-awakens patient from sleep* • Hanging the foot over side of bed or standing relieves pain-extra perfusion to ischemic areas due to gravity • Rest pain is always worrisome-suggests severe ischemia such that frank gangrene of involved limb may occur in the absence of intervention


Conjuntos de estudio relacionados

INSY 3305- Chapter 6,7,9(Comprehensive Exam 1 & 2)

View Set

Accounting 2121 Exam 1 Part 5: What is the normal balance for the following account types?

View Set

Evolve: Urinary/Reproductive System

View Set

Data Collection, Behavior, & Decisions

View Set

Concentration Mastering Chemistry

View Set

Ch. 12 Dealing with Union and Employee - Management Issues

View Set

Pharm Ch 57 Drugs Affecting Gastrointestinal Secretions

View Set

Module 5 SmartBook International Business

View Set